Sie sind auf Seite 1von 261

Table of Contents

1. Kirchhoffs Laws ................................................................................................................... 2


2. Ohms and Kirchhoffs Laws: Simple Circuits...................................................................... 9
3. Ohms and Kirchhoffs Laws: Circuits Containing Dependent Sources ............................. 19
4. Voltage and Current Division .............................................................................................. 27
5. Equivalent Resistors............................................................................................................. 35
6. Equivalent Circuits............................................................................................................... 40
7. More Voltage and Current Division..................................................................................... 50
8. Mesh and Node Equations: Simple Resistive Circuits......................................................... 55
9. Mesh and Node Equations: More Resistive Circuits ........................................................... 65
10. Mesh and Node Equations: Circuits Containing Dependent Sources................................ 74
11. Mesh and Node Equations: More Circuits Containing Dependent Sources ...................... 82
12. Source Transformations ..................................................................................................... 90
13. Superposition ................................................................................................................... 106
14. Thevenin Equivalent Circuits........................................................................................... 115
15. Op Amp Circuits .............................................................................................................. 127
16. Node Equations for Op Amp Circuits.............................................................................. 133
17. Capacitors and Inductors with Current and Voltage Represented Graphically ............... 144
18. Simple First Order Circuits .............................................................................................. 150
19. More First Order Circuits................................................................................................. 157
20. Switched First Order Circuits .......................................................................................... 163
21. Complex Arithmetic......................................................................................................... 172
22. Simple AC Circuits .......................................................................................................... 178
23. More AC Circuits............................................................................................................. 186
24. Power in AC Circuits ....................................................................................................... 197
25. AC Circuits with Coupled Inductors................................................................................ 201
26. AC Circuits with Transformers........................................................................................ 218
27. Network Functions for Simple Circuits ........................................................................... 224
28. Network Functions of Circuits Containing Dependents Sources..................................... 238
29. Complete Response of Switched Circuits........................................................................ 247


1. Kirchhoffs Laws
Introduction

The circuits in this problem set are comprised of unspecified circuit elements. (We dont know if
a particular circuit element is a resistor or a voltage source or something else.) The current and
voltage of each circuit element is labeled, sometimes as a value and sometimes as a variable.
Some of these problems ask that we determine the value of a particular voltage or current. Other
problems ask for the values of the power supplied or received by a particular circuit element.
Kirchhoffs laws are used to determine values of currents or voltages. The passive convention is
used to decide if the product of a particular element current and voltage is the power supplied or
received by the circuit element.

The passive convention is discussed in Section 1.6 of Introduction to Electric Circuits by R. C.
Dorf and J. A. Svoboda and summarized in Table 1.6-1. Kirchhoffs laws are discussed in
Section 3.3 of Introduction to Electric Circuits.


Worked Examples
Example 1:
Consider the circuit shown in Figure 1. Determine the power supplied by element D and the
power received by element F.


Figure 1. The circuit considered in Example 1

Solution: Figure 1 provides a value for the current in element D but not for the voltage, v, across
element D. The voltage and current of element D given in Figure 1 do not adhere to the passive
convention so the product of this voltage and current is the power supplied by element D.
Similarly, Figure 1 provides a value for the voltage across element F but not for the current, i, in
element F. The voltage and current of element F given in Figure 1 do adhere to the passive
convention so the product of this voltage and current is the power received by element F.

2
We need to determine the voltage, v, across element D and the current, i, in element F. We will
use Kirchhoffs laws to determine values of v and i. First, we identify and label the nodes of the
circuit as shown in Figure 2.




Figure 2. Labeling the nodes of the circuit from Figure 1.


Apply Kirchhoffs voltage law (KVL) to the loop consisting of elements C, E, D and B to get

3

+ 6

+ v + (-3) = 0 v = -6 V

The value of the current in element D in Figure 2 is 6 A. The voltage and current of element D
given in Figure 2 do not adhere to the passive convention so

p
D
= v (6) = (-6) (6) = 36 W

is the power supplied by element D. (Equivalently, we could say that element D receives 36 W.)

Next, apply Kirchhoffs current law (KCL) at node c to get

-6

+ i = 1 i = 7 A

The value of the voltage across element F in Figure 2 is -6 V. The voltage and current of element
F given in Figure 2 adhere to the passive convention so

p
F
= (-6) i = (-6) (7) = 42 W

is the power received by element F. (Equivalently, we could say that element F supplies 42 W.)
3
Example 2:
Consider the circuit shown in Figure 3. Determine the power supplied by element B and the
power supplied by element C.



Figure 3. The circuit considered in Example 2


Solution: Figure 3 provides a value for the current in element B but not for the voltage, v, across
element B. The voltage and current of element B given in Figure 1 do not adhere to the passive
convention so the product of this voltage and current is the power supplied by element B.
Similarly, Figure 3 provides a value for the voltage across element C but not for the current, i, in
element C. The voltage and current of element C given in Figure 1 do not adhere to the passive
convention so the product of this voltage and current is the power supplied by element C.

We need to determine the voltage, v, across element B and the current, i, in element C. We will
use Kirchhoffs laws to determine values of v and i. First, we identify and label the nodes of the
circuit as shown in Figure 4.




Figure 4. Labeling the nodes of the circuit from Figure 3.


Apply Kirchhoffs voltage law (KVL) to the loop consisting of elements B, C and A to get

4
-v (-3) 6 = 0 v = -3 V

The value of the current in element B in Figure 4 is 3 A. The voltage and current of element B
given in Figure 4 do not adhere to the passive convention so

p
B
= v (3) = (-3) (3) = 9 W

is the power supplied by element B. (Equivalently, we could say that element B receives 9 W.)

Next, apply Kirchhoffs current law (KCL) at node b to get

2

+ i = 3 i = 1 A

The value of the voltage across element C in Figure 4 is -3 V. The voltage and current of element
C given in Figure 4 do not adhere to the passive convention so

p
C
= (-3) i = (-3) (1) = 3 W

is the power supplied by element C. (Equivalently, we could say that element C receives 3 W.)


Example 3:
Consider the circuit shown in Figure 5. Determine the power supplied by element C and the
power received by element D.



Figure 5. The circuit considered in Example 3


Solution: Figure 5 provides a value for the current in element C but not for the voltage, v, across
element C. The voltage and current of element C given in Figure 5 adhere to the passive
convention so the product of this voltage and current is the power received by element C.
Similarly, Figure 5 provides a value for the voltage across element D but not for the current, i, in
element D. The voltage and current of element D given in Figure 5 do not adhere to the passive
convention so the product of this voltage and current is the power supplied by element D.

5
We need to determine the voltage, v, across element C and the current, i, in element D. We will
use Kirchhoffs laws to determine values of v and i. First, we identify and label the nodes of the
circuit as shown in Figure 6.



Figure 6. Labeling the nodes of the circuit from Figure 5.


Apply Kirchhoffs voltage law (KVL) to the loop consisting of elements C, D and B to get

- v (-4) 6 = 0 v = -2 V

The value of the current in element C in Figure 6 is 7 A. The voltage and current of element C
given in Figure 6 adhere to the passive convention so

p
D
= v (7) = (-2) (7) = 14 W

is the power received by element C. Therefore element C supplies 14 W.

Next, apply Kirchhoffs current law (KCL) at node b to get

7 + (-10)

+ i = 0 i = 3 A

The value of the voltage across element D in Figure 6 is -4 V. The voltage and current of element
D given in Figure 6 do not adhere to the passive convention so the power supplied by element F
is given by

p
F
= (-4) i = (-4) (3) = 12 W

Therefore, element D receives 12 W.
6
Example 4:
Consider the circuit shown in Figure 7. Determine the power supplied by element B and the
power received by element F.


Figure 7. The circuit considered in Example 4


Solution: Figure 7 provides a value for the voltage across element B but not for the current, i, in
element B. The voltage and current of element B given in Figure 7 adhere to the passive
convention so the product of this voltage and current is the power received by element B.
Similarly, Figure 7 provides a value for the current in element F but not for the voltage, v, across
element F. The voltage and current of element F given in Figure 7 do not adhere to the passive
convention so the product of this voltage and current is the power supplied by element F.

We need to determine the current, i, in element B and the voltage, v, across element F. We will
use Kirchhoffs laws to determine values of i and v. First, we identify and label the nodes of the
circuit as shown in Figure 8.



Figure 8. Labeling the nodes of the circuit from Figure 7.


7
Apply Kirchhoffs current law (KCL) at node a to get

i = -3 + (-4) + 10 i = 3 A

The value of the voltage across element B in Figure 8 is 6 V. The voltage and current of element
B given in Figure 8 adhere to the passive convention so

p
B
= (6) i = (6) (3) = 18 W

is the power received by element B. Therefore element B supplies -18 W.

Next, apply Kirchhoffs voltage law (KVL) to the loop consisting of elements D, F, E
and C to get

4

+ v + (-5) (6)

= 0 v = 7 V

The value of the current in element F in Figure 8 is 10 A. The voltage and current of element F
given in Figure 8 do not adhere to the passive convention so

p
D
= v (10) = (7) (10) = 70 W

is the power supplied by element F. Therefore element F receives -70 W.
8
2. Ohms and Kirchhoffs Laws: Simple Circuits

Introduction

The circuits in this set of problems are called simple circuits because

1. Each circuit consists of a small number of circuit elements.
2. Each circuit consists only of resistors, voltage sources, current sources, and either a
voltmeter or ammeter.
3. The voltages of the voltage sources and currents of the current sources have constant
values.

The inputs to these circuits are the voltages of the voltage sources and/or the currents of the
current sources. The output of each circuit is either a voltage measured by a voltmeter or a
current measured by an ammeter. All of the inputs have constant values. Consequently, the
outputs also have constant values.

Use Ohms and Kirchhoffs laws to solve these problems. In particular, its important to notice
that Ohms law applies to a current and voltage that adhere to the passive convention.

Ohms law is discussed in Section 2.6 of Introduction to Electric Circuits by R.C. Dorf and J.A
Svoboda. Kirchhoffs laws are discussed in Section 3.3. Voltage and current sources are
described in Section 2.6. Voltmeters and ammeters are described in Section 2.7. The passive
convention is described in Section 1.6.


Worked Examples
Example 1:
Consider the circuit shown in Figure 1. Find the value of the resistance, R. Find the power
supplied by each source and the power absorbed by the resistor.



Figure 1 The circuit considered in Example 1.

9
Solution: Figure 2 shows the circuit from Figure 1 after replacing the voltmeter by an equivalent
open circuit and labeling the voltage measured by the voltmeter.
Label the element currents and voltages as shown in Figure 3. In anticipation of using
Ohms law, the reference directions of the resistor current and voltage have been chosen to
adhere to the passive convention. Consequently, the product of the resistor current and voltage is
the power absorbed by the resistor, as required.
The reference direction for the voltage source current has been selected so that the
voltage source current and voltage do not adhere to the passive convention. Consequently, the
product of the voltage source current and voltage is the power supplied by the voltage source, as
required.


Figure 2 The circuit from Figure 1 after replacing the voltmeter by an open circuit.



Figure 3 The circuit from Figure 2 after labeling the element currents and voltages.


Similarly, the reference direction for the current source voltage has been selected so that
the current source current and voltage do not adhere to the passive convention. Consequently, the
product of the current source current and voltage is the power supplied by the current source, as
required.
Apply KVL to the loop consisting of the voltage source, the resistor and the open circuit
that replaced to voltmeter to get

56 24 0 80 V
R R
v v + + = =

Apply KCL at the top node of the current source to get

10
8 0 8 A
R R
i i + = =

Next, Ohms law gives

80
10
8
R
R
v
R
i

= = =


A


The power absorbed by the resistor is

( )( ) 8 80 640 W
R R
i v = =

Apply KCL at the node that connects the voltage source and resistor to get

0 8
a R a R
i i i i = + = =

The power supplied by the voltage source is

( ) ( )( ) 24 8 24 192 W
a
i = =

Apply KVL to the mesh consisting of the current source and the open circuit that
replaced to voltmeter to get

56 0 56 V
b b
v v = =

The power supplied by the current source is

( ) ( )( ) 8 8 56 448 W
b
v = =

As expected, the power absorbed by the resistor is equal the sum of the powers delivered by the
sources.

Example 2:
Consider the circuit shown in Figure 4. Find the value of the resistance, R. Find the value of the
power supplied by the voltage source and the power absorbed by the resistor.



Figure 4 The circuit considered in Example 2.
11

Solution: Figure 5 shows the circuit from Figure 4 after replacing the ammeter by an
equivalent short circuit and labeling the current measured by the ammeter.
Label the element currents and voltages as shown in Figure 6. In anticipation of using
Ohms law, the reference directions of the resistor current and voltage have been chosen to
adhere to the passive convention. Consequently, the product of the resistor current and voltage is
the power absorbed by the resistor, as required.
The reference direction for the voltage source current has been selected so that the
voltage source current and voltage do not adhere to the passive convention. Consequently, the
product of the voltage source current and voltage is the power supplied by the voltage source, as
required.



Figure 5 The circuit from Figure 4 after replacing the ammeter by a short circuit.



Figure 6 The circuit from Figure 5 after labeling the element currents and voltages.


Apply KVL to the mesh consisting of the voltage source, the resistor and the short circuit
that replaced to ammeter to get

0 48 0 48 V
R R
v v + + = =

Apply KCL at the right node of the resistor to get

6 6
R R
i i = = A

Next, Ohms law gives
12

48
8
6
R
R
v
R
i

= = =


A


The power absorbed by the resistor is

( )( ) 6 48 288 W
R R
i v = =

Apply KCL at the node that connects the voltage source and resistor to get

0 6
a R a R
i i i i = + = =

The power supplied by the voltage source is

( ) ( )( ) 48 6 48 288 W
a
i = =

As expected, the power absorbed by the resistor is equal the power delivered by the voltage
source.

Example 3:
Consider the circuit shown in Figure 7. Find the value of the resistance, R. Find the value of the
power supplied by each source and the power absorbed by the resistor.
.


Figure 7 The circuit considered in Example 3.


Solution: Figure 8 shows the circuit from Figure 7 after replacing the ammeter by an
equivalent short circuit and labeling the current measured by the ammeter.
Label the element currents and voltages as shown in Figure 9. In anticipation of using
Ohms law, the reference directions of the resistor current and voltage have been chosen to
adhere to the passive convention. Consequently, the product of the resistor current and voltage is
the power absorbed by the resistor, as required.


13


Figure 8 The circuit from Figure 7 after replacing the ammeter by a short circuit.



Figure 9 The circuit from Figure 8 after labeling the element currents and voltages.


The reference direction for the voltage source current has been selected so that the
voltage source current and voltage do not adhere to the passive convention. Consequently, the
product of the voltage source current and voltage is the power supplied by the voltage source, as
required.
Similarly, the reference direction for the current source voltage has been selected so that
the current source current and voltage do not adhere to the passive convention. Consequently, the
product of the current source current and voltage is the power supplied by the current source, as
required.
Apply KVL to the loop consisting of the voltage source, the resistor and the short circuit
that replaced to ammeter to get

0 12 0 12 V
R R
v v + + = =

Apply KCL at the top node of the current source to get

( ) 16 19 3 A
R R
i i = + =

Next, Ohms law gives

12
4
3
R
R
v
R
i

= = =


The power absorbed by the resistor is

14
( )( ) 3 12 36 W
R R
i v = =

Apply KCL at the node that connects the voltage source and resistor to get

0 3
a R a R
i i i i = + = = A
V


The power supplied by the voltage source is

( ) ( )( ) 12 3 12 36 W
a
i = =

Apply KVL to the mesh consisting of the current source and the short circuit that
replaced to ammeter to get

0 0 0
b b
v v = =

The power supplied by the current source is

( ) ( )( ) 16 16 0 0 W
b
v = =

As expected, the power absorbed by the resistor is equal the sum of the powers delivered by the
sources.

Example 4:
Consider the circuit shown in Figure 10. Find the value of the resistance, R. Find the value of the
power supplied by the current source and the power absorbed by the resistor.



Figure 10 The circuit considered in Example 4.


Solution: Figure 11 shows the circuit from Figure 10 after replacing the voltmeter by an
equivalent open circuit and labeling the voltage measured by the voltmeter.
Label the element currents and voltages as shown in Figure 12. In anticipation of using
Ohms law, the reference directions of the resistor current and voltage have been chosen to
adhere to the passive convention. Consequently, the product of the resistor current and voltage is
the power absorbed by the resistor, as required.
15
The reference direction for the current source voltage has been selected so that the current
source current and voltage do not adhere to the passive convention. Consequently, the product of
the current source current and voltage is the power supplied by the current source, as required.
Apply KVL to the mesh consisting of the resistor and the open circuit that replaced to
voltmeter to get
48 0 48 V
R R
v v = =

Apply KCL at the top node of the resistor to get

0 4 0 4
R R
i i + + = = A
Next, Ohms law gives

48
12
4
R
R
v
R
i

= = =


8 A


The power absorbed by the resistor is

( )( ) 4 48 192 W
R R
i v = =


Apply KVL to the mesh consisting of the current source and the resistor to get

0 4
R a a R
v v v v + = = =


The power supplied by the voltage source is

( ) ( )( ) 4 4 48 192 W
a
v = =

As expected, the power absorbed by the resistor is equal the power delivered by the current
source.





Figure 11 The circuit from Figure 10 after replacing the voltmeter by an open circuit.


16


Figure 12 The circuit from Figure 11 after labeling the element currents and voltages.
17
18
3. Ohms and Kirchhoffs Laws: Circuits Containing Dependent
Sources

Introduction

Each circuit in this set of problems:

1. consists of a small number of circuit elements.
2. consists of a dependent source, some resistors, voltage sources, current sources, and
either a voltmeter or ammeter.
3. has constant-valued inputs.

The inputs to these circuits are the voltages of the voltage sources and/or the currents of the
current sources. The output of each circuit is either a voltage measured by a voltmeter or a
current measured by an ammeter. All of the inputs have constant values. Consequently, the
outputs also have constant values.

Use Ohms and Kirchhoffs laws to solve these problems.

Ohms law is discussed in Section 2.6 of Introduction to Electric Circuits by R.C. Dorf and J.A
Svoboda. Kirchhoffs laws are discussed in Section 3.3. Independent voltage and current sources
are described in Section 2.6. Dependent voltage and current sources are described in Section 2.8.
Voltmeters and ammeters are described in Section 2.7.

Worked Examples
Example 1:
Consider the circuit shown in Figure 1. Find the value of the gain, A, of the CCCS (Current
Controlled Current Source).



Figure 1 The circuit considered in Example 1.
19


Solution: Figure 2 shows the circuit from Figure 1 after replacing the ammeter by an
equivalent short circuit and labeling the current measured by the ammeter. Also, the voltage
across the short circuit is labeled.
Figure 3 shows the circuit after labeling the element currents and voltages as follows.
Apply KCL at the bottom node of the voltage source and again at the top node of the voltage
source to see that the current in the voltage source and the 16 resistor are both equal to i
a
, as
labeled in Figure 3. Apply KCL at the right node of the 14 resistor to see that the current in the
14 resistor is 2 A, as labeled in Figure 3. Apply Ohms law to see that the voltage across the
14 resistor is 28 V, as labeled in Figure 3. (Since Ohms law was used, the reference
direction of the voltage across the 14 resistor was chosen so that the reference directions of the
voltage and current of the 14 resistor adhere to the passive convention.) The voltage across the
16 resistor is labeled as v
b
. (In anticipation of using Ohms law, the reference direction of the
voltage across the 16 resistor was chosen so that the reference directions of the voltage and
current of the 16 resistor adhere to the passive convention.)





Figure 2 The circuit from Figure 1 after replacing the ammeter by a short circuit.





Figure 3 The circuit from Figure 2 after labeling the element currents and voltages.

Apply Ohms law to the 16 resistor to get
20

16
b a
v i =

Apply KVL to the loop consisting of the voltage source, the two resistors and the short circuit
that replaced to ammeter to get

( ) 28 0 12 0 16 V 1 A
b b
v v + + + = = =
a
i
A/A

Apply KCL at the top node of the dependent current source to get

( ) 2 ( 1) 1 2 3
a a
Ai i A A = + = =


Example 2:
Consider the circuit shown in Figure 4. Find the value voltage measured by the voltmeter.



Figure 4 The circuit considered in Example 2.

Solution: See Example 3.3-4 in Introduction to Electric Circuits by R.C. Dorf and J.A Svoboda.
21
Example 3:
Consider the circuit shown in Figure 5. Find the value of gain, A, of the CCVS. (Current
Controlled Voltage Source).



Figure 5 The circuit considered in Example 3.


Solution: Figure 6 shows the circuit from Figure 5 after replacing the voltmeter by an
equivalent open circuit and labeling the voltage measured by the voltmeter.
Figure 7 shows the circuit after labeling the element currents and voltages as follows.
Apply KCL at the bottom node of the voltage source and again at the top node of the voltage
source to see that the current in the voltage source and the 8 resistor are both equal to i
a
, as
labeled in Figure 7. Apply KCL at the right node of the 7 resistor to see that the current in the
7 resistor is 0 A, as labeled in Figure 7. Apply Ohms law to see that the voltage across the 7
resistor is 0 V, as labeled in Figure 7. The voltage across the 8 resistor is labeled as v
b
. (In
anticipation of using Ohms law, the reference direction of the voltage across the 8 resistor
was chosen so that the reference directions of the voltage and current of the 8 resistor adhere
to the passive convention.)
Apply Ohms law to the 8 resistor to get

8
b a
v i =

Apply KVL to the loop consisting of the voltage source, the two resistors and the open circuit
that replaced to voltmeter to get

0 3.27 12 0 8.73 V 1.09 A
b b
v v + + = = =
a
i

Apply KVL to the mesh consisting of the voltage source, the 8 resistor and the dependent
voltage source to get

( ) ( ) 12 0 8.73 12 1.09 3 V/A
b a
v Ai A A = = =

22


Figure 6 The circuit from Figure 5 after replacing the voltmeter by an open circuit.



Figure 7 The circuit from Figure 6 after labeling the element currents and voltages.


Example 4:
Consider the circuit shown in Figure 8. Find the value of gain, A, of the VCCS (Voltage
Controlled Current Source).



Figure 8 The circuit considered in Example 4.

23
Solution: Figure 9 shows the circuit from Figure 8 after replacing the ammeter by an
equivalent short circuit and labeling the current measured by the ammeter. Also, the voltage
across the short circuit is labeled.
Figure 10 shows the circuit after labeling the element currents and voltages as follows.
Apply KCL at the bottom node of the current source and again at the top node of the current
source to see that the current in the 6 resistor and the 16 resistor are both equal to 0.2 A as
labeled in Figure 10. Apply Ohms law to see that the voltage across the 6 resistor is 1.2 V and
that the voltage across the 16 resistor is 3.2 V, as labeled in Figure 10. (Since Ohms law was
used, the reference direction of the voltage across each resistor was chosen so that the reference
directions of the voltage and current of each resistor adhere to the passive convention.)




Figure 9 The circuit from Figure 8 after replacing the ammeter by a short circuit.



Figure 10 The circuit from Figure 9 after labeling the element currents and voltages.


Apply KVL to the loop consisting of the voltage source, the two resistors and the short
circuit that replaced to ammeter to get

1.2 0 3.2 0 4.4 V
a a
v v + + = =

Apply KCL at top node of the dependent current source to get

( ) ( ) 0.2 1.11 4.4 1.31 0.3 A/V
a
A v A A = + = =

24
Example 5:
Consider the circuit shown in Figure 11. Find the value of the gain, A, of the VCVS (Voltage
Controlled Voltage Source).



Figure 11 The circuit considered in Example 5.


Solution: Figure 12 shows the circuit from Figure 11 after replacing the voltmeter by an
equivalent open circuit and labeling the voltage measured by the voltmeter.
Figure shows the circuit after labeling the element currents and voltages as follows.
Apply KCL at the bottom node of the current source and again at the top node of the current
source to see that the current in the 3 resistor and the 8 resistor are both equal to 5 A as
labeled in Figure 13. Apply Ohms law to see that the voltage across the 3 resistor is 15 V and
that the voltage across the 8 resistor is 40 V, as labeled in Figure12. (Since Ohms law was
used, the reference direction of the voltage across each resistor was chosen so that the reference
directions of the voltage and current of each resistor adhere to the passive convention.)





Figure 12 The circuit from Figure 11 after replacing the voltmeter by an open circuit.

25


Figure 13 The circuit from Figure 12 after labeling the element currents and voltages.


Apply KVL to the loop consisting of the voltage source, the two resistors and the open
circuit that replaced to voltmeter to get

15 47.14 40 0 7.86 V
a a
v v + = =

Apply KVL to the loop consisting of the dependent voltage source and the open circuit that
replaced to voltmeter to get

( ) 47.14 0 47.14 7.86 47.14 6 V/V
a a
A v A v A A + + = = = =

Example 6:
Consider the circuit shown in Figure 14. Find the value current measured by the ammeter.



Figure 14 The circuit considered in Example 6.

Solution: See Example 3.3-4 in Introduction to Electric Circuits by R.C. Dorf and J.A Svoboda
26
4. Voltage and Current Division
Introduction

The circuits in this problem set consist of a single voltage or current source, some resistors and a
voltmeter or ammeter. The input to each circuit is the voltage of the voltage source or the current
of the current source. The output of each circuit is either the voltage measured by the voltmeter
or the current measured by the ammeter. All of the inputs have constant values. Consequently,
the outputs also have constant values. The output of each circuit is proportional to the input to
that circuit.

Each circuit includes resistors that are connected either in series or in parallel. The problems in
this problem set can be solved using the principles of voltage division, current division or
equivalent resistance.

Series resistors and voltage division are discussed in Section 3.4 of Introduction to Electric
Circuits by R.C. Dorf and J.A Svoboda. Parallel resistors and current division are discussed in
Section 3.4. These ideas are summarized in Table 3.12-1 of Introduction to Electric Circuits.

Worked Examples
Example 1:
Consider the voltage divider circuit shown in Figure 1. Find the value of the resistance R.
Determine the power supplied by the voltage source.



Figure 1 The circuit considered in Example 1.

Solution: The voltmeter measures the voltage across the 4 resistor to be 6 V. (The color
coded probes of the voltmeter indicate the reference direction of the voltage measured by the
voltmeter.) Figure 2a shows the circuit after the replacing the voltmeter by the equivalent open
circuit and adding a label to show the voltage measured by the meter.
Figure 2b shows the voltage divider circuit again, this time with the voltage across the 4
resistor labeled as v
a
. The voltage division equation gives

27

4
15
4
a
v
R
| |
=
|
+
\ .
(1)

The reference direction for v
a
, + on the top, is the opposite of the reference direction for the
voltage measured by the voltmeter, + on the bottom. Thus . Substituting this
value into equation 1 gives
( 6) 6 V
a
v = =

4
6 15
4
R
R
| |
= =
|
+
\ .
6


In Figure 2c the currents in the voltage source and the two resistors have been labeled. With the
reference directions selected as shown in Figure 2c, these three currents are equal. Consequently,
all three currents have been given the same name, i. The value of the current i can be determined
by applying Ohms Law to the 4 resistor:


6
1.5 A
4
i

= =


The current i, and voltage 15 V, of the voltage source adhere to the passive convention. The
product of this current and voltage is the power received by the voltage source. The power
supplied by the voltage source is the negative of the power received by the voltage source. The
power supplied by the voltage source is -(-1.5) (15) = 22.5 W.



(a)


(b)


(c)
Figure 2 (a) The circuit of Figure 1 after replacing the voltmeter by the equivalent open circuit.
(b) The circuit labeled differently. (c) The circuit of Figure 1 after labeling the element currents.
28
Example 2:
Consider the current divider circuit shown in Figure 3. Find the value of the resistance R.
Determine the power supplied by the current source.



Figure 3 The circuit considered in Example 2.


Solution: The ammeter measures the current in the 40 resistor to be 0.4 A. (The color coded
probes of the ammeter indicate the reference direction of the current measured by the ammeter.)
Figure 4a shows the circuit after the replacing the ammeter by the equivalent short circuit and
adding a label to show the current measured by the meter.
Figure 4b shows the current divider circuit again, this time with the current the 40
resistor labeled as i
a
. The current division equation gives

2
40
a
R
i
R
|
=

+
\ .
|
|
(2)

The reference direction for i
a
, downward, is the opposite of the reference direction for the current
measured by the ammeter, upward. Thus . Substituting this value into
equation 2 gives
( 0.4) 0.4 A
a
i = =

0.4 2 10
40
R
R
R
| |
= =
|
+
\ .


In Figure 4c the voltages across the current source and the two resistors have been labeled. With
the reference directions selected as shown in Figure 4c, these three voltages are equal.
Consequently, all three voltages have been given the same name, v. The value of the voltage v
can be determined by applying Ohms Law to the 40 resistor:

( )( ) 0.4 40 16 V v = =

The voltage v, and current 2 A, of the current source adhere to the passive convention. The
product of this current and voltage is the power received by the current source. The power
supplied by the current source is the negative of the power received by the current source. The
power supplied by the current source is -(2) (-16) = 32 W.
29



(a)


(b)




(c)

Figure 4 (a) The circuit of Figure 3 after replacing the ammeter by the equivalent short circuit.
(b) The circuit labeled differently. (c) The circuit of Figure 3 after labeling the element voltages.


30
Example 3:
Consider the circuit shown in Figure 5. Find the value of the resistance R.




Figure 5 The circuit considered in Example 3.


Solution: The ammeter measures the current in the series resistors to be 2 A. (The color coded
probes of the ammeter indicate the reference direction of the current measured by the ammeter.)
Figure 6 shows the circuit after the replacing the series resistors by a single equivalent resistor,
replacing the ammeter by an equivalent short circuit and adding a label to show the current
measured by the meter.




Figure 6 A circuit that is equivalent to the circuit of Figure 5.


The equivalent resistance is given by the sum of the series resistances:

(3) 6
eq
R R = +

Applying KVL to the single mesh circuit in Figure 6 gives

( ) 2 30 0 15
eq eq
R R = =

Substituting this value into Equation 3 gives R = 9 .

31
Example 4:
Consider the circuit shown in Figure 7. Find the value of the resistance R.



Figure 7 The circuit considered in Example 4.


Solution: The voltmeter measures the voltage across the parallel resistors to be 24 V. (The
color coded probes of the voltmeter indicate the reference direction of the voltage measured by
the voltmeter.) Figure 8 shows the circuit after the replacing the parallel resistors by a single
equivalent resistor, replacing the voltmeter by an equivalent open circuit and adding a label to
show the voltage measured by the meter.



Figure 8 A circuit that is equivalent to the circuit of Figure 7.


The equivalent resistance is given by

( ) 40
40
eq
R
R
R
=
+
(4)

Applying KCL at the top node of the circuit in Figure 8 gives


24
3 0
eq
eq
R
R

+ = = 8

Substituting this value into Equation 4 gives

( )
( )( )
40
8 8 8 40 40
40
R
R R R
R
= + = =
+
10
32
Example 5:
Consider the circuit shown in Figure 9. Find the values of the resistance R and of the current, i.



Figure 9 The circuit considered in Example 5.

Solution: See Example 3.4-1 in Introduction to Electric Circuits by R.C. Dorf and J.A Svoboda.


Example 6:
Consider the circuit shown in Figure 10. Find the value of the current measured by the meter.
Show that the sum of the power absorbed by the two resistors is equal to the power supplied by
the voltage source.




Figure 10 The circuit considered in Example 6.


Solution: See Example 3.4-2 in Introduction to Electric Circuits by R.C. Dorf and J.A Svoboda.


33
Example 7:
Consider the circuit shown in Figure 11. Find the value of the voltage measured by the meter.
Show that the sum of the power absorbed by the two resistors is equal to the power supplied by
the current source.



Figure 11 The circuit considered in Example 7.


Solution: See Example 3.5-2 in Introduction to Electric Circuits by R.C. Dorf and J.A Svoboda.
34
5. Equivalent Resistors

Introduction

The circuits in this set of problems consist of several resistors and an ohmmeter. The resistors are
connected together to form a resistor sub-circuit. The ohmmeter measures the equivalent
resistance of the resistor sub-circuit.

These circuits can be simplified by repeatedly replacing series or parallel resistors by an
equivalent resistor. Eventually, the resistor sub-circuit is reduced to a single equivalent resistor.
The ohmmeter measures the resistance of that single equivalent resistor.

(What is a sub-circuit? Its part of a circuit. What is the equivalent resistance of a resistor
sub-circuit? When we reduce the resistor sub-circuit to a single equivalent resistor as described
above, the equivalent resistance is the resistance of the equivalent resistor.)

Series resistors are discussed in Section 3.4 of Introduction to Electric Circuits by R.C. Dorf and
J.A Svoboda. Parallel resistors are discussed in Section 3.5. Equivalent resistance is described in
Section 3.7.


Worked Examples
Example 1:
Consider the circuit shown in Figure 1. Find the value of the resistance R.





Figure 1 The circuit considered in Example 1.


Solution: The terminals in Figure 1 divide the circuit into two parts, the part to the left of the
terminals and the part to the right of the terminals. The part to the left of the terminals consists of
four resistors. The resistor labeled R is connected in series with the 10 resistor and that series
35
combination is connected in parallel with the 36 resistor. The parallel combination of resistors
is, in turn, connected in series with the 7 resistor. These four resistor could be replaced by a
single equivalent resistor. The Ohmmeter measures the resistance of that equivalent resistor.
The resistance of the equivalent resistor is given by

( ) ( )
( )( )
( )
10 36
10 || 36 7 7
10 36
eq
R
R R
R
+
= + + = +
+ +


The Ohmmeter in Figure 1 measures the value of the equivalent resistance to be 19 . Therefore


( )( )
( )
10 36
7 19
10 36
R
R
+
+ =
+ +

Doing some algebra:

( )(
( )( )
36 360
12 36 360 12 12 46
46
24 12 46 360
R
R R
R
R
+
= + = +
+
=
)


Solving this equation gives R = 8 .


Example 2:
Consider the circuit shown in Figure 2. Find the value of the resistance R.



Figure 2 The circuit considered in Example 2.


Solution: The terminals in Figure 2 divide the circuit into two parts, the part to the left of the
terminals and the part to the right of the terminals. The part to the left of the terminals consists of
four resistors. The 12 resistor is connected in parallel with the 24 resistor. The parallel
combination of resistors is, in turn, connected in series with the 6 resistor and with resistor
labeled R. These four resistor could be replaced by a single equivalent resistor. The Ohmmeter
measures the resistance of that equivalent resistor.
36
The resistance of the equivalent resistor is given by

( )
( )( ) 12 24
12 || 24 6 6 8 6 14
12 24
eq
R R R R = + + = + + = + + = +
+
R

The Ohmmeter in Figure 2 measures the value of the equivalent resistance to be 17 . Therefore

14 17 R + =

Solving this equation gives R = 3 .



Example 3:
Consider the circuit shown in Figure 3. Find the value of the resistance R.



Figure 3 The circuit considered in Example 3.


Solution: The terminals in Figure 3 divide the circuit into two parts, the part to the left of the
terminals and the part to the right of the terminals. The part to the left of the terminals consists of
four resistors. The 9 resistor, the 10 resistor and the 17 resistor are connected in series.
That series combination of resistors is, in turn, connected in parallel with resistor labeled R.
These four resistor could be replaced by a single equivalent resistor. The Ohmmeter measures
the resistance of that equivalent resistor.
The resistance of the equivalent resistor is given by

( )
( )
( )
( )
( )
9 10 17 36
9 10 17 ||
9 10 17 36
eq
R R
R R
R R
+ +
= + + = =
+ + + +


The Ohmmeter in Figure 3 measures the value of the equivalent resistance to be 24 . Therefore

37

( )
( )
( )(
( )( )
36
24 36 24 24 36
36
12 24 36
R
R R
R
R
= = +
+
=
)


Solving this equation gives R = 72 .


Example 4:
Consider the circuit shown in Figure 4. Find the value of the resistance R.



Figure 4 The circuit considered in Example 4.


Solution: The terminals in Figure 4 divide the circuit into two parts, the part to the left of the
terminals and the part to the right of the terminals. The part to the left of the terminals consists of
four resistors. The 144 resistor is connected in parallel with the 72 resistor. That parallel
combination of resistors is connected in series with the 96 resistor. The series combination of
resistors is, in turn, connected in parallel with resistor labeled R. These four resistor could be
replaced by a single equivalent resistor. The Ohmmeter measures the resistance of that
equivalent resistor.
The resistance of the parallel combination of the 144 resistor and the 72 resistor is
given by


( )( ) 144 72
144 || 72 48
144 72
= =
+


Next, the resistance of the equivalent resistor is given by

( )
( )
( )
48 96
48 96 ||
48 96
eq
R
R R
R
+
= + =
+ +


The Ohmmeter in Figure 4 measures the value of the equivalent resistance to be 96 . Therefore

38

( )
( )
( )(
144
96 144 96 144 96
144
R
R R
R
= = +
+
)

Solving this equation gives R = 288 .


Example 5:
Consider the circuit shown in Figure 5. Find the value of the resistance measured by the
ohmmeter.



Figure 5 The circuit considered in Example 5.


Solution: See Example 3.7-2 in Introduction to Electric Circuits by R.C. Dorf and J.A Svoboda.
39
6. Equivalent Circuits
Introduction

The circuits in this set of problems consist of a voltage or current source and several resistors.
The resistors are connected together to form a resistor sub-circuit. These circuits can be
simplified by repeatedly replacing series or parallel resistors by an equivalent resistor.
Eventually, the resistor sub-circuit is reduced to a single equivalent resistor.

In each problem we are asked to determine the values of three currents or voltages. These
currents or voltages are identified by the subscripts a, b and c. The computer will guide us to a
solution in three steps:

1. Reduce the resistor sub-circuit to a single resistor.

2. Analyze the reduced circuit, using Ohms law, to find the resistor current and voltage.
Then determine the values of the source current and voltage in the reduced circuit. The
values of the source current and voltage in the original circuit are the same as the values
of the source current and voltage in the reduced circuit.

3. Complete the analysis of the original circuit using voltage or current division.

Series resistors are discussed in Section 3.4 of Introduction to Electric Circuits by R.C. Dorf and
J.A Svoboda. Parallel resistors are discussed in Section 3.5. Circuit analysis using equivalent
resistances is described in Section 3.7.


Worked Examples
Example 1:
Consider the circuit shown in Figure 1. Find the values of the voltage v
c
and the current i
b
.



Figure 1 The circuit considered in Example 1.

40

Solution: The terminals in Figure 1 divide the circuit into two parts, the part to the left of the
terminals and the part to the right of the terminals. The part to the right of the terminals consists
of three resistors. The 10 resistor is connected in parallel with the 40 resistor and that
parallel combination is connected in series with the 4 resistor. These three resistors can be
replaced by a single equivalent resistor as shown in Figure 2a. The resistance of the equivalent
resistor is given by


( )( ) 10 40
4 4 8
10 40
eq
R = + = + =
+
12

The current in the equivalent resistance is determined using Ohms law to be


24
2 A
12
a
i = =

The values of the equivalent resistance and the current i
a
are labeled in Figure 2b.

The circuit shown in Figure 2b is equivalent to the circuit shown in Figure 1.
Consequently, the value of the current i
a
in Figure 1 is equal to the value of the current i
a
in
Figure 2b. Figure 2c shows the circuit from Figure 1 after labeling the value of the current i
a
.
Now the voltage v
c
can be calculated using Ohms law to be

( )( ) 2 4 8 V
c
v = =

The current i
b
can be calculated using current division to be

( )
10
2 0.4 A
10 40
b
i = =
+


Figure 2d shows the circuit from Figure 1 after labeling the values of the voltage v
c
and the
current i
b
.
41



(a)


(b)



(c)



(d)

Figure 2 (a) The circuit of Figure 1 after replacing the resistors by a single equivalent resistor.
(b) The values of the equivalent resistance and the current i
a
. (c) The circuit of Figure 1 after
labeling the value of the current i
a
. (d) The values of the voltage v
c
and the current i
b
.

42
Example 2:
Consider the circuit shown in Figure 3. Find the values of the voltage v
c
and the current i
b
.



Figure 3 The circuit considered in Example 2.



Solution: The terminals in Figure 3 divide the circuit into two parts, the part to the left of the
terminals and the part to the right of the terminals. The part to the right of the terminals consists
of three resistors. The 4 resistor is connected in series with the 2 resistor and that series
combination is connected in parallel with the 3 resistor. These three resistors can be replaced
by a single equivalent resistor as shown in Figure 4a. The resistance of the equivalent resistor is
given by


( )( )
( )
3 4 2
18
2
3 4 2 9
eq
R
+
= = =
+ +


The voltage across the equivalent resistance is determined using Ohms law to be

( )( ) 6 2 6 V
a
v = =

The values of the equivalent resistance and the voltage v
a
are labeled in Figure 4b.
The circuit shown in Figure 4b is equivalent to the circuit shown in Figure 3.
Consequently, the value of the voltage v
a
in Figure 3 is equal to the value of the voltage v
a
in
Figure 4b. Figure 4c shows the circuit from Figure 3 after labeling the value of the voltage v
a
.
Now the current i
b
can be calculated using Ohms law to be


6
2 A
3
b
i = =

The voltage v
c
can be calculated using voltage division to be

( )
4
6 4
4 2
c
v
| |
= =
|
+
\ .
V

43
Figure 4d shows the circuit from Figure 3 after labeling the values of the voltage v
c
and the
current i
b
.






(a)



(b)


(c)


(d)

Figure 4 (a) The circuit of Figure 3 after replacing the resistors by a single equivalent resistor.
(b) The values of the equivalent resistance and the voltage v
a
. (c) The circuit of Figure 3 after
labeling the value of the voltage v
a
. (d) The values of the voltage v
c
and the current i
b
.

44
Example 3:
Consider the circuit shown in Figure 5. Find the values of the voltage v
c
and the current i
b
.



Figure 5 The circuit considered in Example 3.


Solution: The terminals in Figure 5 divide the circuit into two parts, the part to the left of the
terminals and the part to the right of the terminals. The part to the right of the terminals consists
of three resistors. A 10 resistor is connected in series with the 30 resistor and that series
combination is connected in parallel with the other 10 resistor. These three resistors can be
replaced by a single equivalent resistor as shown in Figure 6a. The resistance of the equivalent
resistor is given by


( )( )
( )
10 30 10
400
8
10 30 10 50
eq
R
+
= =
+ +
=

The current in the equivalent resistance is determined using Ohms law to be


40
5 A
8
a
i = =

The values of the equivalent resistance and the current i
a
are labeled in Figure 6b.
The circuit shown in Figure 6b is equivalent to the circuit shown in Figure 5.
Consequently, the value of the current i
a
in Figure 5 is equal to the value of the current i
a
in
Figure 6b. Figure 6c shows the circuit from Figure 5 after labeling the value of the current i
a
.
Consider the 10 resistor having current i
b.
The voltage across this resistor is 40 V.
Notice that the current i
b
and the voltage 40 V do not adhere to the passive convention. The
current i
b
can be calculated using Ohms law to be


40
4 A
10
b
i = =

The voltage v
c
can be calculated using voltage division to be

45
( )
30
40 30 V
30 10
c
v
| |
= =
|
+
\ .


Figure 6d shows the circuit from Figure 5 after labeling the values of the voltage v
c
and the
current i
b
.






(a)



(b)



(c)



(d)

Figure 6 (a) The circuit of Figure 5 after replacing the resistors by a single equivalent resistor.
(b) The values of the equivalent resistance and the current i
a
. (c) The circuit of Figure 5 after
labeling the value of the current i
a
. (d) The values of the voltage v
c
and the current i
b
.

46
Example 4:
Consider the circuit shown in Figure 7. Find the values of the voltage v
c
and the current i
b
.



Figure 7 The circuit considered in Example 4.


Solution: The terminals in Figure 7 divide the circuit into two parts, the part to the left of the
terminals and the part to the right of the terminals. The part to the right of the terminals consists
of three resistors. The 3 resistor is connected in parallel with the 6 resistor and that parallel
combination is connected in series with the 4 resistor. These three resistors can be replaced by
a single equivalent resistor as shown in Figure 8a. The resistance of the equivalent resistor is
given by


( )( ) 6 3
4 4 2
6 3
eq
R = + = + =
+
6

The voltage across the equivalent resistor is v
a
and the value of the current in the equivalent
resistor is 6 A. Notice that this current and voltage do not adhere to the passive convention. The
voltage across the equivalent resistance is determined using Ohms law to be

( )( ) 6 6 36 V
a
v = =

The values of the equivalent resistance and the voltage v
a
are labeled in Figure 8b.
The circuit shown in Figure 8b is equivalent to the circuit shown in Figure 7.
Consequently, the value of the voltage v
a
in Figure 7 is equal to the value of the voltage v
a
in
Figure 8b. Figure 8c shows the circuit from Figure 7 after labeling the value of the voltage v
a
.
The voltage across the 4 resistor is v
c
and the value of the current in the 4 resistor is
6 A. Notice that this current and voltage do not adhere to the passive convention. The voltage v
c

can be calculated using Ohms law to be

( )( ) 6 4 24 V
c
v = =

The current i
b
can be calculated using current division to be

47
( )
6
6 4
6 3
b
i
| |
= =
|
+
\ .
A

Figure 8d shows the circuit from Figure 7 after labeling the values of the voltage v
c
and the
current i
b
.






(a)



(b)


(c)


(d)

Figure 8 (a) The circuit of Figure 7 after replacing the resistors by a single equivalent resistor.
(b) The values of the equivalent resistance and the voltage v
a
. (c) The circuit of Figure 7 after
labeling the value of the voltage v
a
. (d) The values of the voltage v
c
and the current i
b
.

48
49
7. More Voltage and Current Division

Introduction

The circuits in this problem set consist of a single voltage or current source, some resistors, a
dependent source and a voltmeter or ammeter. The input to each circuit is the voltage of the
voltage sources or the current of the current source. The output of each circuit is either the
voltage measured by the voltmeter or the current measured by the ammeter. All of the inputs
have constant values. Consequently, the outputs also have constant values. The output of each
circuit is proportional to the input to that circuit.

Each circuit includes resistors that are connected either in series or in parallel. The problems in
this problem set can be solved using the principles of voltage division and/or current division.

Series resistors and voltage division are discussed in Section 3.4 of Introduction to Electric
Circuits by R.C. Dorf and J.A Svoboda. Parallel resistors and current division are discussed in
Section 3.4. These ideas are summarized in Table 3.12-1. Dependent sources are described in
Section 2.8 of Introduction to Electric Circuits.


Worked examples
Example 1:
Consider the circuit shown in Figure 1. Determine the voltage measured by the voltmeter.



Figure 1 The circuit considered in Example 1

Solution: The 10 and 40 resistors are connected in parallel. Noticing the reference
directions of the voltage v
a
and the current source current, we use Ohms law to write

50
( )
a
10 40
40 || 10 3 3 24 V
10 40
v
| |
= = =
|
+
\ .


Next, denote the voltage measured by the voltmeter as v
m
and use voltage division in the right
part of the circuit to get

( ) ( )
m a a
3 5 5
5 2
15 3 6 6
v v v
| |
= = = =
|
+
\ .
4 20 V

Example 2:
The input to the circuit shown in Figure 2 is the voltage of the voltage source, v
s
. The output is
the voltage measured by the meter, v
m
. The output is proportional to the input, that is

v
m
= k v
s
.

Determine the value of the resistance R that causes the constant of proportionality to be k = 4
V/V.



Figure 2 The circuit considered in Example 2

Solution: Use voltage division in the left part of the circuit to get

18 3
12 18 5
a s
v v
| |
= =
|
+
\ .
s
v

Next, use voltage division in the right part of the circuit to get

( )
m a
20 240
12
20 20
v v
R R
= =
+ +
a
v

Combining these equations gives:

m
240 3 144
20 5 20
s s
v v
R R
| | | |
= =
| |
+ +
\ . \ .
v
51
We require
144
4 4 80 144 16
20
R R
R
= + = =
+


Example 3:
Consider the circuit shown in Figure 3. Determine the value of the resistance R that causes the
value of the voltage measured by the meter to be 4 V.



Figure 3 The circuit considered in Example 3


Solution: Use current division in the left part of the circuit to get

( )
a
40
3 2.4 A
40 10
i
| |
= =
|
+
\ .


Next, denote the voltage measured by the voltmeter as v
m
and use voltage division in the right
part of the circuit to get
( )
m a
5
5
18 18
R R
v i
R R
| | | |
= =
| |
+ +
\ . \ .
a
i

Combining these equations gives:
( )
m
5 1
2.4
18 18
2 R R
v
R R
| |
= =
|
+ +
\ .


When v
m
= 4 V,
12 4 18
4 9
18 12 4
R
R
R

= = =
+

52
Example 4:
Consider the circuit shown in Figure 4. Determine the current measured by the ammeter.



Figure 4 The circuit considered in Example 4

Solution: Use current division in the left part of the circuit to get

( )
a
8
3 1
8 16
i
| |
= =
|
+
\ .
A

Next, denote the current measured by the ammeter as i
m
and use current division in the right part
of the circuit to get
( )
m a
16
5 4
16 4
i i
| |
= =
|
+
\ .
a
i

Combining these equations gives:
i
m
= -4 A

Example 5:
The input to the circuit shown in Figure 5 is the voltage of the voltage source, v
s
. The output is
the current measured by the meter, i
m
.

a. Suppose v
s
= 15 V. Determine the value of the resistance R that causes the value of the
current measured by the meter to be i
m
= 5 A.

b. Suppose v
s
= 15 V and R = 24 . Determine the current measured by the ammeter.

c. Suppose R = 24 . Determine the value input voltage, v
s
, that causes the value of the
current measured by the meter to be i
m
= 3 A.


53


Figure 5 The circuit considered in Example 5


Solution: Use voltage division in the left part of the circuit to get

( )
a s
12 2
12 18 5
v v
| |
= =
|
+
\ .
s
v

Next, use current division in the right part of the circuit to get

( )
m a
16 80
5
16 16
i v
R R
| | | |
= =
| |
+ +
\ . \ .
a
v

Combining these equations gives:
m
80 2 32
16 5 16
s s
i v
R R
| | | | |
= =
| |
+ +
\ . \ . \
v
|
|
.


a. When v
s
= 15 V and i
m
= 5 A

32 400
5 15 80 5 480 80
16 5
R R
R
| |
= + = = =
|
+
\ .


b. When v
s
= 15 V and R = 24
m
32
15 12 A
16 24
i
| |
= =
|
+
\ .


c. When i
m
= 3 A and R = 24
54
8. Mesh and Node Equations: Simple Resistive Circuits
Introduction

The circuits in this set of problems are small circuits that consist of independent sources,
resistors and a meter. In particular, these circuits do not contain dependent sources. These
circuits can be analyzed using mesh equation or using node equations.

Node equations are discussed in Sections 4.3 and 4.4 of Introduction to Electric Circuits by R.C.
Dorf and J.A Svoboda. Mesh equations are discussed in Section 4.6.


Worked Examples
Example 1:
Consider the circuit shown in Figure 1. Find the value of the current source current, I
a
.



Figure 1 The circuit considered in Example 1.


Solution: Figure 2 shows the circuit from Figure 1 after replacing the ammeter by an equivalent
short circuit and labeling the current measured by the ammeter. This circuit can be analyzed
using mesh equations or using node equations. We will do both.



Figure 2 The circuit from Figure 1 after replacing the ammeter by a short circuit.
55



Figure 3 The circuit from Figure 2 after labeling the mesh currents.



1
First, consider analyzing the circuit in Figure 2 using mesh equations. Figure 3 shows the
circuit after labeling the mesh currents, i
1
and i
2
. The mesh current i
2
is equal to the current in the
short circuit that replaced the ammeter so


2
3 A i =

The current in the current source can be expressed in terms of the mesh currents as


2 1
3
a
I i i i = = (1)

Apply KVL to the supermesh to get

( )( )
1 1
2 2 3 4 0 1 i i + = = A
V

Substituting this value into Equation 1 gives . 3 ( 1) 4 A
a
I = =
Next, consider analyzing the circuit in Figure 2 using node equations. Figure 4 shows the
circuit after selecting a reference node and numbering the other nodes. Let v
1
and v
2
denote the
node voltages at node 1 and node 2, respectively.
The voltage of the voltage source can be expressed in terms of the node voltages as


1 1
4 0 4 v v = =
The current in the short circuit that replaced the ammeter is equal to the current in one of the 2
resistors. This current can be expressed in terms of the node voltages as


2
2
0
3 6
2
v
v

= = V

Apply KCL at node 2 to get


2 1
6 4
3 3
2 2
a
v v
I

= + = + = 4 A
56

This value of current source current agrees with the value calculated using mesh equations, as it
must.




Figure 4 The circuit from Figure 2 after labeling the nodes. (The encircled numbers are node
numbers.)



Example 2:
Consider the circuit shown in Figure 5. Find the value of the current source current, I
a
.



Figure 5 The circuit considered in Example 2.


Solution: Figure 6 shows the circuit from Figure 5 after replacing the voltmeter by an equivalent
open circuit and labeling the voltage measured by the voltmeter. This circuit can be analyzed
using mesh equations or using node equations. We will do both.
First, consider analyzing the circuit in Figure 6 using mesh equations. Figure 7 shows the
circuit after labeling the mesh currents, i
1
and i
2
.


57


Figure 6 The circuit from Figure 5 after replacing the voltmeter by a open circuit.



Figure 7 The circuit from Figure 6 after labeling the meshes.


The mesh current i
2
is equal to the current in the 2 resistor at the right of the circuit. The
voltmeter measured to voltage across that 2 resistor to be 9 V. Using Ohms Law, the mesh
current is given by

2
9
4.5 A
2
i = =

The current in the current source can be expressed in terms of the mesh currents as


2 1
4.5
a 1
I i i i = = (2)
Apply KVL to the super mesh to get

( )( )
1 1
8 2 4.5 9 6 0 1.5 A i i + + = =

Substituting this value into Equation 2 gives . 4.5 ( 1.5) 6 A
a
I = =
Next, consider analyzing the circuit in Figure 6 using node equations. Figure 8 shows the
circuit after selecting a reference node and numbering the other nodes. Let v
1
, v
2
and v
3
denote
the node voltages at nodes 1, 2 and 3, respectively.
The voltage of the voltage source can be expressed in terms of the node voltages as


1 1
6 0 6 v v = = V

58
The voltmeter measures the voltage at node 3 to be 9 V. That is . Apply KCL at node 3
to get
3
9 V v =

3 2 3 2
2
9 9
0 0
2 2 2 2
v v v v
v

+ = + = =18 V

Apply KCL at node 2 to get

2 3 2 1
18 6 18 9
6 A
8 2 8 2
a
v v v v
I

= + = + =

This value of current source current agrees with the value calculated using mesh equations, as it
must.


Figure 8 The circuit from Figure 6 after labeling the nodes. (The encircled numbers are node
numbers.)


Example 3:
Consider the circuit shown in Figure 9. Find the value of the current source current, I
a
.



Figure 9 The circuit considered in Example 3.

Solution: Figure 10 shows the circuit from Figure 9 after replacing the ammeter by an equivalent
short circuit and labeling the current measured by the ammeter. This circuit can be analyzed
using mesh equations or using node equations. We will do both.
59
First, consider analyzing the circuit in Figure 10 using mesh equations. Figure 11 shows
the circuit after labeling the mesh currents, i
1
and i
2
.



Figure 10 The circuit from Figure 9 after replacing the ammeter by a short circuit.



Figure 11 The circuit from Figure 10 after labeling the meshes.


The mesh current i
2
flows through the short circuit that replaced the ammeter. The reference
direction of i
2
does not agree with the reference direction of the current measured by the
ammeter, so

2
0.75 A i =

The mesh current i
1
flows through the current source. The reference direction of i
2
does not agree
with the reference direction of the current source current, so


1 a
i = I
Apply KVL to the right mesh to get

( ) ( ) ( ) ( )
2 2 1
2 6 2 0 2 6 0.75 2 0.75 0 2 A
a a
i i i I I + + = + + = =

60
Next, consider analyzing the circuit in Figure 10 using node equations. Figure 12 shows
the circuit after selecting a reference node and numbering the other nodes. Let v
1
and v
2
denote
the node voltages at node 1 and node 2, respectively.
The current measured by the ammeter is equal to the current in the 6 resistor. The
voltage across the 6 resistor can be determined using Ohms Law. The voltage across the 6
resistor can also be expressed in terms of the node voltages. Doing both gives

( )
2 2
6 0.75 0 4.5 V v v = =

The voltage of the voltage source can be expressed in terms of the node voltages as

( )
1 2 1 1
2 2 4.5 2.5 V v v v v = = =

Apply KCL to the supernode to get


1
2.5
0.75 0.75 2 A
2 2
a a
v
I I

+ = + = =
a
I

As expected, this value of current source current agrees with the value calculated using mesh
equations.


Figure 12 The circuit from Figure 10 after labeling the nodes. (The encircled numbers are node
numbers.)
61
Example 4:
Consider the circuit shown in Figure 13. Find the value of the current source current, I
a
.



Figure 13 The circuit considered in Example 4.


Solution: Figure 14 shows the circuit from Figure 13 after replacing the voltmeter by an
equivalent open circuit and labeling the voltage measured by the voltmeter. This circuit can be
analyzed using mesh equations or using node equations. We will do both.
First, consider analyzing the circuit in Figure 14 using mesh equations. Figure 15 shows
the circuit after labeling the mesh currents, i
1
and i
2
.


Figure 14 The circuit from Figure 13 after replacing the voltmeter by a open circuit.


Figure 15 The circuit from Figure 14 after labeling the meshes.
62
The mesh current i
2
is equal to the current in the 4 resistor. The voltmeter measured to voltage
across that 4 resistor to be 9 V. Using Ohms Law, the mesh current is given by


2
9
2.25 A
4
i = =

The mesh current i
1
flows through the current source. The reference direction of i
2
agrees with
the reference direction of the current source current, so


1 a
I i =
Apply KVL to the right mesh to get

( ) ( ) ( )
2 1 2
6 9 20 16 0 15 20 2.25 16 2.25 0 6 A
a a
i i i I I + + = + = =

Next, consider analyzing the circuit in Figure 14 using node equations. Figure 16 shows
the circuit after selecting a reference node and numbering the other nodes. Let v
1
, v
2
and v
3

denote the node voltages at nodes 1, 2 and 3, respectively.
Apply KCL at node 3 to get

2 3 3
4 2
v v v
=
0
(3)
The voltage measured by the voltmeter is expressed in terms of the node voltages as

(4)
2 3
9 V v v =

Substituting this expression into Equation 3 gives

3
3
9
45 V
4 20
v
v = =

Substituting this expression into Equation 4 gives

2
54 V v =

The voltage of the voltage source can be expressed in terms of the node voltages as


1 2 1 1
6 6 54 60 V v v v v = = =

Apply KCL to the supernode to get


2 3 1
60 9
6 A
16 4 16 4
a
v v v
I

= + = + =

As expected, this value of current source current agrees with the value calculated using mesh
equations.
63


Figure 16 The circuit from Figure 14 after labeling the nodes. (The encircled numbers are node
numbers.
64
9. Mesh and Node Equations: More Resistive Circuits

Introduction

The circuits in this set of problems consist of independent sources, resistors and a meter. In
particular, these circuits do not contain dependent sources. These circuits can be analyzed using
mesh equation or using node equations.

Node equations are discussed in Sections 4.3 and 4.4 of Introduction to Electric Circuits by R.C.
Dorf and J.A Svoboda. Mesh equations are discussed in Section 4.6.


Worked Examples
Example 1:
Consider the circuit shown in Figure 1. Find the value of the resistance, R.




Figure 1 The circuit considered in Example 1.


Solution: Figure 2 shows the circuit from Figure 1 after replacing the ammeter by an equivalent
short circuit and labeling the current measured by the ammeter. This circuit can be analyzed
using mesh equations or using node equations. To decide which will be easier, we first count the
nodes and meshes. This circuit has 5 nodes. Selecting a reference node and then applying KCL at
the other four nodes will produce a set of four node equations. The circuit has three meshes.
Applying KVL to these three meshes will produce a set of three mesh equations. Hence,
analyzing this circuit using mesh equations instead of node equations will produce a smaller set
of equations. Further, notice two of the three mesh currents can be determined directly from the
current source currents. This makes the mesh equations easier to solve. We will analyze this
circuit by writing and solving mesh equations.
65
Figure 3 shows the circuit after numbering the meshes. Let i
1
, i
2
and i
3
denote the mesh
currents in meshes 1, 2 and 3, respectively.



Figure 2 The circuit from Figure 1 after replacing the ammeter by a short circuit.



Figure 3 The circuit from Figure 2 after labeling the meshes.



The mesh current i
1
is equal to the current in the 1 A current source so

1
1 A i =

The mesh current i
2
is equal to the current in the 3 A current source so


2
3 A i =
66

The mesh current i
3
is equal to the current in the short circuit that replaced the ammeter so

3
0.5 A i =

Apply KVL to mesh 3 to get

( ) ( ) ( )
3 1 3 3 2
2 12 i i i R i i + + = 0

Substituting the values of the mesh currents gives

( ) ( ) ( ) 2 0.5 1 12 0.5 0.5 3 0 2 R R + + = =


Example 2:
Consider the circuit shown in Figure 4. Find the value of the resistance, R.




Figure 4 The circuit considered in Example 2.


Solution: Figure 5 shows the circuit from Figure 4 after replacing the voltmeter by an equivalent
open circuit and labeling the voltage measured by the voltmeter. This circuit can be analyzed
using mesh equations or using node equations. To decide which will be easier, we first count the
nodes and meshes. This circuit has 4 nodes. Selecting a reference node and then applying KCL at
the other three nodes will produce a set of three node equations. The circuit has three meshes.
Applying KVL to these three meshes will produce a set of three mesh equations. Analyzing this
circuit using mesh equations requires the same number of equations as are required to analyze
the circuit using node equations. Notice that one of the three mesh currents can be determined
directly from the current source current but two of the three node voltages can be determined
directly from the voltage source voltages. This makes the node equations easier to solve. We will
analyze this circuit by writing and solving node equations.
67
Figure 6 shows the circuit after selecting a reference node and numbering the other
nodes. Let v
1
, v
2
and v
3
denote the node voltages at nodes 1, 2 and 3, respectively.



Figure 5 The circuit from Figure 4 after replacing the voltmeter by an open circuit.



Figure 6 The circuit from Figure 5 after labeling the nodes.


The voltage of the 16 V voltage source can be expressed in terms of the node voltages as

1 1
16 0 16 V v v = =

The voltage of the 18 V voltage source can be expressed in terms of the node voltages as

1 2 2 2
18 18 16 2 V v v v v = = =

The voltmeter measures the node voltage at node 3 so

3
16 V v =
Apply KCL at node 3 to get
68
1 3
2
2
v v v
3
R

+ =

Substituting the values of the node voltages gives

16 16 16
2 8
2
R
R

+ = =


Example 3:
Consider the circuit shown in Figure 7. Find the value of the resistance, R.



Figure 7 The circuit considered in Example 3.


Solution: Figure 8 shows the circuit from Figure 7 after replacing the voltmeter by an equivalent
open circuit and labeling the voltage measured by the voltmeter. This circuit can be analyzed
using mesh equations or using node equations. To decide which will be easier, we first count the
nodes and meshes. This circuit has 4 nodes. Selecting a reference node and then applying KCL at
the other three nodes will produce a set of three node equations. The circuit has two meshes.
Applying KVL to these two meshes will produce a set of two mesh equations. Hence, analyzing
this circuit using mesh equations instead of node equations will produce a smaller set of
equations.
One of the two mesh currents can be determined directly from the current source current
and one of the three node voltages can be determined directly from the voltage source voltage.
Another of the node voltages is measured by the voltmeter. Its hard to tell if it will be easier to
analyze this circuit using mesh equations or using node equations. We will analyze this circuit
using mesh equations.
Figure 9 shows the circuit after numbering the meshes. Let i
1
and i
2
denote the mesh
currents in meshes 1 and 2, respectively.

69


Figure 8 The circuit from Figure 7 after replacing the voltmeter by an open circuit.



Figure 9 The circuit from Figure 8 after labeling the meshes.


The mesh current i
2
is equal to the current in the 1 A current source so

2
1 A i =
Apply KVL to mesh 1 to get

(1) ( ) ( )
1 1 2 1 1
3 16 0 3 1 16 R i i i R i i + = + = 0

Apply KVL to mesh 2 to get

( ) ( ) ( )
2 1 2 1 1
2 ( 6.4) 3 0 2 1 ( 6.4) 3 1 0 3.8 A i i i i i = = =

Substituting this value of i
1
into Equation 1 gives

( ) ( ) 3.8 3 3.8 1 16 0 2 R R + = =
70
Example 4:
Consider the circuit shown in Figure 10. Find the value of the resistance, R.



Figure 10 The circuit considered in Example 4.

Solution: Figure 11 shows the circuit from Figure 10 after replacing the voltmeter by an
equivalent open circuit and labeling the voltage measured by the voltmeter. This circuit can be
analyzed using mesh equations or using node equations. To decide which will be easier, we first
count the nodes and meshes. This circuit has four nodes. Selecting a reference node and then
applying KCL at the other three nodes will produce a set of three node equations. The circuit has
two meshes. Applying KVL to these three meshes will produce a set of two mesh equations.
However, notice that two of the three node voltages can be determined directly from the voltage
source voltages and the third node voltage is measured by the voltmeter. It will be easier to
analyze this circuit using node equations than using mesh equations. We will analyze this circuit
by writing and solving node equations.
Figure 12 shows the circuit after selecting a reference node and numbering the other
nodes. Let v
1
, v
2
and v
3
denote the node voltages at nodes 1, 2 and 3, respectively.



Figure 11 The circuit from Figure 10 after replacing the voltmeter by an open circuit.

71


Figure 12 The circuit from Figure 11 after labeling the nodes.


The voltage of the 12 V voltage source can be expressed in terms of the node voltages as

1 1
12 0 12 V v v = =

The voltage of the 6 V voltage source can be expressed in terms of the node voltages as

2 2
6 0 6 v v = = V

The voltmeter measures the node voltage at node 3 so

3
6 V v =
Apply KCL at node 3 to get

1 3 2 3
6 3
v v v v v
R

+ =
3


Substituting the values of the node voltages gives

12 6 6 6 6
3
6 3
R
R

+ = =
72
Example 5:
Consider the circuit shown in Figure 13. Find the value of the resistance, R.



Figure 13 The circuit considered in Example 2.


Solution: See Example 4.3-1 of Introduction to Electric Circuits by R.C. Dorf and J.A Svoboda.




Example 6:
Consider the circuit shown in Figure 14. Find the value of the voltage measured by the voltmeter.



Figure 14 The circuit considered in Example 6.


Solution: See Example 4.4-1 of Introduction to Electric Circuits by R.C. Dorf and J.A Svoboda
73
10. Mesh and Node Equations: Circuits Containing Dependent
Sources

Introduction

The circuits in this set of problems are small circuits that contain a single dependent source.
These circuits can be analyzed using mesh equation or using node equations. When doing so, it
is useful to express the controlling current or voltage of the dependent source as a function of the
mesh currents or node voltages.

Node equations are discussed in Sections 4.3, 4.4 and 4.5 of Introduction to Electric Circuits by
R.C. Dorf and J.A Svoboda. Section 4.5 considers circuits that contain dependent sources. Mesh
equations are discussed in Sections 4.6 and 4.7. Section 4.7 considers circuits that contain
dependent sources.


Worked Examples
Example 1:
Consider the circuit shown in Figure 1. Find the value of the gain, A, of the CCCS.




Figure 1 The circuit considered in Example 1.



Solution: Figure 2 shows the circuit from Figure 1 after replacing the ammeter by an equivalent
short circuit and labeling the current measured by the ammeter. This circuit can be analyzed
using mesh equations. Figure 3 shows the circuit after numbering the meshes. Let i
1
and i
2
denote
the mesh currents in meshes 1 and 2, respectively.

74



Figure 2 The circuit from Figure 1 after replacing the ammeter by a short circuit.



Figure 3 The circuit from Figure 2 after labeling the meshes.

This circuit contains a dependent source. When analyzing such a circuit using mesh equations, it
is important to express the controlling current or voltage of the dependent source in terms of the
mesh currents. In this circuit, the controlling current of the dependent source is the current i
a
.
The controlling current of the dependent source is related to mesh current i
1
by

1 a
i i =

The mesh current i
2
is the current measured by the ammeter so


2
2 A i =

Apply KVL to the supermesh to get


1 2
16 14 12 0 i i + + =

Substituting for the mesh currents gives

( ) ( ) 16 14 2 12 0 1 A
a a
i i + + = =

The current in the dependent source is related to the mesh currents by

75

2 1 a
Ai i i =
Substituting the values of the currents gives

( ) 1 2 1 3 A/ A A = = A


Example 2:
Consider the circuit shown in Figure 4. Find the value of the gain, A, of the CCVS.




Figure 4 The circuit considered in Example 2.


Solution: Figure 5 shows the circuit from Figure 4 after replacing the voltmeter by an equivalent
open circuit and labeling the voltage measured by the voltmeter. This circuit can be analyzed
using node equations. Figure 6 shows the circuit after selecting a reference node and numbering
the other nodes. Let v
1
, v
2
and v
3
denote the node voltages at nodes 1, 2 and 3, respectively.




Figure 5 The circuit from Figure 4 after replacing the voltmeter by an open circuit.
76


Figure 6 The circuit from Figure 5 after labeling the nodes.


The voltage of the 12 V voltage source can be expressed in terms of the node voltages as

1 1
12 0 12 V v v = =

The voltmeter measures the node voltage at node 3 so

3
3.27 V v =

This circuit contains a dependent source. When analyzing such a circuit using node equations, it
is important to express the controlling current or voltage of the dependent source in terms of the
node voltages. In this circuit, the controlling current of the dependent source is the current i
a
. The
controlling current of the dependent source is equal to the current directed from right to left in
the 8 resistor. Hence i
a
is related to node voltages by


2 1
3.27 12
1.09 A
8 8
a
v v
i

= = =

The dependent source voltage is related to the node voltages by


2 2
0
a
Ai v v = =

The value of the current in an open circuit is always zero so applying KCL at node 3 gives

2 3
0
7
v v
=

Substituting for the node voltages gives

( )
3.27
0 1.09 3.27 0 3 V/A
7
a
Ai
A A

= = =
77
Example 3:
Consider the circuit shown in Figure 7. Find the value of the gain, A, of the VCCS.



Figure 7 The circuit considered in Example 3.

Solution: Figure 8 shows the circuit from Figure 7 after replacing the ammeter by an equivalent
short circuit and labeling the current measured by the ammeter. Also, the 6 and 16 series
resistors have been replaced by the equivalent 22 resistor. This circuit can be analyzed using
mesh equations. Figure 9 shows the circuit after numbering the meshes. Let i
1
and i
2
denote the
mesh currents in meshes 1 and 2, respectively.



Figure 8 The circuit from Figure 7 after replacing the ammeter by an short circuit.



Figure 9 The circuit from Figure 8 after labeling the meshes.
78

The mesh current i
1
is equal to the current in the independent current source so

1
0.2 A i =

The mesh current i
2
is the current measured by the ammeter so

2
1.11 A i =

This circuit contains a dependent source. When analyzing such a circuit using mesh equations, it
is important to express the controlling current or voltage of the dependent source in terms of the
mesh currents. In this circuit, the controlling voltage of the dependent source is the voltage v
a
.
Applying KVL to the supermesh gives

( )
1
22 0 0 22 0.2 4.4 V
a a
i v v + = = =

The current in the dependent source is related to the mesh currents by


1 2 a
A v i i =

Substituting the values of the currents and voltage gives

( ) ( ) 4.4 0.2 1.11 0.3 A/V A A = =

Example 4:
Consider the circuit shown in Figure 10. Find the value of the gain, A, of the VCVS.




Figure 10 The circuit considered in Example 4.


79
Solution: Figure 11 shows the circuit from Figure 10 after replacing the voltmeter by an
equivalent open circuit and labeling the voltage measured by the voltmeter. Also, the 3 and 8
series resistors have been replaced by the equivalent 11 resistor. This circuit can be
analyzed using node equations. Figure 12 shows the circuit after selecting a reference node and
numbering the other nodes. Let v
1
and v
2
denote the node voltages at nodes 1 and 2, respectively.



Figure 11 The circuit from Figure 10 after replacing the voltmeter by an open circuit.



Figure 12 The circuit from Figure 11 after labeling the nodes.


This circuit contains a dependent source. When analyzing such a circuit using node equations, it
is important to express the controlling current or voltage of the dependent source in terms of the
node voltages. In this circuit, the controlling voltage of the dependent source is the voltage v
a
.
This voltage is related to node voltages by


1 1
0
a
v v = = v
2

The dependent source voltage is related to the node voltages by

(1)
2
0
a
A v v v = =

The voltmeter measures the node voltage at node 2 so

2
47.14 V v =
80
Apply KCL at node 1 to get

1 2
5 0
11
v v
+ =

Substituting the value of v
2
gives

1
1
47.14
5 0 47.14 55 7.86 V
11
v
v

+ = = =

Substituting voltage values into Equation 1 gives

( ) 7.86 47.14 6 V/V A A = =
81
11. Mesh and Node Equations: More Circuits Containing
Dependent Sources
Introduction

The circuits in this set of problems each contain a single dependent source. These circuits can be
analyzed using mesh equation or using node equations. When doing so, it is useful to express
the controlling current or voltage of the dependent source as a function of the mesh currents or
node voltages.

Node equations are discussed in Sections 4.3, 4.4 and 4.5 of Introduction to Electric Circuits by
R.C. Dorf and J.A Svoboda. Section 4.5 considers circuits that contain dependent sources. Mesh
equations are discussed in Sections 4.6 and 4.7. Section 4.7 considers circuits that contain
dependent sources.


Worked Examples
Example 1:
Consider the circuit shown in Figure 1. Find the value of the gain, A, of the VCVS.




Figure 1 The circuit considered in Example 1.


Solution: Figure 2 shows the circuit from Figure 1 after replacing the voltmeter by an equivalent
open circuit and labeling the voltage measured by the voltmeter. We will analyze this circuit by
writing and solving node equations. Figure 3 shows the circuit after selecting a reference node
and numbering the other nodes. Let v
1
, v
2
and v
3
denote the node voltages at nodes 1, 2 and 3,
respectively.
82


Figure 2 The circuit from Figure 1 after replacing the voltmeter by an open circuit.



Figure 3 The circuit from Figure 2 after labeling the nodes.

The voltage across the dependent source is represented in three ways. It is A*v
a
with the + of
reference direction at the top, 5.14 V with the + at the bottom and v
3
0 = v
3
with the + at the
top. Consequently
3
5.14 V
a
v A v = =

The voltage of the 12 V voltage source can be expressed in terms of the node voltages as

1 1
12 0 12 V v v = =

The controlling voltage of the dependent source, v
a
, is the voltage across an open circuit. This
voltage can be expressed in terms of the node voltages at the nodes of the open circuit. Hence


2 2
0
a
v v v = =
Apply KCL to node 2 to get


2 3 1 2 2 2
2
12 ( 5.14)
1.71 V
12 3 12 3
v v v v v v
v

= = =
Finally,
2
5.14
3 V/V
1.71
a a
a
A v A v
A
v v

= = = =


83
Example 2:
Consider the circuit shown in Figure 4. Find the value of the gain, A, of the VCCS.



Figure 4 The circuit considered in Example 2.

Solution: Figure 5 shows the circuit from Figure 4 after replacing the ammeter by an equivalent
short circuit and labeling the current measured by the ammeter. We will analyze this circuit by
writing and solving node equations. Figure 6 shows the circuit after selecting a reference node
and numbering the other nodes. Let v
1
, v
2
and v
3
denote the node voltages at nodes 1, 2 and 3,
respectively.



Figure 5 The circuit from Figure 4 after replacing the ammeter by a short circuit.


Figure 6 The circuit from Figure 5 after labeling the nodes.
84
The voltage of the 36 V voltage source can be expressed in terms of the node voltages as

1 1
36 0 36 V v v = =

The controlling voltage of the dependent source, v
a
, is the voltage across an open circuit. This
voltage can be expressed in terms of the node voltages at the nodes of the open circuit. Hence

2 2
0
a
v v v = =

Node 3 is connected to the reference node by the short circuit that replaced the ammeter. The
voltages across a short circuit is 0 V. Consequently

3 3
0 0 V 0 V v v = =
Apply KCL at node 2 to get

2 3 1 2 2 2
2
36 0
10.3 V
20 8 20 8
v v v v v v
v

= = =

Apply KCL at node 3 to get

2 3
( 39.9) 41.2 V
8
a a
v v
A v A v

= + =

Finally,
2
41.2
4 A/V
10.3
a a
a
A v A v
A
v v
= = = =
85
Example 3:
Consider the circuit shown in Figure 7. Find the value of the gain, A, of the CCCS.



Figure 7 The circuit considered in Example 3.


Solution: Figure 8 shows the circuit from Figure 7 after replacing the voltmeter by an equivalent
open circuit and labeling the voltage measured by the voltmeter. We will analyze this circuit
using mesh equations. Figure 9 shows the circuit after numbering the meshes. Let i
1
and i
2
denote
the mesh currents in meshes 1 and 2, respectively.



Figure 8 The circuit from Figure 7 after replacing the voltmeter by an open circuit.



Figure 9 The circuit from Figure 8 after labeling the meshes.
86
The controlling current of the dependent source, i
a
, is the current in a short circuit. This short
circuit is common to meshes 1 and 2. The short circuit current can be expressed in terms of the
mesh currents as
1 2 a
i i i =

The dependent source is in only one mesh, mesh 2. The reference direction of the dependent
source current does not agree with the reference direction of i
2
. Consequently

2 a
Ai i =
Apply KVL to mesh 1 to get


1 1
3
32 24 0 A
4
i i = =

Apply KVL to mesh 2 to get


2 2
15
32 (30) 0 A
16
i i = =

Finally,
2
1 2
15
16
5 A/A
3 15
4 16
a
a
Ai i
A
i i i

= = = =



87
Example 4:
Consider the circuit shown in Figure 10. Find the value of the gain, A, of the CCVS.



Figure 10 The circuit considered in Example 4.


Solution: Figure 11 shows the circuit from Figure 10 after replacing the voltmeter by an
equivalent open circuit and labeling the voltage measured by the voltmeter. We will analyze this
circuit using mesh equations. Figure 11 shows the circuit after numbering the meshes. Let i
1
and
i
2
denote the mesh currents in meshes 1 and 2, respectively.



Figure 11 The circuit from Figure 10 after replacing the voltmeter by an open circuit.



Figure 12 The circuit from Figure 11 after labeling the meshes.
88

The voltage across the dependent source is represented in two ways. It is A*v
a
with the + of
reference direction at the bottom and -7.2 V with the + at the top. Consequently

( ) 7.2 7.2 V
a
A v = =

The controlling current of the dependent source, i
a
, is the current in a short circuit. This short
circuit is common to meshes 1 and 2. The short circuit current can be expressed in terms of the
mesh currents as
1 2 a
i i i =

Apply KVL to mesh 1 to get


1 1
10 36 0 3.6 A i i = =

Apply KVL to mesh 2 to get


2 2
4 ( 7.2) 0 1.8 A i i + = =

Finally,
1 2
7.2
4 V/A
3.6 1.8
a a
a
Ai Ai
A
i i i
= = = =

89
12. Source Transformations

Introduction

The circuits in this set of problems consist of independent sources, resistors and a meter. In
particular, these circuits do not contain dependent sources. Each of these circuits has a series-
parallel structure that makes it possible to simplify the circuit by repeatedly

Performing source transformations.
Replacing series or parallel resistors by an equivalent resistor.
Replacing series voltage sources by an equivalent voltage source.
Replacing parallel current sources by an equivalent source source.

Each simplification is done in such a way that the voltage or current measured by the meter is not
disturbed. Generally, that requires beginning the simplification at the opposite end of the circuit
from the meter and then working toward the meter.

Eventually, the circuit is small enough to be easily solved using Ohms and Kirchhoffs Laws.

Source transformations are discussed in Section 5.3 of Introduction to Electric Circuits by R.C.
Dorf and J.A Svoboda. Series resistors are discussed in Section 3.4. Parallel resistors are
discussed in Section 3.5. Series voltage sources and parallel current sources are described in
Section 3.6.


Worked Examples
Example 1:
Consider the circuit shown in Figure 1. Find the value of the voltage measured by the voltmeter.




Figure 1 The circuit considered in Example 1.


90
Solution: The voltmeter measures the voltage across the current source. (The color-coded probes
of the voltmeter indicate the reference direction of the voltage measured by the voltmeter.)
Figure 2 shows the circuit after the replacing the voltmeter by the equivalent open circuit and
adding a label to show the voltage measured by the meter.

Figures 3 through 17 illustrate the use of source transformations and equivalent resistances to
simplify the circuit.





Figure 2 The circuit from Figure 1 after the replacing the voltmeter by an open circuit.




Figure 3 Separating the circuit from Figure 2 into two parts.



Figure 4 The circuit from Figure 3 after doing a source transformation.

91


Figure 5 The circuit from Figure 4 after changing the order of parallel elements.



Figure 6 Separating the circuit from Figure 5 into two parts.



Figure 7 The circuit from Figure 6 after replacing parallel resistors with an equivalent resistor.



Figure 8 Separating the circuit from Figure 7 into two parts.


92


Figure 9 The circuit from Figure 8 after doing a source transformation.


Figure 10 The circuit from Figure 9 after changing the order of series elements.



Figure 11 Separating the circuit from Figure 10 into two parts.



Figure 12 The circuit from Figure 11 after replacing series voltage sources with an equivalent
voltage source

93


Figure 13 Separating the circuit from Figure 12 into two parts.



Figure 14 The circuit from Figure 13 after doing a source transformation.




Figure 15 The circuit from Figure 14 after changing the order of parallel elements.




Figure 16 Separating the circuit from Figure 15 into two parts.


94


Figure 17 The circuit from Figure 16 after replacing parallel resistors with an equivalent resistor.


Figure 18 The reduced circuit.

Figure 18 shows the simplified circuit after labeling the current i, of the resistor. Applying KCL
at the top node of the circuit gives

8 2
2 A
3 3
i i + = =

The voltage measured by the meter, v
m
, is also the voltage across the resistor. Ohms law gives

m
2
3 2
3
v
| |
= =
|
\ .
V

95
Example 2:
Consider the circuit shown in Figure 19. Find the value of the resistance, R.



Figure 19 The circuit considered in Example 2.


Solution: The voltmeter measures the voltage across one of the current sources. Figure 20 shows
the circuit after the replacing the voltmeter by the equivalent open circuit and adding a label to
show the voltage measured by the meter.

Figures 21 through 24 illustrate the use of source transformations and equivalent resistances to
simplify the circuit.



Figure 20 The circuit from Figure 19 after the replacing the voltmeter by an open circuit.




Figure 21 Separating the circuit from Figure 20 into two parts.

96


Figure 22 The circuit from Figure 21 after doing a source transformation.



Figure 23 Separating the circuit from Figure 22 into two parts.




Figure 24 The circuit from Figure 23 after replacing series resistors with an equivalent resistor



Figure 25 The circuit from Figure 24 after numbering the nodes.

97

Figure 25 shows the simplified circuit after numbering the nodes. Let v
1
, and v
2
denote the node
voltages at nodes 1 and 2 respectively. Then the node voltages are

v
1
= -45 V and v
2
= 5 V

Applying KCL at node 2 gives

5 40 5
4 1 5
15
R
R

= + = =
98
Example 3:
Consider the circuit shown in Figure 26. Find the value of the voltage measured by the voltmeter.



Figure 26 The circuit considered in Example 3.


Solution: The voltmeter measures the voltage across the current source. (The color-coded probes
of the voltmeter indicate the reference direction of the voltage measured by the voltmeter.)
Figure 27 shows the circuit after the replacing the voltmeter by the equivalent open circuit and
adding a label to show the voltage measured by the meter.

Figures 28 through 35 illustrate the use of source transformations and equivalent resistances to
simplify the circuit.



Figure 27 The circuit from Figure 26 after the replacing the voltmeter by an open circuit.


99


Figure 28 Separating the circuit from Figure 27 into two parts.



Figure 29 The circuit from Figure 28 after doing a source transformation.



Figure 30 Separating the circuit from Figure 29 into two parts.

100


Figure 31 The circuit from Figure 32 after replacing parallel resistors with an equivalent resistor.



Figure 32 Separating the circuit from Figure 31 into two parts.




Figure 33 The circuit from Figure 32 after doing a source transformation.

101


Figure 34 Separating the circuit from Figure 33 into two parts.



Figure 35 The circuit from Figure 34 after replacing series resistors with an equivalent resistor.




Figure 36 The reduced circuit.


Figure 36 shows the simplified circuit after labeling the current and voltage of the 9 resistor.
Applying KVL to the loop circuit gives

m m
9 12 0 3 v v = = V
102
Example 4:
Consider the circuit shown in Figure 37. Find the value of the voltage measured by the voltmeter.



Figure 37 The circuit considered in Example 4.


Solution: The voltmeter measures the voltage across 4 resistor. (The color-coded probes of
the voltmeter indicate the reference direction of the voltage measured by the voltmeter.) Figure
38 shows the circuit after the replacing the voltmeter by the equivalent open circuit and adding a
label to show the voltage measured by the meter.

Figures 40 through 47 illustrate the use of source transformations and equivalent resistances to
simplify the circuit.


Figure 39 The circuit from Figure 38 after the replacing the voltmeter by an open circuit.



Figure 40 Separating the circuit from Figure 39 into two parts.

103


Figure 41 The circuit from Figure 40 after doing a source transformation.



Figure 42 The circuit from Figure 41 after changing the order of series elements.



Figure 43 Separating the circuit from Figure 42 into two parts.



Figure 44 The circuit from Figure 43 after replacing series voltage sources with an equivalent
voltage source.
104


Figure 45 Separating the circuit from Figure 44 into two parts.



Figure 46 The circuit from Figure 45 after replacing series resistors with an equivalent resistor.




Figure 47 The reduced circuit.


Figure 47 shows the simplified circuit. Voltage division gives

4
90 9 V
36 4
m
v
| |
= =
|
+
\ .

105
13. Superposition

Introduction

Each circuit in this problem set has two inputs and one output. The inputs to these circuits are the
voltages of independent voltage sources and the currents of independent current sources. The
output of each circuit is the voltage or current measure by the meter.

These circuits can be analyzed using superposition. Superposition says that the output caused by
two inputs working together can be determined by finding the outputs due to each input working
separately and then adding those separate outputs.

Superposition is discussed in Section 5.4 of Introduction to Electric Circuits by R.C. Dorf and
J.A Svoboda.


Worked Examples
Example 1:
Consider the circuit shown in Figure 1. Find the value of the current source current, I
a
.



Figure 1 The circuit considered in Example 1.


Solution: The inputs to this circuit are the voltage of the independent voltage source and the
current of the independent current source. The response of this circuit is the current measured by
the ammeter. The two inputs work together to produce a response that has a value of 3 A. This
circuit can be analyzed using superposition to separate the part of the response caused by the
voltage source from the part of the response caused by the current source.
Figure 2 shows the circuit from Figure 1 after replacing the ammeter by an equivalent
short circuit and labeling the current measured by the ammeter.
Figure 3a shows the circuit used to determine, i
1
, the part of the response caused by the
voltage source. The current source current is set to zero to determine the part of the response
106
caused by the voltage source. Consequently, the current source is replaced by an open circuit, i.e.
a zero current source, in Figure 3a.
Figure 3b shows the circuit used to determine, i
2
, the part of the response caused by the
current source. The voltage source voltage is set to zero to determine the part of the response
caused by the current source. Consequently, the voltage source is replaced by a short circuit, i.e.
a zero voltage source, in Figure 3b.



Figure 2 The circuit from Figure 1 after replacing the ammeter by a short circuit.



(a)


(b)

Figure 3 The circuit from Figure 2 after using superposition. (a) The response to the voltage
source acting alone. (b) The response to the current source acting alone.



Apply KVL to the circuit in Figure 3a to get

1 1 1
2 2 4 0 1 A i i i + = =

Use current division in Figure 3b to get

2
2 1
2 2 2
a a
i I = =
+
I

107
The response to both sources working together is equal to the sum of the responses to the
individual sources working separately. Consequently,

1 2
1
3 1
2
a a
i i I I = + = + = 4 A

Example 2:
Consider the circuit shown in Figure 4. Find the value of the current source current, I
a
.



Figure 4 The circuit considered in Example 2.


Solution: The inputs to this circuit are the voltage of the independent voltage source and the
current of the independent current source. The response of this circuit is the voltage measured by
the voltmeter. The two inputs work together to produce a response that has a value of 9 V. This
circuit can be analyzed using superposition to separate the part of the response caused by the
voltage source from the part of the response caused by the current source.
Figure 5 shows the circuit from Figure 4 after replacing the voltmeter by an equivalent
open circuit and labeling the voltage measured by the voltmeter.
Figure 6a shows the circuit used to determine, v
1
, the part of the response caused by the
voltage source. The current source current is set to zero to determine the part of the response
caused by the voltage source. Consequently, the current source is replaced by an open circuit, i.e.
a zero current source, in Figure 6a.
Figure 6b shows the circuit used to determine, v
2
, the part of the response caused by the
current source. The voltage source voltage is set to zero to determine the part of the response
caused by the current source. Consequently, the voltage source is replaced by a short circuit, i.e.
a zero voltage source, in Figure 6b.


108


Figure 5 The circuit from Figure 4 after replacing the voltmeter by an open circuit.



(a)


(b)

Figure 6 The circuit from Figure 5 after using superposition. (a) The response to the voltage
source acting alone. (b) The response to the current source acting alone.


Apply voltage division to the circuit in Figure 3a to get

1
2
6 1 V
8 2 2
v = =
+ +


Use current division in Figure 3b to get

( )
8 2
8 2 2 3
b a
i I = =
+ +
a
I

Ohms Law gives
2
2 4
2 2
3 3
b a
v i I
| |
= = =
|
\ .
a
I

The response to both sources working together is equal to the sum of the responses to the
individual sources working separately. Consequently,

1 2
4
9 1
3
a a
v v I I = + = + = 6 A
109
Example 3:
Consider the circuit shown in Figure 7. Find the value of the current source current, I
a
.



Figure 7 The circuit considered in Example 3.


Solution: The inputs to this circuit are the voltage of the independent voltage source and the
current of the independent current source. The response of this circuit is the current measured by
the ammeter. The two inputs work together to produce a response that has a value of 0.75 A.
This circuit can be analyzed using superposition to separate the part of the response caused by
the voltage source from the part of the response caused by the current source.
Figure 8 shows the circuit from Figure 7 after replacing the ammeter by an equivalent
short circuit and labeling the current measured by the ammeter.
Figure 9a shows the circuit used to determine, i
1
, the part of the response caused by the
voltage source. The current source current is set to zero to determine the part of the response
caused by the voltage source. Consequently, the current source is replaced by an open circuit, i.e.
a zero current source, in Figure 9a.
Figure 9b shows the circuit used to determine, i
2
, the part of the response caused by the
current source. The voltage source voltage is set to zero to determine the part of the response
caused by the current source. Consequently, the voltage source is replaced by a short circuit, i.e.
a zero voltage source, in Figure 9b.





Figure 8 The circuit from Figure 7 after replacing the ammeter by a short circuit.

110


(a)


(b)


Figure 9 The circuit from Figure 8 after using superposition. (a) The response to the voltage
source acting alone. (b) The response to the current source acting alone.

.
Apply KVL to the circuit in Figure 9a to get

1 1 1
1
2 6 2 0 A
4
i i i = =

Use current division in Figure 3b to get

2
2 1
2 6 4
a a
i I = =
+
I

The response to both sources working together is equal to the sum of the responses to the
individual sources working separately. Consequently,

1 2
3 1 1
2 A
4 4 4
a a
i i I I = + = + =
111
Example 4:
Consider the circuit shown in Figure 10. Find the value of the current source current, I
a
.



Figure 10 The circuit considered in Example 4.


Solution: The inputs to this circuit are the voltage of the independent voltage source and the
current of the independent current source. The response of this circuit is the voltage measured by
the voltmeter. The two inputs work together to produce a response that has a value of 9 V. This
circuit can be analyzed using superposition to separate the part of the response caused by the
voltage source from the part of the response caused by the current source.
Figure 11 shows the circuit from Figure 10 after replacing the voltmeter by an equivalent
open circuit and labeling the voltage measured by the voltmeter.
Figure 12a shows the circuit used to determine, v
1
, the part of the response caused by the
voltage source. The current source current is set to zero to determine the part of the response
caused by the voltage source. Consequently, the current source is replaced by an open circuit, i.e.
a zero current source, in Figure 12a.
Figure 12b shows the circuit used to determine, v
2
, the part of the response caused by the
current source. The voltage source voltage is set to zero to determine the part of the response
caused by the current source. Consequently, the voltage source is replaced by a short circuit, i.e.
a zero voltage source, in Figure 12b.




Figure 11 The circuit from Figure 10 after replacing the voltmeter by a open circuit.

112


(a)


(b)

Figure 12 The circuit from Figure 11 after using superposition. (a) The response to the voltage
source acting alone. (b) The response to the current source acting alone.


Apply voltage division to the circuit in Figure 12a to get

( )
1
4
6 0.6 V
16 20 4
v = =
+ +


Use current division in Figure 3b to get

( )
16 2
16 20 4 5
b a
i I = =
+ +
a
I

The Ohms Law gives

2
2 8
4 4 1.6
5 5
b a a
v i I I
| |
= = = =
|
\ .
a
I

The response to both sources working together is equal to the sum of the responses to the
individual sources working separately. Consequently,

1 2
9 0.6 1.6 6 A
a a
v v I I = + = + =
113
114
14. Thevenin Equivalent Circuits
Introduction

In each of these problems, we are shown a circuit and its Thevenin or Norton equivalent circuit.
The Thevenin and Norton equivalent circuits are described using three parameters: V
oc
, the open
circuit voltage of the circuit, I
sc
, the short circuit of the circuit and R
th
, the Thevenin resistance of
the circuit. Each problem, asks us to determine the value of asked to determine the value of V
oc
,
I
sc
or R
th
.

Thevenin equivalent circuits are discussed in Section 5.5 of Introduction to Electric Circuits by
R.C. Dorf and J.A Svoboda. Norton equivalent circuits are discussed in Section 5.6.

Worked Examples
Example 1:
The circuit shown in Figure 1b is the Thevenin equivalent circuit of the circuit shown in Figure
1a. Find the value of the open circuit voltage, V
oc
and Thevenin resistance, R
th
.




Figure 1 The circuit considered in Example 1.


Solution: The circuit from Figure 1a can be reduced to its Thevenin equivalent circuit in four
steps shown in Figure 2a, b, c and d.

A source transformation transforms the series voltage source and 20 resistor in Figure
1a into the parallel current source and 20 resistor in Figure 2a. The current source current is
calculated from the voltage source voltage and resistance as
20 V
1 A
20
=

. After the source


transformation, the 20 resistor is parallel to the 80 resistor. Replacing these parallel resistors
with the equivalent 16 resistor produces the circuit shown in Figure 2b.
A second source transformation transforms the parallel current source and 16 resistor
in Figure 2b into the series voltage source and 16 resistor in Figure 2c. The voltage source
115
voltage is calculated from the current source current and resistance as . After
the source transformation, the two16 resistors are in series. Replacing these series resistors
with the equivalent 32 resistor produces the circuit shown in Figure 2d.
( )( ) 1 A 16 16 V =
Comparing Figure 2d to Figure 1b shows that the Thevenin resistance is R
th
= 32 and
the open circuit voltage, V
oc
= -16 V.



(a)



(b)

(c)

(d)

Figure 2 The circuit from Figure 1a can be reduced to its Thevenin equivalent circuit in four
steps shown here as (a), (b), (c), and (d).


116
Example 2:
The circuit shown in Figure 3b is the Thevenin equivalent circuit of the circuit shown in Figure
1a. Find the value of the open circuit voltage, V
oc
and Thevenin resistance, R
th
.




Figure 3 The circuit considered in Example 2.


Solution: The circuit from Figure 3a can be reduced to its Thevenin equivalent circuit in five
steps shown in Figure 4a, b, c, d and e.

A source transformation transforms the parallel current source and 3 resistor in Figure
3a into the series voltage source and 3 resistor in Figure 4a. The voltage source voltage is
calculated from the current source current and resistance as ( ) . After the source
transformation, the 3 and 6 resistors are in series. Also, the 6V and 3 V voltage sources are
in series. Replacing the series resistors with the equivalent 9 resistor and the series voltage
sources with the equivalent 8 V source produces the circuit shown in Figure 4b.
( ) 2 A 3 6 V =

A second source transformation transforms the series 8 V voltage source and 9 resistor
in Figure 4b into the parallel current source and 9 resistor in Figure 4c. The current source
current is calculated from the voltage source voltage and resistance as
8 V
0.89 A
9
=

. After the
source transformation, the 9 resistor is parallel to the 6 resistor. Replacing these parallel
resistors with the equivalent 3.6 resistor produces the circuit shown in Figure 4d.

A third source transformation transforms the parallel 0.89 A current source and 3.6
resistor in Figure 4d into the series voltage source and 3.6 resistor in Figure 4e. The voltage
source voltage is calculated from the current source current and resistance as
. ( )( ) 0.89 A 3.6 3.2 V =
Comparing Figure 4e to Figure 3b shows that Thevenin resistance is R
th
= 3.6 and that
the open circuit voltage, V
oc
= -3.2 V.
117


(a)



(b)

(c)

(d)


(e)

Figure 4 The circuit from Figure 3a can be reduced to its Thevenin equivalent circuit in five
steps shown here as (a), (b), (c), (d) and (e).
118
Example 3:
The circuit shown in Figure 5b is the Thevenin equivalent circuit of the circuit shown in Figure
5a. Find the value of the open circuit voltage, V
oc
and Thevenin resistance, R
th
.




Figure 5 The circuit considered in Example 3.


Solution: The circuit from Figure 5a can be reduced to its Thevenin equivalent circuit in four
steps shown in Figure 6a, b, c and d.
A source transformation transforms the series 10 V voltage source and 5 resistor in
Figure 5a into the parallel current source and 5 resistor in Figure 6a. The current source
current is calculated from the voltage source voltage and resistance as
10
. After the
source transformation, the 5 resistor is parallel to the 20 resistor. Also, the 2 A current
source is parallel to the 1 A current source. Replacing these parallel resistors with the equivalent
4 resistor and replacing the parallel current sources with the equivalent 1 A current source
produces the circuit shown in Figure 6b.
V
2 A
5
=

A second source transformation transforms the parallel 1 A current source and 4


resistor in Figure 6b into the series voltage source and 4 resistor in Figure 6c. The voltage
source voltage is calculated from the current source current and resistance as ( ) .
After the source transformation, the two 4 resistors are in series. Replacing the series resistors
with the equivalent 8 produces the circuit shown in Figure 6d.
( ) 1 A 4 4 V =
Comparing Figure 6d to Figure 5b shows that Thevenin resistance is R
th
= 8 and that
the open circuit voltage, V
oc
= 4 V.
119

(a)


(b)

(c)

(d)

Figure 6 The circuit from Figure 5a can be reduced to its Thevenin equivalent circuit in four
steps shown here as (a), (b), (c), and (d).



Example 4:
The circuit shown in Figure 7b is the Thevenin equivalent circuit of the circuit shown in Figure
7a. Find the value of the open circuit voltage, V
oc
and Thevenin resistance, R
th
. Also, determine
the value of the short circuit current, I
sc
.




Figure 7 The circuit considered in Example 4.
120
Solution: To determine the value of the open circuit voltage, V
oc
, we connect an open circuit
across the terminals of the circuit and then calculate the value of the voltage across that open
circuit. Figure 8 shows the circuit from Figure 7a after adding the open circuit and labeling the
open circuit voltage. Also, the meshes have been identified and labeled in anticipation of writing
mesh equations. Let i
1
and i
2
denote the mesh currents in meshes 1 and 2, respectively.
In Figure 8, mesh current i
2
is equal to the current in the open circuit. Consequently,
. The controlling current of the CCVS is expressed in terms of the mesh currents as
2
0 A i =


1 2 1
0
a
i i i i i = = =
1

Apply KVL to mesh 1 to get


( ) ( ) ( ) ( )
1 1 2 1 2 1 1 1
1
3 2 6 10 0 3 2 0 6 0 10 0
10
1.43 A
7
i i i i i i i i
i
+ = + =
= =

Apply KVL to mesh 2 to get


( ) ( ) ( )
2 1 2 1
5 6 0 6 6 1.43 8.58 V
oc oc
i V i i V i + = = = =

Next, to determine the value of the short circuit current, I
sc
, we connect a short circuit
across the terminals of the circuit and then calculate the value of the current in that short circuit.
Figure 9 shows the circuit from Figure 7a after adding the short circuit and labeling the short
circuit current. Also, the meshes have been identified and labeled in anticipation of writing mesh
equations. Let i
1
and i
2
denote the mesh currents in meshes 1 and 2, respectively.
In Figure 9, mesh current i
2
is equal to the current in the short circuit. Consequently,
2 sc
i I = . The controlling current of the CCVS is expressed in terms of the mesh currents as

1 2 1 a s
i i i i I = =
c




Figure 8 Calculating the open circuit voltage, V
oc
, using mesh equations.

Apply KVL to mesh 1 to get

121
(1)
( ) ( )
1 1 2 1 2 1 2
3 2 6 10 0 7 4 1 i i i i i i i + = = 0

Apply KVL to mesh 2 to get

( )
2 1 2 1 2 1
11
5 6 0 6 11 0
6
i i i i i i = + = =
2
i

Substituting into equation 1 gives


2 2 2
11
7 4 10 1.13 A 1.13 A
6
sc
i i i I
| |
= = =
|
\ .






Figure 9 Calculating the short circuit current, I
sc
, using mesh equations.



Figure 10 Calculating the Thevenin resistance,
T
th
T
v
R
i
= , using mesh equations.


To determine the value of the Thevenin resistance, R
th
, first replace the 10 V voltage
source by a 0 V voltage source, i.e. a short circuit. Next, connect a current source across the
terminals of the circuit and then label the voltage across that current source as shown in Figure
10. The Thevenin resistance will be calculated from the current and voltage of the current source
as
122
T
th
T
v
R
i
=

In Figure 10, the meshes have been identified and labeled in anticipation of writing mesh
equations. Let i
1
and i
2
denote the mesh currents in meshes 1 and 2, respectively.
In Figure 10, mesh current i
2
is equal to the negative of the current source current.
Consequently, i . The controlling current of the CCVS is expressed in terms of the mesh
currents as
2 T
= i
1 2 1 a T
i i i i i = = +

Apply KVL to mesh 1 to get


( ) ( )
1 1 2 1 2 1 2 1
4
3 2 6 0 7 4 0
7
i i i i i i i i + = = =
2
i (2)

Apply KVL to mesh 2 to get

( )
2 1 2 1 2
5 6 0 6 11
T T
i v i i i i v + = + =

Substituting for i
1
using equation 2 gives

2 2 2
4
6 11 7.57
7
T T
i i v i
| |
+ = =
|
\ .
v
Finally,
2
7.57
T T T
th
T T
v v v
R
i i i

= = = =



As a check, notice that

( )( ) 7.57 1.13 8.55
th sc oc
R I V = =
123
Example 5:
The circuit shown in Figure 11b is the Thevenin equivalent circuit of the circuit shown in Figure
11a. Find the value of the open circuit voltage, V
oc
and Thevenin resistance, R
th
. Also, determine
the value of the short circuit current, I
sc
.



Figure 11 The circuit considered in Example 5.

Solution: To determine the value of the open circuit voltage, V
oc
, we connect an open circuit
across the terminals of the circuit and then calculate the value of the voltage across that open
circuit. Figure 12 shows the circuit from Figure 11a after adding the open circuit and labeling the
open circuit voltage. Also, the nodes have been identified and labeled in anticipation of writing
node equations. Let v
1
, v
2
and v
3
denote the node voltages at nodes 1, 2 and 3, respectively.
In Figure 12, node voltage v
1
is equal to the negative of the voltage source voltage.
Consequently, v . The controlling voltage of the VCCS, v
1
24 V =
2 a
v v =
a
, is equal to the node voltage
at node 2, i.e. . The voltage at node 3 is equal to the open circuit voltage, i.e. .
3 oc
v V =
Apply KCL at node 2 to get


1 2 2 3
1 3 2
2 3 48
3 6
oc a
v v v v
v v v V v

= + = + = 3



Figure 12 Calculating the open circuit voltage, V
oc
, using node equations.
124
Apply KCL at node 3 to get

2 3
2 2 3
4
0 9 0 9
6 3
a o
v v
v v v v

+ = = =
c
V

Combining these equations gives

( ) 3 48 9 72 V
oc a oc oc
V v V V + = = =

Next, to determine the value of the short circuit current, I
sc
, we connect a short circuit
across the terminals of the circuit and then calculate the value of the current in that short circuit.
Figure 13 shows the circuit from Figure 7a after adding the short circuit and labeling the short
circuit current. Also, the nodes have been identified and labeled in anticipation of writing node
equations. Let v
1
, v
2
and v
3
denote the node voltages at nodes 1, 2 and 3, respectively.
In Figure 13, node voltage v
1
is equal to the negative of the voltage source voltage.
Consequently, v . The voltage at node 3 is equal to the voltage across a short, .
The controlling voltage of the VCCS, v
1
24 V =
3
0 v =
2
v =
a
, is equal to the node voltage at node 2, i.e. . The
voltage at node 3 is equal to the voltage across a short, i.e. .
a
v
3
0 v =
Apply KCL at node 2 to get


1 2 2 3
1 3 2
2 3 48 3 1
3 6
a a
v v v v
v v v v v

= + = = = 6 V

Apply KCL at node 3 to get

( )
2 3
2
4 9 9
16 24 A
6 3 6 6
sc a sc sc
v v
v I v I I

+ = = = =





Figure 13 Calculating the short circuit current, I
sc
, using mesh equations.
125


Figure 14 Calculating the Thevenin resistance,
T
th
T
v
R
i
= , using mesh equations.

To determine the value of the Thevenin resistance, R
th
, first replace the 24 V voltage
source by a 0 V voltage source, i.e. a short circuit. Next, connect a current source circuit across
the terminals of the circuit and then label the voltage across that current source as shown in
Figure 14. The Thevenin resistance will be calculated from the current and voltage of the current
source as
T
th
T
v
R
i
=
Also, the nodes have been identified and labeled in anticipation of writing node equations. Let
v
1
, v
2
and v
3
denote the node voltages at nodes 1, 2 and 3, respectively.
In Figure 14, node voltage v
1
is equal to the across a short circuit, i.e. . The
controlling voltage of the VCCS, v
1
0 v =
a
=
a
, is equal to the node voltage at node 2, i.e. v . The
voltage at node 3 is equal to the voltage across the current source, i.e. v v .
2
v
3 T
=
Apply KCL at node 2 to get


1 2 2 3
1 3 2
2 3
3 6
T a
v v v v
v v v v v

= + = = 3

Apply KCL at node 3 to get

2 3
2 2 3
4
0 9 6 0
6 3
9 6 0
3 6 0 2
T T
a T T
T T T T
v v
v i v v i
v v i
v v i v i

+ + = + =
+ =
+ = = 6
T

Finally,
3
T
th
T
v
R
i
= =
As a check, notice that
126
15. Op Amp Circuits

Introduction

The circuits in this problem set are popular op amp circuits. Indeed, these circuits are used so
often that they have been given names: the inverting and noninverting amplifiers, the summing
amplifier and the current-to-voltage converter.

To solve these problems, we identify the particular circuit, recall the equation that describes that
circuit and then substitute appropriate values into that equation.

These popular op amp circuits are discussed in Section 6.6 of Introduction to Electric Circuits by
R.C. Dorf and J.A Svoboda. In particular, Figure 6.6-1 provides a catalog of popular op amp
circuits together with equations that describe those circuits.


Worked Examples
Example 1:
Consider the circuit shown in Figure 1. Find the value of voltage measured by the voltmeter.



Figure 2 The circuit considered in Example 2.


Solution: Recognize that this circuit is an inverting amplifier. The inverting amplifier is
described by Figure 6.6-1a in Introduction to Electric Circuits. Comparing Figure 1 to Figure
6.6-1a gives


1 f
0.8 V, 2.9 k and 14.5 k
in
v R R = = =

127
Also, the voltmeter in Figure 1 measures the voltage labeled as v
out
in Figure 6.6-1a. Using this
correspondence, the equation given in Figure 6.6-1a indicates that value of the voltage measured
by the voltmeter is
( )
14500
0.8 4 V
2900
out
v
| |
= =
|
\ .




Example 2:
Consider the circuit shown in Figure 2. Find the value of the resistance, R.



Figure 2 The circuit considered in Example 2.


Solution: Recognize that this circuit is a noninverting amplifier. The noninverting amplifier is
described by Figure 6.6-1b in Introduction to Electric Circuits. Comparing Figure 2 to Figure
6.6-1b gives

1 f
0.7 V, 4.88 V, and 15 k
in out
v v R R R = = = =

Using this correspondence, the equation given in Figure 6.6-1b to describe the inverting
amplifier indicates that
( )
15000
4.88 1 0.7
R
| |
= +
|
\ .

Solving for R gives
2.5 k R =


128
Example 3:
Consider the circuit shown in Figure 3. Find the value of voltage measured by the voltmeter.
Determine the value of the power supplied by the op amp.



Figure 3 The circuit considered in Example 3.


Solution: Recognize that this circuit is a current-to-voltage converter. The current-to-voltage
converter is described by Figure 6.6-1g in Introduction to Electric Circuits. Comparing Figure 3
to Figure 6.6-1g gives

f
5 mA and 2.5 k
in
i R = =

Also, the voltmeter in Figure 3 measures the voltage labeled as v
out
in Figure 6.6-1g. Using this
correspondence, the equation given in Figure 6.6-1g indicates that the value of the voltage
measured by the voltmeter is

( ) ( )
3 3
2.5 10 5 10 12.5 V
out
v

= =

To calculate the power supplied by the op amp, we first calculate the output current of the op
amp, labeled as i
o
in Figure 3. To determine the value i
o
, apply KCL at the output node of the op
amp to get

0
2500 20000
out out
o
v v
i

+ =

Using v
out
= 12.5 V gives

( )
3
0 12.5
12.5
5.63 10 5.63 mA
2500 20000
o o
i i

+ = = =

The power delivered by the op amp is given by

( ) ( )
3 3
12.5 5.63 10 70.38 10 70.38 mW
out o
v i

= = =
129


Example 4:
Consider the circuit shown in Figure 4. Find the value of voltage measured by the voltmeter.



Figure 4 The circuit considered in Example 4.


Solution: The inputs to the circuit in Figure 4 are the voltages of the voltage sources and the
output is the voltage measured by the voltmeter. The principle of superposition indicates that the
response to two inputs working together can be calculated by determining the response to each
input working separately and then adding the separate responses. Figure 5 shows the circuits
used to calculate the responses to the inputs working separately.
Recognize that the circuit in Figure 5a is an inverting amplifier. The inverting amplifier is
described by Figure 6.6-1a in Introduction to Electric Circuits. Comparing Figure 5a to Figure
6.6-1a gives


1 1 f
0.8 V, , 2.3 k and 6.9 k
in out
v v v R R = = = =

Using this correspondence, the equation given in Figure 6.6-1a indicates that the response to the
0.8 V source acting alone is

( )
1
6900
0.8 2.4 V
2300
v
| |
= =
|
\ .


Recognize that the circuit in Figure 5b is a noninverting amplifier. The noninverting
amplifier is described by Figure 6.6-1b in Introduction to Electric Circuits. Comparing Figure 5b
to Figure 6.6-1b gives


2 1 f
1.0 V, , 2.3 k and 6.9 k
in out
v v v R R = = = =

130
Using this correspondence, the equation given in Figure 6.6-1b indicates that the response to the
1.0 V source acting alone is
( )
2
6900
1 1.0 4 V
2300
v
| |
= + =
|
\ .


Finally, the principle of superposition indicates that the value of the voltage measured by the
voltmeter is

1 2
2.4 4 6.4 V v v + = + =






(a)


(b)

Figure 5 Applying superposition to the circuit shown in Figure 4. (b) The circuit used to
calculate the response to the 0.8 V source acting alone and (b) the circuit used to calculate the
response to the 1.0 V source acting alone.
131
Example 5:
Consider the circuit shown in Figure 6. Find the value of voltage measured by the voltmeter.



Figure 6 The circuit considered in Example 5.


Solution: Recognize that this circuit is a summing amplifier. The summing amplifier is
described by Figure 6.6-1d in Introduction to Electric Circuits. Comparing Figure 6 to Figure
6.6-1d gives


1 2 1 2 f
0.75 V, 0.5 V, 20 k , 20 k , and 120 k v v R R R = = = = =

Also, the voltmeter in Figure 6 measures the voltage labeled as v
out
in Figure 6.6-1d. Using this
correspondence, the equation given in Figure 6.6-1d indicates that the value of the voltage
measured by the voltmeter is

( ) ( )
120000 120000
0.75 0.5 0.5 V
20000 12000
out
v
( | | | |
= + =
| | (
\ . \ .


132
16. Node Equations for Op Amp Circuits

Introduction

The circuits in this problem each contain one or more ideal op amps. To analyze these circuits,
we write and solve a set of node equations.

Ideal op amps are described in Section 6.4 of Introduction to Electric Circuits by R.C. Dorf and
J.A Svoboda. Section 6.5 shows how to analyze op amp circuits using node equations.


Worked Examples
Example 1:
Consider the circuit shown in Figure 1. Find the value of the voltage measured by the voltmeter.



Figure 1 The circuit considered in Example 1.


Solution: Figure 2 shows the circuit from Figure 1 after replacing the voltmeter by an equivalent
open circuit and labeling the voltage measured by the voltmeter. We will analyze this circuit by
writing and solving node equations. The nodes of the circuit are numbered in Figure 2. Let v
1
,
v
2
, v
3
and v
4
denote the node voltages at nodes 1, 2, 3 and 4 respectively.
The inputs to this circuit are the voltages of the voltage sources. These inputs are related
to the node voltages at the nodes of the voltage sources by

1 1
0 0.75 0.75 V v v = =
and
2 2
0 0.5 0.5 V v v = =

133
The output of this circuit is the voltage measured by the voltmeter. The output voltage is related
to the node voltages by

4 4
0
m
v v v = =




Figure 2 The circuit from Figure 1 after replacing the voltmeter by an open circuit and labeling
the nodes. (Encircled numbers are node numbers)



The noninverting input of the op amp is connected to the reference node. The node voltage at the
inverting input of an ideal op amp is equal to the node voltage at the noninverting input. The
inverting input of the op amp is connected to node 3. Consequently,

3
0 V v =
Apply KCL to node 3 to get


1 3 2 3 3 4
1 2 3 3
0 6 10 16
20000 12000 120000
v v v v v v
v v v v

+ = + + =
4
v
v


Using v
m
= v
4
, and v
3
= 0 shows that the output voltage is related to the input voltages by

( )
1 2
6 10
m
v v = +

Using v
1
= -0.75 V, and v
2
= 0.5 V gives the value of the voltage measured by the voltmeter to be

( ) ( ) ( )
6 0.75 10 0.5 0.5 V
m
v = + =

134
Example 2:
Consider the circuit shown in Figure 3. Find the value of the voltage measured by the voltmeter.



Figure 3 The circuit considered in Example 2.

Solution: Figure 4 shows the circuit from Figure 3 after replacing the voltmeter by an equivalent
open circuit and labeling the voltage measured by the voltmeter. We will analyze this circuit by
writing and solving node equations. The nodes of the circuit are numbered in Figure 4. Let v
1
,
v
2
, v
3
and v
4
denote the node voltages at nodes 1, 2, 3 and 4 respectively.
The inputs to this circuit are the voltages of the voltage sources. These inputs are related
to the node voltages at the nodes of the voltage sources by

1 1
0 0.8 0.8 v v = = V
and
2 2
0 1 1 V v v = =




Figure 4 The circuit from Figure 3 after replacing the voltmeter by an open circuit and labeling
the nodes. (Encircled numbers are node numbers)

135
The output of this circuit is the voltage measured by the voltmeter. The output voltage is related
to the node voltages by
4 4
0
m
v v v = =

The noninverting input of the op amp is connected to node 2. The node voltage at the inverting
input of an ideal op amp is equal to the node voltage at the noniverting input. The inverting input
of the op amp is connected to node 3. Consequently,

3 2
1 V v v = =
Apply KCL to node 3 to get

( )
1 3 3 4
1 3 3
0 3
2300 6900
v v v v
v v v v

= + =
4
v


Using v
m
= v
4
, and v
3
= v
2
shows that the output voltage is related to the input voltages by

1 2
3 4
m
v v = +

Using v
1
= -0.8 V, and v
2
= 1 V gives the value of the voltage measured by the voltmeter to be

( ) ( ) 3 0.8 4 1 6.4 V
m
v = + =



Example 3:
Consider the circuit shown in Figure 5. Find the value of the voltage measured by the voltmeter.



Figure 5 The circuit considered in Example 3.


136
Solution: Figure 6 shows the circuit from Figure 5 after replacing the voltmeter by an equivalent
open circuit and labeling the voltage measured by the voltmeter. We will analyze this circuit by
writing and solving node equations. The nodes of the circuit are numbered in Figure 6. Let v
1
,
v
2
, v
3
and v
4
denote the node voltages at nodes 1, 2, 3 and 4 respectively.
The output of this circuit is the voltage measured by the voltmeter. The output voltage is
related to the node voltages by


4 4
0
m
v v v = =

The inputs to this circuit are the voltage of the voltage source and the currents of the
current sources. The voltage of the voltage source is related to the node voltages at the nodes of
the voltage sources by

3 3
0 2.75 2.75 V v v = =




Figure 6 The circuit from Figure 5 after replacing the voltmeter by an open circuit and labeling
the nodes. (Encircled numbers are node numbers)


Apply KCL to node 2 to get

6 3 2
3 2
0 60 10 1.8 V
30000
v v
v v

= + =
Using v
3
= -2.75 V gives
2
4.55 V v =

The noninverting input of the op amp is connected to node 2. The node voltage at the inverting
input of an ideal op amp is equal to the node voltage at the noninverting input. The inverting
input of the op amp is connected to node 1. Consequently,

1 2
4.55 V v v = =
137

Apply KCL to node 1 to get


6 1 4
1 4
20 10 0 0.8 V
40000
v v
v v


= + =

Using v
m
= v
4
, and v
1
= -4.55 V gives the value of the voltage measured by the voltmeter to be

4.55 0.8 5.35 V
m
v = =

Example 4:
Consider the circuit shown in Figure 7. Find the value of the voltage measured by the voltmeter.



Figure 7 The circuit considered in Example 4.

Solution: Figure 8 shows the circuit from Figure 7 after replacing the voltmeter by an equivalent
open circuit and labeling the voltage measured by the voltmeter. We will analyze this circuit by
writing and solving node equations. Figure 8 shows the circuit after numbering the nodes. Let
v
1
, v
2
, v
3
and v
4
denote the node voltages at nodes 1, 2, 3 and 4 respectively.
The input to this circuit is the voltage of the voltage source. This input is related to the
node voltages at the nodes of the voltage source by

1 1
0 3.35 3.35 V v v = =

The output of this circuit is the voltage measured by the voltmeter. The output voltage is related
to the node voltages by

4 4
0
m
v v v = =

138


Figure 8 The circuit from Figure 7 after replacing the voltmeter by an open circuit and labeling
the nodes. (Encircled numbers are node numbers)


The noninverting input of the op amp is connected to the reference node. The node voltage at the
inverting input of an ideal op amp is equal to the node voltage at the noninverting input. The
inverting input of the op amp is connected to node 2. Consequently,

2
0 V v =

Apply KCL to node 2 to get


2 3 1 2
3 1 2
0 2
20000 40000
v v v v
v v v

= + = + =
1
3 2 v

Apply KCL to node 3 to get

2 3 3 3 4
4 2 3
5 10
40000 10000 8000
v v v v v
v v v

= + = + =
3
10 v
1
v


Combining these equations gives


4 3
2 4 v v = =

Using v
m
= v
4
, and v
1
= -3.35 V gives the value of the voltage measured by the voltmeter to be

( ) 4 3.35 13.4 V
m
v = =




139
Example 5:
Consider the circuit shown in Figure 9. Find the value of the voltage measured by the voltmeter.



Figure 9 The circuit considered in Example 5.


Solution: Figure 10 shows the circuit from Figure 9 after replacing the voltmeter by an
equivalent open circuit and labeling the voltage measured by the voltmeter. We will analyze this
circuit by writing and solving node equations. The nodes of the circuit are numbered in Figure
10. Let v
1
, v
2
, v
3
, v
4
, v
5
and v
6
denote the node voltages at nodes 1, 2, 3, 4, 5 and 6 respectively.
The inputs to this circuit are the voltages of the voltage sources. These inputs are related
to the node voltages at the nodes of the voltage sources by

1 1
0 0.15 0.15 V v v = =
and
2 2
0 3.5 3.5 V v v = =

The output of this circuit is the voltage measured by the voltmeter. The output voltage is related
to the node voltages by

3 6 m
v v v =
140


Figure 10 The circuit from Figure 9 after replacing the voltmeter by an open circuit and labeling
the nodes. (Encircled numbers are node numbers)


The noninverting input of the top op amp is connected to node 1. The node voltage at the
inverting input of an ideal op amp is equal to the node voltage at the noninverting input. The
inverting input of the op amp is connected to node 4. Consequently,

4 1
0.15 V v v = =

The noninverting input of the bottom op amp is connected to node 2. The node voltage at the
inverting input of an ideal op amp is equal to the node voltage at the noninverting input. The
inverting input of the op amp is connected to node 5. Consequently,

5 2
0.35 V v v = =

Apply KCL to node 4 to get


3 4 4 5
3 4
9 7
0
140000 40000 2 2
v v v v
v v

= + =
5
v

Apply KCL to node 4 to get


4 5 5 6
6 4
5 7
0
40000 100000 2 2
v v v v
v v

= + = +
5
v

Using v
m
= v
3
v
6
shows that the output voltage is related to the input voltages by

141
( )
4 5 1 2 1
9 7 5 7
7
2 2 2 2
m
v v v v v v
| | | |
= + =
| |
\ . \ .
2
v

Using v
1
= -0.15 V, and v
2
= 0.35 V gives the value of the voltage measured by the voltmeter to
be

( ) ( ) ( ) ( ) 7 0.15 0.35 7 0.5 3.5 V
m
v = = =



Example 6:
Consider the circuit shown in Figure 11. Find the value of the voltage measured by the voltmeter.


Figure 11 The circuit considered in Example 6.


Solution: Figure 12 shows the circuit from Figure 11 after replacing the voltmeter by an
equivalent open circuit and labeling the voltage measured by the voltmeter. We will analyze this
circuit by writing and solving node equations. The nodes of the circuit are numbered in Figure
12. Let v
1
, v
2
, v
3
and v
4
denote the node voltages at nodes 1, 2, 3 and 4 respectively.
The input to this circuit is the voltage of the voltage source. This input is related to the
node voltages at the nodes of the voltage sources by

1 1
0 1.0 1.0 V v v = =

The output of this circuit is the voltage measured by the voltmeter. The output voltage is related
to the node voltages by

4 4
0
m
v v v = =

142

Figure 12 The circuit from Figure 11 after replacing the voltmeter by an open circuit and
labeling the nodes. (Encircled numbers are node numbers)


The noninverting input of the op amp is connected to the reference node. The node voltage at the
inverting input of an ideal op amp is equal to the node voltage at the noninverting input. The
inverting input of the op amp is connected to node 3. Consequently,

3
0 V v =

Apply KCL to node 2 to get


2 3 1 2 2
1 2
3 6
6000 9000 18000
v v v v v
v v

= + =
3
v

Using v
1
= 1 V, and v
3
= 0 gives v
2
= 0.5 V. Next, apply KCL to node 3 to get

( )
2 3 3 4
2 3 3 4
0 3
18000 54000
v v v v
v v v v

= + =

Using v
m
= v
4
, v
3
= 0 and v
2
= 0.5 V gives the value of the voltage measured by the voltmeter to
be

2
3 1.5 V
m
v v = =
143
17. Capacitors and Inductors with Current and Voltage
Represented Graphically

Introduction

Each of the circuits in this problem set consists of a single capacitor or inductor and a single
independent source. When the independent source is a current source, the current source current
is equal to the current in the capacitor or inductor. Similarly, when the independent source is a
voltage source, the voltage source voltage is equal to the voltage across the capacitor or inductor.

These problems can be solved using the element equations for the capacitor and inductor. (An
element equation is the equation that describes the relationship between the element voltage
and element current.) The element equations of capacitors and inductors involve derivatives and
integrals. Since the voltages and currents in these problems are described graphically, its useful
to interpret derivatives as slopes and integrals as areas.

Capacitors and inductors are described in Sections 7.3 and 7.6 of Introduction to Electric
Circuits by R.C. Dorf and J.A Svoboda.


Worked Examples
Example 1:
Consider the circuit shown in Figure 1. Find the value of the capacitance C.



Figure 1 The circuit considered in Example 1.


Solution: The current and voltage of the capacitor are related by

( ) ( )
d
i t C v t
dt
= (1)
144

Since i(t) and v(t) are represented graphically, by plots rather than equations, it is useful to
interpret Equation 1 as

( ) ( ) the value of the slope of i t C v t =

Pick a time when both the value of i(t) and the slope v(t) are easily determined. For example, at
time t = 3 seconds, i(3) = 8 mA = 0.008 A and

( )
0 12 V
3 4
2 5 s
d
v
dt

= =



(The notation ( ) 3
d
v
dt
indicates that the derivative ( )
d
v t
dt
is evaluated at time t = 3 s.) Using
Equation 1 at time t = 3 s gives

( )
A
0.008 4 0.002 0.002 F 2 mF
V/s
C C = = = =

Example 2:
Consider the circuit shown in Figure 2. Find the value of the inductor voltage, v(t), at time t = 3
ms. (That is, find v(0.003).)



Figure 2 The circuit considered in Example 2.


Solution: The voltage and current of the inductor are related by

( ) ( )
d
v t L i t
dt
= (2)

Since i(t) and v(t) are represented graphically, by plots rather than equations, it is useful to
interpret Equation 2 as
145

( ) ( ) the value of the slope of v t L i t =

The value of the voltage at time t = 3 ms = 0.003 s is required. We need to determine the slope of
i(t) at time t = 0.003 s. That slope is

( )
( ) 0 0.012
A
0.003 6
0.002 0.004 s
d
i
dt

= =



(The notation (0.003
d
i
dt
) indicates that the derivative ( )
d
i t
dt
is evaluated at time t = 0.003 s.)

The inductance of the inductor in Figure 2 is L = 3 H. Using Equation 2 gives

( ) ( )
A
0.003 3 6 18 H 18 V
s
v = = =

Example 3:
Consider the circuit shown in Figure 3. Find the value of the inductance L.



Figure 3 The circuit considered in Example 3.


Solution: The current and voltage of the inductor are related by

( ) ( ) ( )
0
0
1
t
t
i t v d i t
L
=

+ (3)
or
( ) ( ) ( )
0
0
1
t
t
i t i t v d
L
=

(4)

Since i(t) and v(t) are represented graphically, by plots rather than equations, it is useful to
interpret Equation 4 using
146

i(t) - i(t
0
) = the difference between the values of voltage at times t and t
0

and
( )
0
t
t
v d

= the area under the plot of v(t) versus t, for times between t and t
0


Pick convenient values t and t
0
, for example, t
0
= 2 ms and t = 6 ms. Then

i(t) - i(t
0
) = 1 (-2) = 3 A
and
( )
0
0.006
0.002
30
t
t
v d d =

= (30) (0.006-0.002) = 0.12 V-s

Using Equation 4 gives

( )
1 V-s
3 0.12 0.040 0.040 H 40 mH
A
L
L
= = = =

Example 4:
Consider the circuit shown in Figure 4. Find the value of the inductor current, i(t), at time t = 3
ms. (That is, find i(0.003).)



Figure 4 The circuit considered in Example 4.


Solution: The current and voltage of the inductor are related by

( ) ( ) ( )
0
0
1
t
t
i t v d i t
L
=

+ (5)

Since i(t) and v(t) are represented graphically, by plots rather than equations, it is useful to
interpret Equation 5 using

147
( )
0
t
t
v d


30
= the area under the plot of v(t) versus t, for times between t and t
0


The values t and t
0
are specified to be t
0
= 0 ms and t = 3 ms. Then i(t
0
) = 2 A and

( )
0
0.002 0.003
0 0.002
0
t
t
v d d d = +

= 0 + (-30) (0.003-0.002) = -0.03 V-s

At time t
0
= 0 ms the inductor current is give to be i(t
0
) = 2 A . The inductance of the inductor in
Figure 4 is L = 20 mH = 0.02 H. Using Equation 5 gives

( ) ( )
1
0.003 0.03 2 0.5 A
0.02
i = + =


Example 5:
Consider the circuit shown in Figure 5. Find the value of the capacitance C.



Figure 5 The circuit considered in Example 5.


Solution: The current and voltage of the capacitor are related by

( ) ( )
d
i t C v t
dt
= (6)

Since i(t) and v(t) are represented graphically, by plots rather than equations, it is useful to
interpret Equation 6 as

( ) ( ) the value of the slope of i t C v t =

Pick a time when both the value of i(t) and the slope v(t) are easily determined. For example, at
time t = 2.5 seconds, i(2.5) = 50 mA = 0.050 A and
148

( )
( ) 3 2
V
2.5 1
2 3 s
d
v
dt

= =



(The notation (2.5
d
v
dt
) indicates that the derivative ( )
d
v t
dt
is evaluated at time t = 2.5 s.)
Using Equation 6 at time t = 2.5 s gives

( )
A
0.050 1 0.050 0.050 F 50 mF
V/s
C C = = = =
149
18. Simple First Order Circuits
Introduction

Each of the circuits in this problem set is a first order circuit because each circuit contains
either a single inductor or a single capacitor.

The input to each of these first order circuits is the voltage of an independent voltage source.
This input has a constant value before time t = 0 and a different constant value after time t = 0.
The input changes value abruptly at time t = 0. Inputs of this kind can be described analytically
using the step function, u(t), and thats what has been done in these problems.

First order circuits are discussed in Section 8.3 of Introduction to Electric Circuits by R.C. Dorf
and J.A Svoboda. The step function is described in Section 8.6.


Worked Examples
Example 1:
Figure 1 shows a first order circuit. The input to the circuit is the voltage of the voltage source,
v
s
(t). The output is the voltage across the capacitor, v
o
(t). Determine the output of this circuit
when the input is v t . ( ) ( ) 8 15 V
s
u t = +



Figure 1 The circuit considered in Example 1.


Solution: The value of the input is one constant, -8 V, before time t = 0 and a different constant,
7 V, after time t = 0. The response of the first order circuit to the change in the value of the input
will be
(1) ( )
o
for 0
a t
v t A B e t

= + >

where the values of the three constants A, B and a are to be determined.
The values of A and B are determined from the steady state responses of this circuit
before and after the input changes value. Figures 2a and b show the circuits used to calculate
those steady state responses. Figures 2a and b require some explanation.
150
Capacitors act like open circuits when the input is constant and the circuit is at steady
state. Consequently, the capacitor is replaced by an open circuit in Figure 2a and also in Figure
2b.





(a)


(b)

Figure 2 Circuits used to calculate the steady state response (a) before t = 0 and (b) after t = 0.


The value of the capacitor voltage at time t = 0, will be equal to the steady state capacitor
voltage before the input changes. At time t = 0 the output voltage is

( )
( ) 0
o
0
a
v A B e A

= + = + B
A


Consequently, the capacitor voltage is labeled as A + B in Figure 2a.
The value of the capacitor voltage at time t = , will be equal to the steady state capacitor
voltage after the input changes. At time t = the output voltage is

( )
( )
o
a
v A B e

= + =

Consequently, the capacitor voltage is labeled as A in Figure 2b.
Analysis of the circuit in Figure 2a gives

8 V A B + =

Analysis of the circuit in Figure 2b gives

A = 7 V
Therefore
B = -15 V

The value of the constant a in Equation 1 is determined from the time constant, , which
is in turn calculated from the values of the capacitance C and of the Thevenin resistance, R
t
, of
the part of the circuit that is connected to the capacitor.
151


1
t
R C
a
= =

Figure 3 shows the circuit used to calculate R
t
. It is seen from Figure 3 that

R
t
= 6
Therefore

( )( )
3
1 1
2.5
s 6 66.7 10
a

= =



(The time constant is .) Substituting the values of A, B and a into
Equation 1 gives
( )( )
3
6 66.7 10 0.4 s

= =

( )
o 2.5
8 V for 0
7 15 V for 0
t
t
v t
e t





Figure 3 The circuit used to calculate R
t
.


152
Example 2:
Figure 4 shows a first order circuit. The input to the circuit is the voltage of the voltage source,
v
s
(t). The output is the current the inductor, i
o
(t). Determine the output of this circuit when the
input is . ( ) ( ) 4 8 V
s
v t u t =



Figure 4 The circuit considered in Example 2.


Solution: The value of the input is one constant, 4 V, before time t = 0 and a different constant, -
4 V, after time t = 0. The response of the first order circuit to the change in the value of the input
will be

(2) ( )
o
for 0
a t
i t A B e t

= + >

where the values of the three constants A, B and a are to be determined.
The values of A and B are determined from the steady state responses of this circuit
before and after the input changes value. Figures 5a and b show the circuits used to calculate
those steady state responses. Figures 5a and b require some explanation.
Inductors act like short circuits when the input is constant and the circuit is at steady
state. Consequently, the inductor is replaced by an open circuit in Figure 5a and also in Figure
5b.
The value of the inductor current at time t = 0, will be equal to the steady state inductor
current before the input changes. At time t = 0 the output current is

( )
( ) 0
o
0
a
i A B e A

= + = + B

Consequently, the inductor current is labeled as A + B in Figure 5a.


153


(a)


(b)

Figure 5 Circuits used to calculate the steady state response (a) before t = 0 and (b) after t = 0.



A
The value of the inductor current at time t = , will be equal to the steady state inductor
current after the input changes. At time t = the output current is

( )
( )
o
a
i A B e

= + =

Consequently, the inductor current is labeled as A in Figure 5b.
Analysis of the circuit in Figure 5a gives

0.2 A A B + =

Analysis of the circuit in Figure 5b gives

A = -0.2 A
Therefore
B = 0.4 A

The value of the constant a in Equation 2 is determined from the time constant, , which
is in turn calculated from the values of the inductance L and of the Thevenin resistance, R
t
, of the
circuit connected to the inductor.


1
t
L
a R
= =

Figure 6 shows the circuit used to calculate R
t
. It is seen from Figure 6 that

R
t
= 20
Therefore

20 1
2
10 s
a = =

154
(The time constant is
10
0.5 s
20
= = .) Substituting the values of A, B and a into Equation 2 gives
( )
o 2
0.2 A for 0
0.2 0.4 A for 0
t
t
i t
e t


=

+






Figure 6 The circuit used to calculate R
t
.



Example 3:
Figure 7 shows a first order circuit. The input to the circuit is the voltage of the voltage source,
v
s
(t). The output is the voltage across the resistor v
o
(t). Setting the input of this circuit equal to
( ) ( ) 6 13 V
s
v t u t =

causes the output of this circuit to be
( )
o 2
6 V for 0
7 13 V for 0
t
t
v t
e t


=

+



Determine the value of the inductance, L.



Figure 7 The circuit considered in Example 3.

Solution: The inductance is determined by considering the time constant of the circuit. The time
constant is determined from exponent in the transient part of the output voltage.
155

1
2 H
2 4
t
L L
L
R
= = = =


Example 4:
Figure 8 shows a first order circuit. The input to the circuit is the voltage of the voltage source,
v
s
(t). The output is the voltage across the resistor, v
o
(t). Setting the input of this circuit equal to
( ) ( ) 8 15 V
s
v t u t = +

causes the output of this circuit to be

( )
o 2.5
0 for
15 V for 0
t
t
v t
e t

<
=

>

0


Determine the value of the capacitance, C.




Figure 8 The circuit considered in Example 4.


Solution: The capacitance is determined by considering the time constant of the circuit. The time
constant is determined from exponent in the transient part of the output voltage.

( )
( ) ( )
1 1
6 0.0667 F
2.5 6 2.5
t
R C C C = = = = =
156
19. More First Order Circuits

Introduction

Each of the circuits in this problem set is a first order circuit because each circuit contains
either a single inductor or a single capacitor.

The input to each of these first order circuits is the voltage of an independent voltage source.
This input has a constant value before time t = 0 and a different constant value after time t = 0.
The input changes value abruptly at time t = 0. Inputs of this kind can be described analytically
using the step function, u(t), and thats what has been done in these problems.

First order circuits are discussed in Section 8.3 of Introduction to Electric Circuits by R.C. Dorf
and J.A Svoboda. The step function is described in Section 8.6.


Worked Examples
Example 1:
Figure 1 shows a first order circuit. The input to the circuit is the voltage of the voltage source,
v
s
(t). The output is the voltage across the capacitor, v
o
(t). Determine the output of this circuit
when the input is v t . ( ) ( ) 7 14 V
s
u t =



Figure 1 The circuit considered in Example 1.

Solution: The value of the input is one constant, 7 V, before time t = 0 and a different constant,
-7 V, after time t = 0. The response of the first order circuit to the change in the value of the input
will be
(1) ( )
o
for 0
a t
v t A B e t

= + >

where the values of the three constants A, B and a are to be determined.
The values of A and B are determined from the steady state responses of this circuit
before and after the input changes value. Figures 2a and b show the circuits used to calculate
those steady state responses. Figures 2a and b require some explanation.
157
Capacitors act like open circuits when the input is constant and the circuit is at steady
state. Consequently, the capacitor is replaced by an open circuit in Figure 2a and also in Figure
2b.





(a)


(b)

Figure 2 Circuits used to calculate the steady state response (a) before t = 0 and (b) after t = 0.


The value of the capacitor voltage at time t = 0, will be equal to the steady state capacitor
voltage before the input changes. At time t = 0 the output voltage is

( )
( ) 0
o
0
a
v A B e A

= + = + B
A


Consequently, the capacitor voltage is labeled as A + B in Figure 2a.
The value of the capacitor voltage at time t = , will be equal to the steady state capacitor
voltage after the input changes. At time t = the output voltage is

( )
( )
o
a
v A B e

= + =

Consequently, the capacitor voltage is labeled as A in Figure 2b.
Apply the voltage division rule to the circuit in Figure 2a to get


5
7 4.38 V
3 5
A B + = =
+


Apply the voltage division rule to the circuit in Figure 2b to get

( )
5
7 4.38 V
3 5
A = =
+

Therefore
B = -8.76 V

158
The value of the constant a in Equation 1 is determined from the time constant, , which
is in turn calculated from the values of the capacitance C and of the Thevenin resistance, R
t
, of
the circuit connected to the capacitor.


1
t
R C
a
= =




Figure 3 The circuit used to calculate R
t
.


Figure 3 shows the circuit used to calculate R
t
. It is seen from Figure 3 that



( ) ( ) 5 3
1.875
5 3
t
R = =
+


Therefore

( )( )
3
1 1
1.16
s 1.875 460 10
a

= =



(The time constant is .) Substituting the values of A, B and a into
Equation 1 gives
( )( )
3
1.875 460 10 0.86 s

= =

( )
o 1.16
4.38 V for 0
4.38 8.76 V for 0
t
t
v t
e t


=

+





159
Example 2:
Figure 4 shows a first order circuit. The input to the circuit is the voltage of the voltage source,
v
s
(t). The output is the current the inductor, i
o
(t). Determine the output of this circuit when the
input is . ( ) ( ) 7 13 V
s
v t u t = +



Figure 4 The circuit considered in Example 2.

Solution: The value of the input is one constant, -7 V, before time t = 0 and a different constant,
6 V, after time t = 0. The response of the first order circuit to the change in the value of the input
will be
(2) ( )
o
for 0
a t
i t A B e t

= + >

where the values of the three constants A, B and a are to be determined.
The values of A and B are determined from the steady state responses of this circuit
before and after the input changes value. Figures 5a and b show the circuits used to calculate
those steady state responses. Figures 5a and b require some explanation.
Inductors act like short circuits when the input is constant and the circuit is at steady
state. Consequently, the inductor is replaced by a short circuit in Figure 5a and also in Figure 5b.





(a)


(b)

Figure 5 Circuits used to calculate the steady state response (a) before t = 0 and (b) after t = 0.


The value of the inductor current at time t = 0, will be equal to the steady state inductor
current before the input changes. At time t = 0 the output current is
160

( )
( ) 0
o
0
a
i A B e A

= + = + B

Consequently, the inductor current is labeled as A + B in Figure 5a.


A
The value of the inductor current at time t = , will be equal to the steady state inductor
current after the input changes. At time t = the output current is

( )
( )
o
a
i A B e

= + =

Consequently, the inductor current is labeled as A in Figure 5b.
Notice that the voltage across the 4 resistor in Figure 5a is zero. Consequently, the
current in the 4 resistor is also zero. The voltage across the 5 resistor is 7 V. The current in
the 5 resistor is equal to A + B. Using Ohms Law gives


7
1.4 A
5
A B

+ = =

Analysis of the circuit in Figure 5b gives


6
=1.2 A
5
A =
Therefore
B = -1.6 A

The value of the constant a in Equation 2 is determined from the time constant, , which
is in turn calculated from the values of the inductance L and of the Thevenin resistance, R
t
, of the
circuit connected to the inductor.


1
t
L
a R
= =

Figure 6 shows the circuit used to calculate R
t
. It is seen from Figure 6 that


( ) ( ) 5 4
2.22
5 4
t
R = =
+

Therefore

2.22 1
1.85
1.2 s
a = =

(The time constant is
1.2
0.54 s
2.22
= = .) Substituting the values of A, B and a into Equation 2
gives
161

( )
o 1.85
1.4 A for 0
1.2 2.6 A for 0
t
t
i t
e t






Figure 6 The circuit used to calculate R
t
162
20. Switched First Order Circuits
Introduction

Each of the circuits in this problem set is a first order circuit because each circuit contains
either a single inductor or a single capacitor.

The input to each of these first order circuits is the voltage of an independent voltage source or
current of an independent current source. In these problems, the input has a constant value.

Each of these circuits contains a switch. That switch that opens or closes at time t = 0, changing
the circuit.

First order circuits are discussed in Section 8.3 of Introduction to Electric Circuits by R.C. Dorf
and J.A Svoboda. Switches are described in Section 2.10.


Worked Examples
Example 1:
The circuit shown in Figure 1 is at steady state before the switch opens a time t = 0. The input to
the circuit is the voltage of the voltage source, V
s
. This voltage source is a dc voltage source, that
is, V
s
, is a constant. The output of this circuit is the voltage across the capacitor, v
o
(t). The output
voltage is given by

(1) ( )
o
0.4
4 16 V for 0
t
v t e t

= + >

Determine the values of the input voltage, V
s
, the capacitance, C, and the resistance, R.



Figure 1 The circuit considered in Example 1.

163
Solution: Before the switch opens, the circuit will be at steady state. Because the only input to
this circuit is the constant voltage of the voltage source, all of the element currents and voltages,
including the capacitor voltage, will have constant values. Closing the switch disturbs the circuit
by shorting out resistor R. Eventually the disturbance dies out and the circuit is again at steady
state. All the element currents and voltages will again have constant values, but probably
different constant values than they had before the switch closed.




Figure 2 The circuit from Figure 1, at steady state when t < 0.


Figure 2 shows the circuit before t = 0, when the switch is closed and the circuit is at
steady state. The closed switch is modeled as a short circuit. The combination of resistor and a
short circuit connected is equivalent to a short circuit. Consequently, a short circuit replaces the
switch and the resistor R. A capacitor in a steady-state dc circuit acts like an open circuit, so an
open circuit replaces the capacitor. The voltage across that open circuit is the capacitor voltage,
v
o
(t).

Because the circuit in Figure 2 is at steady state, the value of the capacitor voltage will be
constant. This constant is the value of the capacitor voltage just before the switch opens. In the
absence of unbounded currents, the voltage of a capacitor must be continuous. The value of the
capacitor voltage immediately after the switch opens is equal to the value immediately before the
switch opens. This value is called the initial condition of the capacitor and has been labeled as
v
o
(0) in Figure 2. There is no current in the horizontal resistor due to the open circuit.
Consequently, v
o
(0) is equal to the voltage across the vertical resistor, which is equal to the
voltage source voltage. Therefore
( )
o s
0 v V =

The value of v
o
(0) can also be obtained by setting t = 0 in Equation 1. Doing so gives

( )
0
o
0 4 16 20 V v e = + =
Consequently,
s
20 V V =


164


Figure 3 The circuit from Figure 1, at steady state when t > 0.



Next, consider the circuit after the switch opens. Eventually (certainly as t ) the
circuit will again be at steady state. Figure 3 shows the circuit at t = , when the switch is open
and the circuit is at steady state. The open switch is modeled as an open circuit. A capacitor in a
steady-state dc circuit acts like an open circuit, so an open circuit replaces the capacitor. The
voltage across that open circuit is the steady-state capacitor voltage, v
o
(). There is no current in
the horizontal resistor and v
o
() is equal to the voltage across the vertical resistor. Using voltage
division,

( ) ( )
o
10
20
10
v
R
=
+


The value of v
o
() can also be obtained by setting t = in Equation 1. Doing so gives

( )
o
4 16 4 V v e

= + =
Consequently,

( )
10
4 20 4 40 200 40
10
R R
R
= + = =
+

Finally, the exponential part of v
o
(t) is known to be of the form
t

e where
t
R C = and
R
t
is the Thevenin resistance of the part of the circuit connected to the capacitor. Figure 4 shows
the circuit that is used to determine R
t
. An open circuit has replaced the open switch. (Equation 1
describes the capacitor voltage for t > 0, that is, while the switch is open.) Independent sources
are set to zero when calculating R
t
, so the voltage source has been replaced by an short circuit.
From Figure 4

( )( )
t
40 10
10 18
40 10
R = + =
+

165
so
t
18 R C C = =
From Equation 1
0.4 2.5 s
t
t

= =
Consequently,
2.5 18 0.13889 138.89 mF C C = = =





Figure 4 The circuit that is used to determine R
t
.

166
Example 2:
Consider the circuit shown in Figure 5. The input to the circuit is the voltage of the voltage
source, 24 V. The output of this circuit, the voltage across the 6 resistor, is given by

(2) ( )
o
0.35
12 6 V when 0
t
v t e t

= >

Determine the value of the inductance, L and of the resistances R
1
and R
2
.




Figure 5 The circuit considered in Example 2.


Solution: Before the switch closes, the circuit will be at steady state. Because the only input to
this circuit is the constant voltage of the voltage source, all of the element currents and voltages,
including the inductor current, will have constant values. Closing the switch disturbs the circuit
by shorting out the resistor R
1
. Eventually the disturbance dies out and the circuit is again at
steady state. All the element currents and voltages will again have constant values, but probably
different constant values than they had before the switch closed.

Referring to Figure 5, we see that the inductor current is equal to the current in the 6
resistor. Consequently,

o
0.35
( ) 12 6
0.35
( ) 2 A when 0
6 6
t
v t e
t
i t e t


= = = > (3)

In the absence of unbounded voltages, the in any inductor is continuous. Consequently, the value
of the inductor current immediately before t = 0 is equal to the value immediately after t = 0.
Figure 6 shows the circuit before t = 0, when the switch is open and the circuit is at steady state.
The open switch is modeled as an open circuit. An inductor in a steady-state dc circuit acts like a
short circuit, so a short circuit replaces the inductor. The current in that short circuit is the steady
state inductor current, i(0). Apply KVL to the loop in Figure 6 to get
167

( ) ( ) ( ) ( )
1 2
1 2
24
0 0 6 0 24 0 0
6
R i R i i i
R R
+ + = =
+ +


The value of i(0) can also be obtained by setting t = 0 in Equation 3. Do so gives

( )
0
0 2 1 A i e = =

Consequently,

1 2
1 2
24
1
6
R R
R R
= + =
+ +
18 (4)






Figure 6 The circuit from Figure 5, at steady state when t < 0.





Figure 7 The circuit from Figure 5, at steady state when t > 0.
168
Next, consider the circuit after the switch closes. Eventually (certainly as t ) the
circuit will again be at steady state. Figure 7 shows the circuit at t = , when the switch is closed
and the circuit is at steady state. The closed switch is modeled as a short circuit. The combination
of resistor and a short circuit connected is equivalent to a short circuit. Consequently, a short
circuit replaces the switch and the resistor R
1
. An inductor in a steady-state dc circuit acts like a
short circuit, so a short circuit replaces the inductor. The current in that short circuit is the steady
state inductor current, i(). Apply KVL to the loop in Figure 7 to get

( ) ( ) ( )
2
2
24
0 6 0 24 0 0
6
R i i i
R
+ = =
+


The value of i() can also be obtained by setting t = in Equation 1. Doing so gives

( ) 2 2 i e

= = A
Consequently
2
2
24
2 6
6
R
R
= =
+


Substituting this value into Equation 4 gives

1
12 R =




Figure 8 The circuit that is used to determine R
t
.

Finally, the exponential part of i(t) is known to be of the form
t

e where
t
L
R
= and R
t

is the Thevenin resistance of the part of the circuit that is connected to the inductor. Figure 8
shows the circuit that is used to determine R
t
. An short circuit has replaced combination of
resistor R
1
and the closed switch. Independent sources are set to zero when calculating R
t
, so the
voltage source has been replaced by an short circuit. From Figure 8

169
2
6 6 6 12
t
R R = + = + =
so
12
t
L L
R
= =
From Equation 1
0.35 2.857 s
t
t

= =
Consequently,

2.857 34.29 H
12
L
L = =



Example 3:
The circuit shown in Figure 9 is at steady state before the switch opens at time t = 0. The input to
the circuit is the voltage of the left-hand voltage source, 8 V. The output of this circuit is the
voltage across the capacitor, v(t). Determine the value of the capacitor voltage,
50 ms after the switch opens.



Figure 9 The circuit considered in Example 3.

Solution: See Example 8.3-1 of Introduction to Electric Circuits by R.C. Dorf and J.A. Svoboda.
170
Example 4:
The circuit shown in Figure 10 is at steady state before the switch closes at time
t = 0. The input to the circuit is the current of the current source, 4 mA. The output of this circuit
is the current in the inductor, i(t). How long will it take the inductor current to reach 2 mA?


Figure 10 The circuit considered in Example 4.

Solution: See Example 8.3-2 of Introduction to Electric Circuits by R.C. Dorf and J.A. Svoboda.

Example 5:
The circuit shown in Figure 11 is at steady state before the switch closes at time
t = 0. The input to the circuit is the voltage of the voltage source, 12 V. The output of this circuit
is the voltage across the capacitor, v(t). Determine the capacitor voltage, v(t), for t > 0.


Figure 11 The circuit considered in Example 5.

Solution: See Example 8.3-3 of Introduction to Electric Circuits by R.C. Dorf and J.A. Svoboda.
171
Example 6:
The circuit shown in Figure 12 is at steady state before the switch closes at time
t = 0. The input to the circuit is the voltage of the voltage source, 12 V. The output of this circuit
is the current in the inductor, i(t). Determine the inductor current, i(t), for t > 0.



Figure 12 The circuit considered in Example 6.

Solution: See Example 8.3-4 of Introduction to Electric Circuits by R.C. Dorf and J.A. Svoboda.
21. Complex Arithmetic

Introduction

Each problem in this problem set presents an equation involving complex numbers and variables.
Use complex arithmetic to determine the values of the variables.

Appendix B of Introduction to Electric Circuits by R.C. Dorf and J.A Svoboda provides a review
of complex arithmetic.


Worked Examples

Example 1
Given
45
6
5
15 8
j j
e Ae
j

=
+

Find the values of A and .

172
Solution:

( ) 45 152 45 45 107
152
6 6 30
5 5 1.76
15 8 17 17
j j j
j
e e e
j e

= = =
+
j
e





Example 2:
Given:
45
10
2.36
j
e
a j b

=
+

Find the values of a and b.

Solution:
45
45
10
4.24 3 3
2.36
j
j
e j a
e

= = = j b +

173
Example 3
Given:
( ) 3 8 3
j
2 Ae j

+ = j
Find the values of A and .

Solution:
( )
90
90 111 21
111
32 32 32
3.75
3 8 8.54 8.54
j
j j j
j
j e
Ae e e
j e

= = = =
+



Example 4:
Given:
( )
135
2 3 5
j j
Ae j e

= + +
Find the values of A and .

Solution:
( ) ( ) (
( ) (
135
161
2 3 5 2 3 3.54 3.54
2 3.54 3 3.54
1.54 0.54
1.63
j j
j
Ae j e j j
j j
j
e


= + + = + +
= +
=
=
)
)



Example 5:
Given:
15
4 3
2
j
j
j
Ae
e

=
Find the values of A and .

Solution:
( )
37
37 15 52
15 15
4 3 5 5
2.5
2 2 2
j
j j j
j j
j e
Ae e e
e e

= = = =

174
Example 6:
Given:
135
5 13
6
j
j
a j b
e

+
= +
Find the values of a and b.

Solution:
( )
111
111 135
135 135
24
5 13 13.9 13.9
6 6 6
2.32 2.12 0.94
j
j
j j
j
j e
a j b e
e e
e j




+
+ = = =
= =



Example 7:
Given:
6
4 3
j
j
a j b

=
+

Find the values of a and b.

Solution:
( )
90
90 143
143
53
6 6 6
4 3 5 5
1.2 0.722 0.958
j
j
j
j
j e
a j b e
j e
e j

+

+ = = =

= = +



Example 8
Given:
( )
120 15
6 4 3 2
j j
e j e a

+ + = + j b
Find the values of a and b.

Solution:

( ) ( )
( )
( )
( )
120 120
120 120
120 120 240
6 4 3 1.93 0.52 6 2.07 3.52
6 4.08
6 4.08 24.48
12.2 21.2
j j
j j
j j
a j b e j j e j
e e
e e
j


+
+ = + + + = +
=
= =
=


175
Example 9:
Given:
120
4 3
j
Ae j b

+ = + j
Find the values of A and b.

Solution:
120
4 3
0.5 0.866 4 3
j
Ae j b j
A j A j b

+ = +
+ + = + j
18


Equating real and imaginary parts:

0.5 4 8 A A = =
and
0.866 3 3 0.866 8 3.93 A b b + = = =


Example 10:
Given:
( )
120
6 4 8
j j
e j b e

+ + =
Find the values of A and .

Solution:

( )
120
120
120
6 4 8 18
18
4 8cos 8sin 3 1.5
6
j j
j
j
e j b e
j b j e j
e

+ + =
+ + + = = = 2.6

Equating real and imaginary parts:

8 cos 2.5 71.8 = =
and
8 sin 2.6 2.6 8 cos(71.8 ) 10.2 b b + = = =


176
Example 11:
Given:
( )
60
4 2 2
j
a j j Ae + =
Find the values of a and A.

Solution:
( )
( )
60
4 2 2
8 2 2 0.5 0.866
10 2 0.5 0.866
j
a j j Ae
j a A j
j a A j A

+ =
+ = +
= +
A


Equating real and imaginary parts:

10 0.5 20 A A = =

and
( ) 2 0.866 20 8.66 a a = =


177
22. Simple AC Circuits

Introduction

Each problem in this problem set involves the steady state response of a linear, time-invariant
circuit to a single sinusoidal input. Such a response is known to be sinusoidal and to have the
same frequency as the input. The linear, time invariant circuits in this problem set consist of
resistors, capacitors and inductors. The input to each circuit is either the voltage of an
independent voltage source or the current of an independent current source.

The circuits in this problem set

1. Are linear and time-invariant
2. Have a single sinusoidal input
3. Are at steady state

Consequently, these circuits can be represented in the frequency-domain, using impedances and
phasors. Analysis in the frequency-domain is preferred to analysis in the time-domain because
the frequency-domain analysis involves algebraic equations rather than differential equations.
Unfortunately, solving the frequency domain equations requires algebra and arithmetic with
complex numbers.

The steady-state response of circuits with sinusoidal inputs is the topic of Chapter 10 of
Introduction to Electric Circuits by R.C. Dorf and J.A Svoboda. In particular, phasors are
described in Section 10.6 and 10.7 and impedance is defined in Section 10.8. Table 10.7-1
summarizes the correspondence between the time domain and the frequency domain. Also,
Appendix B provides a review of complex arithmetic.

Worked Examples
Example 1:
Consider the circuit shown in Figure 1. The input to the circuit is the voltage of the voltage
source, v
s
(t). The output is the voltage across the resistor, v
o
(t). Determine inductance, L, of the
inductor.


Figure 1 The circuit considered in Example 1.

178
Solution: The input voltage is a sinusoid. The output voltage is also a sinusoid and has the same
frequency as the input voltage. Apparently the circuit has reached steady state. Consequently, the
circuit in Figure 1 can be represented in the frequency domain, using phasors and impedances.
Figure 2 shows the frequency domain representation of circuit from Figure 1. The voltages V
s
()
and V
o
() in Figure 2 are the phasors corresponding to v
s
(t) and v
o
(t) from Figure 1. The inductor
and the resistor are represented as impedances in Figure 2. The impedance of the inductor is
, as shown in Figure 2. ( ) 1 j L j L j L = =



Figure 2 The circuit from Figure 1, represented in the frequency domain, using impedances and
phasors.

The current I() in Figure 2 is given by

( )
( ) 2.77 307
0.396 307 A
7 7
o


= = =
V
I

The inductor voltage, V
L
(), in Figure 2 is given by


( ) ( ) ( ) 4.41 178 2.77 307
3.427 37
= =
=
L s o
V V V
The impedance of the inductor is given by


( )
( )
3.427 37
8.65 270 8.65
0.396 307
j L j


= = = =

L
V
I


Therefore L = 8.65 H.

Alternate Solution: Apply the voltage divider principle to the circuit in Figure 2 to get

( )
7
2.27 307 4.41 178
7 j L
=
+

Doing some algebra gives

179
( )( )
7 2.27 307 2.27 307
0.628 51
7 4.41 178 1 180 4.41 178 j L

= = =
+


Solving for L gives

( )
( )
7 1 0.628 51
8.66 0.013 8.65 H
0.628 51
L j
j

= =



Example 2:
Consider the circuit shown in Figure 3. The input to the circuit is the voltage of the voltage
source, v
s
(t). The output is the voltage across the capacitor, v
o
(t). Determine resistance, R, of the
resistor.


Figure 3 The circuit considered in Example 2.

Solution: The input voltage is a sinusoid. The output voltage is also a sinusoid and has the same
frequency as the input voltage. Apparently the circuit has reached steady state. Consequently, the
circuit in Figure 3 can be represented in the frequency domain, using phasors and impedances.
Figure 4 shows the frequency domain representation of circuit from Figure 3. The voltages V
s
()
and V
o
() in Figure 4 are the phasors corresponding to v
s
(t) and v
o
(t). The capacitor and the
resistor are represented as impedances in Figure 4. The impedance of the capacitor was
calculated as

( )
1 1
0.997
3 0.3342
j j j j
C
= =




Figure 4 The circuit from Figure 3, represented in the frequency domain, using impedances and
phasors.
180

The current I() in Figure 3 is given by

( )
( ) 1.89 230
1.89 320 A
1 90
o
j


= = =

V
I

The resistor voltage, V
R
(), in Figure 4 is given by


( ) ( ) ( ) 7.83 126 1.89 230
7.597 40
= =
=
R s o
V V V
The impedance of the resistor is given by


( )
( )
7.597 40
4.02 360 4
1.89 320
R


= = =

R
V
I


Therefore R = 4 .

Alternate Solution: Apply the voltage divider principle to the circuit in Figure 4 to get

( ) 1.89 230 7.83 126
j
j R

=
+

Doing some algebra gives

( )( )
1.89 230 1.89 230
0.241 76
7.83 126 1 180 7.83 126
j
j R

= = =
+


Solving for L gives

( ) ( )
0.241 76 1
4.02 0.0022 4
0.241 76
j
R j

= =


181
Example 3:
Consider the circuit shown in Figure 5. The input to the circuit is the voltage of the voltage
source, v
s
(t). The output is the voltage across the inductor, v
o
(t). Determine amplitude, A, of v
o
(t).



Figure 5 The circuit considered in Example 3.


Solution: The input voltage is a sinusoid. The output voltage is also a sinusoid and has the same
frequency as the input voltage. Apparently the circuit has reached steady state. Consequently, the
circuit in Figure 5 can be represented in the frequency domain, using phasors and impedances.
Figure 6 shows the frequency domain representation of circuit from Figure 5. The voltages V
s
()
and V
o
() in Figure 6 are the phasors corresponding to v
s
(t) and v
o
(t). The inductor and the
resistor are represented as impedances. The impedance of the inductor is
, as shown in Figure 6. ( )( ) 4 0.54 2.16 j L j j = =



Figure 6 The circuit from Figure 5, represented in the frequency domain, using impedances and
phasors.


Apply the voltage divider principle to the circuit in Figure 6 to get

( )
2.16
311 7.28 77 4.254 49 4.254 311
3 2.16
j
A
j
= = =
+


Therefore A = 4.254 V.
182
Example 4:
Consider the circuit shown in Figure 7. The input to the circuit is the voltage of the voltage
source, v
s
(t). The output is the voltage across the resistor, v
o
(t). Determine capacitance, C, of the
capacitor.


Figure 7 The circuit considered in Example 4.


Solution: The input voltage is a sinusoid. The output voltage is also a sinusoid and has the same
frequency as the input voltage. Apparently the circuit has reached steady state. Consequently, the
circuit in Figure 7 can be represented in the frequency domain, using phasors and impedances.
Figure 8 shows the frequency domain representation of circuit from Figure 7.



Figure 8 The circuit from Figure 7, represented in the frequency domain, using impedances and
phasors.


The current I() in Figure 3 is given by

( )
( ) 1.59 125
1.59 125 A
1 1 0
o


= = =

V
I



The capacitor voltage, V
C
(), in Figure 4 is given by


( ) ( ) ( ) 7.68 47 1.59 125
7.512 35
= =
=
C s o
V V V
The impedance of the capacitor is given by

183

( )
( )
1 7.512 35
4.725 90
2 1.59 125
j
C


= = =

C
V
I

Solving for C gives
( )
0.106 F
2 4.725 90
j
C

= =




Alternate Solution: Apply the voltage divider principle to the circuit in Figure 8 to get

( )
1
1.59 125 7.68 47
1
1
2
j
C
=
| |
+
|
\ .

Doing some algebra gives
2 1.59 125
0.207 78
2 7.68 47
C
C j

= =


Solving for C gives


( )
( )
( ) ( )
( )
( )
( )
( )
( )
0.207 78 1 90 0.207 78
2 0.207 78 1 2 0.043 0.202 1
0.207 168
2 0.957 0.202
0.207 168
0.106 0 0.106
2 0.978 168
j
C
j
j
j

= =
+

=
+

= = + =





Example 5:
Consider the circuit shown in Figure 9. The input to the circuit is the voltage of the voltage
source, v
s
(t). The output is the voltage across the capacitor, v
o
(t). Determine amplitude, A, of
v
o
(t).


Figure 9 The circuit considered in Example 5.


Solution: The input voltage is a sinusoid. The output voltage is also a sinusoid and has the same
frequency as the input voltage. Apparently the circuit has reached steady state. Consequently, the
184
circuit in Figure 9 can be represented in the frequency domain, using phasors and impedances.
Figure 10 shows the frequency domain representation of circuit from Figure 9. The impedance of
the capacitor was calculated as


( ) ( )
1 1
1.07
1 0.933
j j j
C
= =

Apply the voltage divider principle to the circuit in Figure 10 to get

( )
1.07
64 7.2 139 1.864 64
4 1.07
j
A
j

= =


Therefore A = 1.864 V.


Figure 10 The circuit from Figure 9, represented in the frequency domain, using impedances and
phasors
185
23. More AC Circuits
Introduction

Each problem in this problem set involves the steady state response of a linear, time-invariant
circuit to a single sinusoidal input. Such a response is known to be sinusoidal and to have the
same frequency as the input. The linear, time invariant circuits in this problem set consist of
resistors, capacitors and inductors. The input to each circuit is either the voltage of an
independent voltage source or the current of an independent current source.

The circuits in this problem set

4. Are linear and time-invariant
5. Have a single sinusoidal input
6. Are at steady state

Consequently, these circuits can be represented in the frequency-domain, using impedances and
phasors. Analysis in the frequency-domain is preferred to analysis in the time-domain because
the frequency-domain analysis involves algebraic equations rather than differential equations.
Unfortunately, solving the frequency domain equations requires algebra and arithmetic with
complex numbers.

The steady-state response of circuits with sinusoidal inputs is the topic of Chapter 10 of
Introduction to Electric Circuits by R.C. Dorf and J.A Svoboda. In particular, phasors are
described in Section 10.6 and 10.7 and impedance is defined in Section 10.8. Table 10.7-1
summarizes the correspondence between the time domain and the frequency domain. Also,
Appendix B provides a review of complex arithmetic.

186
Worked Examples
Example 1:
Consider the circuit shown in Figure 1. The input to the circuit is the voltage of the voltage
source, v
s
(t) = . The output is the voltage across the 5 W resistor, v ( ) 9.10 cos 3 80 V t +
4.43 c
o
(t).
Determine capacitance, C, of the capacitor and the resistance, R, of the resistor required to cause
the output voltage to be v
o
(t) = . ( ) os 3 93 V t +



Figure 1 The circuit considered in Example 1.

Solution: The input voltage is a sinusoid. The output voltage is also a sinusoid and has the same
frequency as the input voltage. Apparently the circuit has reached steady state. Consequently, the
circuit in Figure 1 can be represented in the frequency domain, using phasors and impedances.
Figure 2 shows the frequency domain representation of the circuit from Figure 1. The voltages
V
s
() and V
o
() in Figure 2 are the phasors corresponding to v
s
(t) and v
o
(t) from Figure 1. The
capacitor and the resistors are represented as impedances in Figure 2. The impedance of the
capacitor is
1
3
j j
C C
=
1
where 3 radians/second is the value of the frequency of v
s
(t).

The impedance of the capacitor is connected in parallel with the impedance of the resistor. These
parallel impedances are replaced be the equivalent impedance, Z
e
, in Figure 3. The equivalent
impedance, Z
e
(w), is related to the impedances of the resistor and capacitor by the equation


( )
1 1 1 1
3
1
3
j C
R R
j
C

= + = +

e
Z
(1)
187


Figure 2 The circuit from Figure 1, represented in the frequency domain, using impedances and
phasors.



Figure 3 The circuit from Figure 2, after replacing parallel impedances with the equivalent
impedance.



Applying the voltage-division principle to the circuit in Figure 3 gives

( ) ( )
e
5
5

| |
=
|
+
\ .
o
V
Z

s
V (2)
Solve Equation 2 for Z
e
to get

( )
( ) ( ) ( )
( )
e
5

=
s o
o
V V
Z
V
(3)

Values of the phasor voltages, V
s
() and V
o
(), are given in Figure 2. Substituting these values
into Equation 3 gives
188

( )
( )
( ) ( ( ) )
( ) ( )
( )
( )
e
5 9.1 80 4.43 93
4.43 93
5 1.58 8.96 .231 4.42
4.43 93
5 1.81 4.54 5 4.89 68
4.43 93 4.43 93
5 4.89
68 93
4.43
5.52 25 5.00 2.33
j j
j
j


=

+ +
=

+
= =

=
= =
Z



Substituting this value of Z
e
(w) into Equation 1 gives

( ) ( )
1 1 1
3 0 25 0.181 25 0.164 0.0765
5.52 25 5.52
j C j
R
+ = = = = +

(4)

Equation 4 indicates that two complex numbers are equal. Therefore the real parts of these
complex numbers are equal and, also, the imaginary parts of the complex numbers are equal.
Equating the real parts gives
1
0.164 6.1 R
R
= =
Equating the imaginary parts gives

3 0.0765 0.0255 F 25.5 mF C C = = =

Therefore the required value of the resistance R in Figure 1 is R = 6.1 W and the required value
of the capacitance C is C = 25.5 mF.



Alternate derivation of Equation 2: If we dont recall the voltage division principle, we can
obtain Equation 2 by applying Kirchhoffs Voltage Law (KVL) and Ohms Law to the circuit in
Figure 3. First, we apply KVL to the single mesh in the circuit to get

( ) ( ) ( ) 0 +
a o s
V V V =



Next, we use Ohms Law to express the voltages V
a
() and V
o
() in terms of the current I()

( ) ( ) ( ) ( ) and 5 = =
a e o
V Z I V I
Therefore
( ) ( ) ( ) 5 0 + =
e s
Z I I V

Next, we solve this equation for I()
189
( )
( )
5

=
+
s
e
V
I
Z


Finally, we apply Ohms Law to get

( ) ( ) ( )
e
5
5
5

| |
= =
|
+
\ .
o s
V I V
Z

) )



Example 2:
Consider the circuit shown in Figure 4. The input to the circuit is the voltage of the voltage
source, v
s
(t), and the output is the voltage across the 2 W resistor, v
o
(t). When the input is v
s
(t) =
, the corresponding output is v ( 5.59 cos 2 185 V t +
o
(t) = . Determine
the voltage across the 7 W resistor, v
( 2.97 cos 2 146 V t +
a
(t), and also the value of the inductance, L, of the inductor.



Figure 4 The circuit considered in Example 2.


Solution: The input voltage is a sinusoid. The output voltage is also a sinusoid and has the same
frequency as the input voltage. Apparently the circuit has reached steady state. Consequently, the
circuit in Figure 4 can be represented in the frequency domain, using phasors and impedances.
Figure 5 shows the frequency domain representation of the circuit from Figure 4. The voltages
V
s
(),V
a
() and V
o
() in Figure 5 are the phasors corresponding to v
s
(t), v
a
(t) and v
o
(t) from
Figure 4. The inductor and the resistors are represented as impedances in Figure 5. The
impedance of the inductor is where 2 radians/second is the value of the frequency
of v
2 j L j L =
s
(t).



190



Figure 5 The frequency domain representation of the circuit from Figure 4. The encircled
numbers label the nodes of the circuit.


First, we calculate the value of V
a
(). Apply KVL to the mesh in Figure 5 that consists of the
two resistors and the voltage source to get

( ) ( ) ( ) ( ) ( )
( ) (
2.97 146 5.59 185
2.46 1.66 5.57 0.49
3.08 2.15
3.76 35
j j
j
= =
= +
= +
=
a o s
V V V
)



The voltage across the 7 W resistor, v
a
(t), is the sinusoid corresponding to this phasor. Therefore

( ) ( ) 3.76 cos 2 35 V
a
v t t = +

We can determine the value of the inductance by applying Kirchhoffs Current Law (KCL) at
node 2 in Figure 4.
( ) ( ) ( )
0
2 7 2 j L

+ +
a a o
V V V
=

Solving this equation for j2L gives

( )
( ) ( )
14
2
2 7
j L

=
+
a
a o
V
V V


Substituting the values of the phasors V
a
() and V
o
() into this equation gives

191
( )
( )
( ) ( )
( ) ( )
( ) ( )
( )
14 3.76 35
2
2 3.08 2.15 7 ( 2.46 1.66)
14 180 3.76 35
11.06 15.92
14 180 37.6 35
19.38 125
14 37.6
180 35 125
19.38
2.72 90
2.72
j L
j j
j
j

=
+ + +

=
+

=

= +
=
=


Therefore the value of the inductance is
2.72
1.36
2
L = = H.

Example 3:
Consider the circuit shown in Figure 6. The input to the circuit is the voltage of the voltage
source, v
s
(t) = . The output is the voltage across the 4 W resistor, v ( ) 8.50 cos 5 31 V t +
4.20
o
(t).
Determine inductance, L, of the inductor and the resistance, R, of the resistor required to cause
the output voltage to be v
o
(t) = . ( ) cos 5 2 V t +



Figure 6 The circuit considered in Example 3.

Solution: The input voltage is a sinusoid. The output voltage is also sinusoid and has the same
frequency as the input voltage. Apparently the circuit has reached steady state. Consequently, the
circuit in Figure 6 can be represented in the frequency domain, using phasors and impedances.
Figure 7 shows the frequency domain representation of the circuit from Figure 6. The voltages
V
s
(), V
o
() and V
a
() in Figure 7 are the phasors corresponding to v
s
(t), v
o
(t) and v
a
(t) from
Figure 6. The inductor and the resistors are represented as impedances in Figure 7. The
impedance of the inductor is where 5 radians/second is the value of the frequency
of v
5 j L j = L
s
(t).
192



Figure 7 The circuit from Figure 6, represented in the frequency domain, using impedances and
phasors.


Figure 8 The circuit from Figure 7, after replacing parallel impedances with the equivalent
impedance.


The impedance of the inductor is connected in parallel with the impedance of the resistor. These
parallel impedances are replaced be the equivalent impedance, Z
e
, in Figure 8. The equivalent
impedance, Z
e
(w), is related to the impedances of the resistor and capacitor by the equation


( )
1 1 1 1
5 5
j
1
R j L R L
= + =
e
Z
(5)

Applying the voltage-division principle to the circuit in Figure 8 gives

( ) ( )
e
4
4

| |
=
|
+
\ .
o
V
Z

s
V (6)
Solve Equation 6 for Z
e
to get

( )
( ) ( ) ( )
( )
e
4

=
s o
o
V V
Z
V
(7)

193
Values of the phasor voltages, V
s
() and V
o
(), are given in Figure 7. Substituting these values
into Equation 7 gives


( )
( )
( ) ( ( ) )
( ) ( )
( )
( )
e
4 8.5 31 4.2 2
4.2 2
4 7.29 4.38 4.2 0.15
4.2 2
4 3.09 4.23 4 5.24 54
4.2 2 4.2 2
4 5.24
54 2
4.2
4.99 52 3.07 3.93
j j
j
j


=

+ +
=

+
= =

=
= = +
Z



Substituting this value of Z
e
(w) into Equation 5 gives

( )
1 1 1 1
0 52 0.20 52 0.123 0.158
5 4.99 52 4.99
j j
R L
= = = =

(8)

Equation 8 indicates that two complex numbers are equal. Therefore the real parts of these
complex numbers are equal and, also, the imaginary parts of the complex numbers are equal.
Equating the real parts gives

1
0.123 8.1 R
R
= =

Equating the imaginary parts gives

1
5 1.27 H
0.158
L L = =

Therefore the required value of the resistance R in Figure 6 is R = 8.1 W and the required value
of the inductance L is L = 1.27 H.
194
Example 4:
Consider the circuit shown in Figure 9. The input to the circuit is the voltage of the voltage
source, v
s
(t), and the output is the voltage across the 5 W resistor, v
o
(t). When the input is v
s
(t) =
, the corresponding output is ( ) 8.93 cos 2 54 V t +
v
o
(t) = 3.83 . Determine the voltage across the 9 W resistor, v ( ) cos 2 83 V t +
a
(t), and also the
value of the capacitance, C, of the capacitor.



Figure 9 The circuit considered in Example 4.

Solution: The input voltage is a sinusoid. The output voltage is also a sinusoid and has the same
frequency as the input voltage. Apparently the circuit has reached steady state. Consequently, the
circuit in Figure 9 can be represented in the frequency domain, using phasors and impedances.
Figure 10 shows the frequency domain representation of the circuit from Figure 9. The voltages
V
s
(),V
a
() and V
o
() in Figure 10 are the phasors corresponding to v
s
(t), v
a
(t) and v
o
(t) from
Figure 9. The capacitor and the resistors are represented as impedances in Figure 10. The
impedance of the capacitor is
1
2
j j
C C
=
1
)

where 2 radians/second is the value of the


frequency of v
s
(t).

First, we calculate the value of V
a
(). Applying KVL to the mesh in Figure 10 that consists of
the two resistors and the voltage source to get

( ) ( ) ( ) ( ) ( )
( ) (
3.83 83 8.93 54
0.47 3.80 5.25 7.22
4.78 3.42
5.88 216
j j
j
= =
= + +
=
=
a o s
V V V


The voltage across the 9 W resistor, v
a
(t), is the sinusoid corresponding to this phasor. Therefore

( ) ( ) 5.88 cos 2 216 V
a
v t t = +
195


Figure 10 The frequency domain representation of the circuit from Figure 9. The encircled
letters label the nodes of the circuit.

We can determine the value of the inductance by applying Kirchhoffs Current Law (KCL) at
node b in Figure 4.
( ) ( ) ( )
0
1
9 4
2
j
C

+ +

a a o
V V V
=
( ) ( )
( ) ( )
2 0
9 4
j C

+ +
a o
a
V V
V =
Solving this equation for gives 2 j C
( ) ( )
( )
4 9
2
36
j C

+
=

a o
a
V V
V


Substituting the values of the phasors V
a
() and V
o
() in this equation gives

( )
( )
( )
( ) ( )
( ) ( )
( ) ( )
4 4.78 3.42 9 (0.47 3.80)
2
36 5.88 216
14.89 20.52
36 5.88 216
25.35 126
36 180 5.88 216
25.35
126 180 216
36 5.88
0.120 90 0.120
j j
j C
j
j
+ +
=

+
=


=

= +
= =



Therefore the value of the capacitance is
0.12
0.06 60 mF
2
= = = C .
196
24. Power in AC Circuits

Introduction

Each problem in this problem set involves the steady state response of a circuit to a single
sinusoidal input. The circuits in this problem set consist of resistors, capacitors and inductors.
The input to each circuit is either the voltage of an independent voltage source or the current of
an independent current source.

Each problem in this problem set asks a question regarding the complex power received or
supplied by an element of the circuit. Circuit analysis in the frequency domain provides the
answers to these questions.

Complex power is discussed in Sections 11.5 and 11.6 of Introduction to Electric Circuits by
R.C. Dorf and J.A Svoboda. In particular, Table 11.5-1 summarizes the equations used to
calculate complex power in the frequency domain. Circuit analysis in the frequency-domain is
described in Sections 10.6 thru 10.11. Table 10.7-1 summarizes the correspondence between the
time domain and the frequency domain. Also, Appendix B provides a review of complex
arithmetic.

Worked Examples
Example 1:
Consider the circuit shown in Figure 1. The input to the circuit is the voltage of the voltage
source, v
s
(t). The output is the voltage across the inductor, v
o
(t). Determine the following:

a. The average power supplied by the voltage source.
b. The average power received by the resistor.
c. The average power received by the inductor.
d. The power factor of the impedance of the series connection of the resistor and inductor.




Figure 1 The circuit considered in Example 1.


197
Solution: The input voltage is a sinusoid. The output voltage is also a sinusoid and has the same
frequency as the input voltage. Apparently the circuit has reached steady state. Consequently, the
circuit in Figure 1 can be represented in the frequency domain, using phasors and impedances.
Figure 2 shows the frequency domain representation of the circuit from Figure 1. The voltages
V
s
() and V
o
() in Figure 2 are the phasors corresponding to v
s
(t) and v
o
(t) from Figure 1. The
inductor and the resistor are represented as impedances in Figure 2. The impedance of the
inductor is , as shown in Figure 2. ( )( ) 4 0.54 2.16 j L j j = =



Figure 2 The circuit from Figure 1, represented in the frequency domain, using impedances and
phasors.

The current I() in Figure 2 is calculated from V
o
() and the impedance of the inductor using
ohms Law:

( )
( )
( )
4.254 311 4.254
311 90 1.969 221 A
2.16 2.16 90 2.16 j


= = = =

o
V
I

Once we know I() we are ready to answer the questions asked in this problem.

a. The average power supplied by the source is calculated from I() and V
s
(). The average
power of the source is given by


( ) ( )
( ) ( ) ( )
( ) ( )
( )
( )
7.28 1.969
cos cos 77 221
2 2
7.167 cos 144 5.8 W

=
= =
S
S
V I
V I
(1)

Notice that I() and V
s
() adhere to the passive convention. Consequently, Equation 1 gives the
power received by the voltage source rather than the power supplied by the voltage source. The
power supplied is the negative of the power received. Therefore, the power supplied by the
voltage source is

5.8 W
S
P =
198

b. The resistor voltage, V
R
(), in Figure 2 is given by

( ) ( ) ( ) 3 1.969 221 5.907 221 V R = = =
R
V I

The average power received by the resistor is calculated from I() and V
R
():


( ) ( )
( ) ( ) ( )
( ) ( )
(
( )
5.907 1.969
cos cos 221 221
2 2
5.8 cos 0 5.8 W
R
P

= =
= =
R
R
V I
V I )
(2)

Notice that I() and V
R
() adhere to the passive convention. Consequently, P
R
is the power
received by the resistor, as required.

Alternately, the power received by a resistor can be calculated from the current I() and the
resistance, R. To see how, first notice that the voltage and current of a resistor are related by

( ) ( ) ( ) ( ) ( ) ( ) ( )
( ) ( )
( ) ( )
R
R R



=

= =

=

R
R R R
R
V I
V I V V I I
V I


Substituting these expressions for |V
R
()| and V
R
() into Equation 2 gives

( ) ( )
( ) ( ) ( )
( )
( ) ( )
2
2
cos
2 2
3 1.969
5.8 W
2
R
R R
P

= =
= =
I I I
I I


c. The average power received by the inductor is calculated from I() and V
o
():


( ) ( )
( ) ( ) ( )
( ) ( )
(
( )
4.254 1.969
cos cos 311 221
2 2
4.188 cos 90 0 W
L
P

= =
= =
o
o
V I
V I )
(3)

The phase angle of the inductor voltage is always 90 greater than the phase angle of the inductor
current. Consequently, the value of average power received by any inductor is zero.

d. The power factor of the impedance of the series connection of the resistor and inductor
can be calculated from I() and the voltage across the impedance. That voltage is V
R
()+V
o
(),
which is calculated by applying Kirchhoffs Voltage Law to the circuit in Figure 2:

199
( ) ( ) ( ) 0 + +
R o s
V V V =

=


( )
( )
( ) ( ) 7.28 77
(1 180 ) 7.28 77
7.28 257
+ = =
=
=
R o s
V V V

Now the power factor is calculated as

( ) ( )
pf cos ( ) ( ) ( ) cos(257 221 ) 0.809 = + = =
R o
V V I

The power factor is said to be lagging because 257-221=36>0.


Observation: Average power is conserved. In this example, that means that the average power
supplied by the voltage source must be equal to the sum of the average powers received by the
resistor and the inductor. This fact provides a check on the accuracy of our calculations.


Alternate Solution: If the value of V
o
() had not be given, then I() would be calculated by
writing and solving a mesh equation. Referring to Figure 2, the mesh equation is

( ) ( ) 3 2.16 7.28 77 0 j + + I I
Solving for I() gives

( )
( )( )
( )( )
1 180 7.28 77
7.28 77
3 2.16 3.697 36
1 7.28
(180 77 36) 1.969 221 A
3.697
j



= =
+
= + =
I



as before.

200
25. AC Circuits with Coupled Inductors
Introduction

The circuits in this problem set contain coupled inductors. Each problem involves the steady
state response of such a circuit to a single sinusoidal input. That input is either the voltage of an
independent voltage source or the current of an independent current source.

Circuit analysis in the frequency-domain provides the solutions to these problems.

Coupled inductors are described in Section 11.9 of Introduction to Electric Circuits by R.C. Dorf
and J.A Svoboda. In particular, Table 11.14-1 summarizes the equations that represent coupled
inductors in the frequency domain. Circuit analysis in the frequency domain is described in
Sections 10.6 thru 10.11. Table 10.7-1 summarizes the correspondence between the time domain
and the frequency domain.

Worked Examples

Example 1:
Consider the circuit shown in Figure 1. The input to the circuit is the current of the current
source, i
s
(t). The output is the voltage across the right-hand coil, v
o
(t). Determine the steady-state
output voltage, v
o
(t).



Figure 1 The circuit considered in Example 1.


Solution: The input current is a sinusoid and the circuit is at steady state. The output voltage is
also a sinusoid and has the same frequency as the input current. Consequently, the circuit can be
represented in the frequency domain, using phasors and impedances. Figure 2 shows the
frequency domain representation of the circuit from Figure 1.

201


Figure 2 The circuit from Figure 1, represented in the frequency domain, using impedances and
phasors.

The circuit in Figure 2 consists of a single mesh. The mesh current is equal to the current source
I
s
(). The current I
s
() enters the dotted end of both coils, consequently the voltage across the
right hand coil is given by


( ) ( ) ( ) ( ) ( )
( )(
10 5 15 15 0.707 47
15 90 0.707 47
(15)(0.707) (90 47)
10.605 137 V
j j j j = + = =
=
= +
=
o s s s
V I I I
)

In the time domain, the output voltage is given by



( ) ( ) 10.6 cos 5 137 V
o
v t t = +

Example 2:
Consider the circuit shown in Figure 3. The input to the circuit is the current of the current
source, i
s
(t). The output is the voltage across the right-hand coil, v
o
(t). Determine the steady-state
output voltage, v
o
(t).


Figure 3 The circuit considered in Example 2.

Solution: The circuit shown in Figure 3 is very similar to the circuit shown in Figure 1. There is
only one difference: the dot of the right-hand coil is located at the bottom of the coil in Figure 3
and at the top of the coil in Figure 1. As in Example 1, our first step is to represent the circuit in
202
the frequency domain, using phasors and impedances. Figure 4 shows the frequency domain
representation of the circuit from Figure 3.



Figure 4 The circuit from Figure 3, represented in the frequency domain, using impedances and
phasors.


The circuit in Figure 4 consists of a single mesh. The mesh current is equal to the current source
I
s
(). The current I
s
() enters the dotted end of the left-hand coil and enters the undotted end of
the right-hand coil. Consequently the voltage across the right-hand coil is given by


( ) ( ) ( ) ( ) ( )
( )(
10 5 5 5 0.707 47
5 90 0.707 47
(5)(0.707) (90 47)
3.535 137 V
j j j j = = =
=
= +
=
o s s s
V I I I
)

In the time domain, the output voltage is given by



( ) ( ) 3.535 cos 5 137 V
o
v t t = +


Example 3:
Consider the circuit shown in Figure 5. The input to the circuit is the current of the current
source, i
s
(t). The output is the voltage across the right-hand coil, v
o
(t). Determine the steady-state
output voltage, v
o
(t).



Figure 5 The circuit considered in Example 3.

203
Solution: The input current is sinusoid and the circuit is at steady state. The output voltage is
also a sinusoid and has the same frequency as the input current. Consequently, the circuit can be
represented in the frequency domain, using phasors and impedances. Figure 6 shows the
frequency domain representation of the circuit from Figure 5.




Figure 6 The circuit from Figure 5, represented in the frequency domain, using impedances and
phasors.


The coil currents, I
1
() and I
2
(), and the coil voltages, V
1
() and V
2
(), are labeled in Figure
6. Reference directions for these currents and voltages have been selected so that the current and
voltage of each coil adhere to the passive convention. Notice that one coil current, I
1
(), enters
the undotted end of its coil while the other coil current, I
2
(), enters the dotted of its coil.

The device equations for coupled coils are:

(1) ( ) ( ) ( )
2
16 8 j j =
1 1
V I I
2

and
(2) ( ) ( ) ( )
2
8 12 j j = +
1
V I I

The coils are connected in parallel, consequently V . Equating the expressions for
V
( ) ( )
1 2
= V
1
() and V
2
() in Equations 1 and 2 gives

( ) ( ) ( ) ( )
2 2
16 8 8 12 j j j j = +
1 1
I I I I

( ) ( )
2
24 20 j j =
1
I I

( ) ( )
2
5
6
=
1
I I

Apply Kirchhoffs Current Law (KCL) to the top node of the coils to get

204
( ) ( ) ( ) ( ) ( ) ( )
s 2 2 2
5 1
0.121 140
6 6
= = + = + =
1
I I I I I I
2
1

( ) ( )
2
6
0.121 140 0.066 140 A
11
= = I

Now the output voltage can be calculated using Equation 2:

( ) ( ) ( ) ( )
( ) ( )
( ) ( )
( )
( )
( )( )
2 2
2 2
2
8 12
8 5 12 6
5
8 12
6 6
5.333
5.333 90 0.066 140 0.352 230 V
j j
j j j
j

= = +
+
| |
= + =
|
\ .
=
= =
o 1
V V I I
I I
I
2
I



In the time domain, the output voltage is given by

( ) ( ) 0.352 cos 4 230 V
o
v t t = +



Example 4:
Consider the circuit shown in Figure 7. The input to the circuit is the current of the current
source, i
s
(t). The output is the voltage across the right-hand coil, v
o
(t). Determine the steady-state
output voltage, v
o
(t).



Figure 7 The circuit considered in Example 4.

Solution: The circuit shown in Figure 7 is very similar to the circuit shown in Figure 5. There is
only one difference: the dot of the left-hand coil is located at the bottom of the coil in Figure 5
and at the top of the coil in Figure 7. As in Example 3, our first step is to represent the circuit in
the frequency domain, using phasors and impedances. Figure 8 shows the frequency domain
representation of the circuit from Figure 7.

205


Figure 8 The circuit from Figure 7, represented in the frequency domain, using impedances and
phasors.


The coil currents, I
1
() and I
2
(), and the coil voltages, V
1
() and V
2
(), are labeled in Figure
8. Reference directions for these currents and voltages have been selected so that the current and
voltage of each coil adhere to the passive convention. Notice that the coil currents, I
1
() and
I
2
(), both enter the dotted of their respective coils.

The device equations for coupled coils are:

(3) ( ) ( ) ( )
2
16 8 j j = +
1 1
V I I
2

and
(4) ( ) ( ) ( )
2
8 12 j j = +
1
V I I

The coils are connected in parallel, consequently V . Equating the expressions for
V
( ) ( )
1 2
= V
1
() and V
2
() in Equations 3 and 4 gives

( ) ( ) ( ) ( )
2 2
16 8 8 12 j j j j + = +
1 1
I I I I

( ) ( )
2
8 4 j j =
1
I I

( ) ( )
2
1
2
=
1
I I

Apply Kirchhoffs Current Law (KCL) to the top node of the coils to get

( ) ( ) ( ) ( ) ( ) ( )
s 2 2 2
1 3
0.121 140
2 2
= = + = + =
1
I I I I I I
2

( ) ( )
2
2
0.121 140 0.08067 140 A
3
= = I

Now the output voltage can be calculated using Equation 4:

206
( ) ( ) ( ) ( )
( ) ( )
( )
( )( )
2 2
2 2
2
8 12
1
8 12
2
16
16 90 0.04033 140 1.291 230 V
j j
j j
j

= = +
| |
= +
|
\ .
=
= =
o 1
V V I I
I I
I


In the time domain, the output voltage is given by

( ) ( ) 1.291cos 4 230 V
o
v t t = +


Example 5:
Consider the circuit shown in Figure 9. The input to the circuit is the voltage of the voltage
source, v
s
(t). The output is the voltage across the right-hand coil, v
o
(t). Determine the steady-state
output voltage, v
o
(t).



Figure 9 The circuit considered in Example 5.

Solution: The input voltage is a sinusoid and the circuit is at steady state. The output voltage is
also a sinusoid and has the same frequency as the input voltage. Consequently, the circuit can be
represented in the frequency domain, using phasors and impedances. Figure 10 shows the
frequency domain representation of the circuit from Figure 9.



Figure 10 The circuit from Figure 9, represented in the frequency domain, using impedances and
phasors.

207
Notice that the current, I
2
(), in the right-hand coil is zero and that both coil currents, I
1
() and
I
2
(), enter the undotted ends of their respective coils. The voltage, V
1
(),across the left-hand
coil is given by

(5) ( ) ( ) ( ) ( ) 25 15 0 25 j j j = + =
1 1
V I I
1
=
=

Apply KVL to the mesh consisting of the voltage source, resistor and the left-hand coil to get

( ) ( ) 8 7.32 95 0 +
1 1
I V

Substituting the expression for V
1
() from Equation 5 gives

( ) ( ) 8 25 7.32 95 0 j +
1 1
I I
Now solve for I
1
() to get

( )
7.32 95 7.32 95
0.279 23 V
8 25 26.25 72 j


= = =
+
1
I



The voltage, V
o
(), across the right-hand coil is given by

( ) ( ) ( ) ( ) ( )
( )
15 10 0 15 15 0.279 23
(15 90 ) 0.279 23
4.185 113 V
j j j j = + = =
=
=
o 1 1
V I I


In the time domain, the output voltage is given by

( ) ( ) 4.185 cos 4 113 V
o
v t t = +


208
Example 6:
Consider the circuit shown in Figure 11. The input to the circuit is the voltage of the voltage
source, v
s
(t). The output is the voltage across the right-hand coil, v
o
(t). Determine the steady-state
output voltage, v
o
(t).




Figure 11 The circuit considered in Example 6.

Solution: The circuit shown in Figure 11 is very similar to the circuit shown in Figure 9. There
is only one difference: the dot of the right-hand coil is located at the bottom of the coil in Figure
9 and at the top of the coil in Figure 11. As in Example 5, our first step is to represent the circuit
in the frequency domain, using phasors and impedances. Figure 12 shows the frequency domain
representation of the circuit from Figure 11.



Figure 12 The circuit from Figure 11, represented in the frequency domain, using impedances
and phasors.

Notice that the current, I
2
(), in the right-hand coil is zero and that one of coil currents, I
1
(),
enters the undotted end of the coil while the other coil current, I
2
(), enters the dotted end of the
coil. The voltage, V
1
(),across the left-hand coil is given by

(6) ( ) ( ) ( ) ( ) 25 15 0 25 j j j = =
1 1
V I I
1
=

Apply KVL to the mesh consisting of the voltage source, resistor and the left-hand coil to get

( ) ( ) 8 7.32 95 0 +
1 1
I V

209
Substituting the expression for V
1
() from Equation 6 gives

( ) ( ) 8 25 7.32 95 0 j +
1 1
I I =
Now solve for I
1
() to get

( )
7.32 95 7.32 95
0.279 23 V
8 25 26.25 72 j


= = =
+
1
I



The voltage, V
o
(), across the right-hand coil is given by

( ) ( ) ( ) ( ) ( )
( )
15 10 0 15 15 0.279 23
(15 90 ) 0.279 23 4.185 67 V
j j j j = + = =
= =
o 1 1
V I I

In the time domain, the output voltage is given by

( ) ( ) 4.185 cos 4 67 V
o
v t t =



Example 7:
Consider the circuit shown in Figure 13. The input to the circuit is the voltage of the voltage
source, v
s
(t). The output is the voltage across the right-hand coil, v
o
(t). Determine the steady-state
output voltage, v
o
(t).



Figure 13 The circuit considered in Example 7.


Solution: The input voltage is a sinusoid and the circuit is at steady state. The output voltage is
also a sinusoid and has the same frequency as the input voltage. Consequently, the circuit can be
represented in the frequency domain, using phasors and impedances. Figure 14 shows the
frequency domain representation of the circuit from Figure 13.

210


Figure 14 The circuit from Figure 13, represented in the frequency domain, using impedances
and phasors.


The circuit in Figure 14 consists of a single mesh. Notice that the mesh current, I(), enters the
undotted ends of both coils. Apply KVL to the mesh to get

( ) ( ) ( ) ( ) ( ) ( ) ( )
5 12 6 6 15 5.94 140 0 j j j j + + + + I I I I I =

( ) ( ) ( ) 5 12 6 6 15 5.94 140 0 j j j j + + + + = I I

( )
( )
5.94 140 5.94 140 5.94 140
0.151 57 A
5 12 6 6 15 5 39 39.3 83 j j


= = = =
+ + + + +
I



Notice that the voltage, V
o
(), across the right-hand coil and the mesh current, I(), adhere to
the passive convention. The voltage across the right-hand coil is given by is given by

( ) ( ) ( ) ( ) ( )
( )
15 6 21 21 0.151 57
(21 90 ) 0.151 57
3.17 147 V
j j j j = + = =
=
=
o
V I I I

In the time domain, the output voltage is given by

( ) ( ) 3.17 cos 3 147 V
o
v t t = +


211
Example 8:
Consider the circuit shown in Figure 15. The input to the circuit is the voltage of the voltage
source, v
s
(t). The output is the voltage across the right-hand coil, v
o
(t). Determine the steady-state
output voltage, v
o
(t).



Figure 15 The circuit considered in Example 6.

Solution: The circuit shown in Figure 15 is very similar to the circuit shown in Figure 13. There
is only one difference: the dot of the left-hand coil is located at the right of the coil in Figure 13
and at the left of the coil in Figure 15. As in Example 7, our first step is to represent the circuit in
the frequency domain, using phasors and impedances. Figure 16 shows the frequency domain
representation of the circuit from Figure 15.



Figure 16 The circuit from Figure 15, represented in the frequency domain, using impedances
and phasors.

The circuit in Figure 14 consists of a single mesh. Notice that the mesh current, I(), enters the
dotted end of the left-hand coil and the undotted end of the right-hand coil. Apply KVL to the
mesh to get

( ) ( ) ( ) ( ) ( ) ( ) ( )
5 12 6 6 15 5.94 140 0 j j j j + + + I I I I I =

( ) ( ) ( ) 5 12 6 6 15 5.94 140 0 j j j j + + = I I

( )
( )
5.94 140 5.94 140 5.94 140
0.376 68.4 A
5 12 6 6 15 5 15 15.8 71.6 j j


= = = =
+ + +
I
212

Notice that the voltage, V
o
(), across the right-hand coil and the mesh current, I(), adhere to
the passive convention. The voltage across the right-hand coil is given by is given by

( ) ( ) ( ) ( ) ( )
( )
15 6 9 9 0.376 68.4
(9 90 ) 0.376 68.4
3.38 158.4 V
j j j j = = =
=
=
o
V I I I

In the time domain, the output voltage is given by

( ) ( ) 3.38 cos 3 158.4 V
o
v t t = +


Example 9:
Consider the circuit shown in Figure 17. The input to the circuit is the voltage of the voltage
source, v
s
(t). The output is the voltage across the right-hand coil, v
o
(t). Determine the steady-state
output voltage, v
o
(t).



Figure 17 The circuit considered in Example 9.

Solution: The input voltage is a sinusoid and the circuit is at steady state. The output voltage is
also a sinusoid and has the same frequency as the input voltage. Consequently, the circuit can be
represented in the frequency domain, using phasors and impedances. Figure 18 shows the
frequency domain representation of the circuit from Figure 17.

The coil currents, I
1
() and I
2
(), and the coil voltages, V
1
() and V
2
(), are labeled in Figure
18. Reference directions for these currents and voltages have been selected so that the current
and voltage of each coil adhere to the passive convention. Notice that both coil currents, I
1
()
and I
2
(), enter the undotted ends of their respective coils.
213


Figure 18 The circuit from Figure 17, represented in the frequency domain, using impedances
and phasors.


The device equations for coupled coils are:

(7) ( ) ( ) ( )
2
16 8 j j = +
1 1
V I I
2

and
(8) ( ) ( ) ( )
2
8 20 j j = +
1
V I I

The coils are connected in parallel, consequently V . Equating the expressions for
V
( ) ( )
1 2
= V
1
() and V
2
() in Equations 7 and 8 gives

( ) ( ) ( ) ( )
2 2
16 8 8 20 j j j j + = +
1 1
I I I I

( ) ( )
2
8 12 j j =
1
I I

( ) ( )
2
3
2
=
1
I I

Apply Kirchhoffs Current Law (KCL) to the top node of the coils to get

( ) ( ) ( ) ( ) ( ) ( )
2 2 2 2
3 5
2 2
= + = + =
1
I I I I I I
Therefore
( ) ( ) ( ) ( )
2
3
and
5 5
=
1
I I I I
2
= (9)

Substituting the expressions for I
1
() and I
2
() from Equation 9 into Equation 7 gives

214
( ) ( ) ( )
( ) ( )
( ) ( )
16 3 8 2
3 2
16 8 12.8
5 5 5
j j j j
+
| | | |
= + = =
| |
\ . \ .
1
V I I I I
=
=
(10)

Apply KVL to the mesh consisting of the voltage source, resistor and left-hand coil to get

( ) ( ) 4 5.7 158 0 +
1
I V
Using Equation 10 gives

( ) ( ) 4 12.8 5.7 158 0 j + I I
Solving for I () gives
( )
5.7 158 5.7 158
0.425 85 A
4 12.8 13.41 73 j


= = =
+
I



Now the output voltage can be calculated using Equation 10:

( ) ( ) ( )
( )
( )( )
12.8
12.8 0.425 85
12.8 90 0.425 85 5.44 175 V
j
j
= =
=
= =
o 1
V V I


In the time domain, the output voltage is given by

( ) ( ) 5.44 cos 4 175 V
o
v t t = +



Example 10:
Consider the circuit shown in Figure 19. The input to the circuit is the voltage of the voltage
source, v
s
(t). The output is the voltage across the right-hand coil, v
o
(t). Determine the steady-state
output voltage, v
o
(t).



Figure 19 The circuit considered in Example 10.

215
Solution: The circuit shown in Figure 19 is very similar to the circuit shown in Figure 17. There
is only one difference: the dot of the right-hand coil is located at the bottom of the coil in Figure
17 and at the top of the coil in Figure 19. Figure 20 shows the frequency domain representation
of the circuit from Figure 19.



Figure 20 The circuit from Figure 19, represented in the frequency domain, using impedances
and phasors.


The coil currents, I
1
() and I
2
(), and the coil voltages, V
1
() and V
2
(), are labeled in Figure
20. Reference directions for these currents and voltages have been selected so that the current
and voltage of each coil adhere to the passive convention. Notice that one of coil currents, I
1
(),
enters the undotted end of the coil while the other coil current, I
2
(), enters the dotted end of the
coil.

The device equations for coupled coils are:

(11) ( ) ( ) ( )
2
16 8 j j =
1 1
V I I
2

and
(12) ( ) ( ) ( )
2
8 20 j j = +
1
V I I

The coils are connected in parallel, consequently V . Equating the expressions for
V
( ) ( )
1 2
= V
1
() and V
2
() in Equations 11 and 12 gives

( ) ( ) ( ) ( )
2 2
16 8 8 20 j j j j = +
1 1
I I I I

( ) ( )
2
24 28 j j =
1
I I

( ) ( ) ( )
2 2
28 7
24 6
= =
1
I I I

Apply Kirchhoffs Current Law (KCL) to the top node of the coils to get
216

( ) ( ) ( ) ( ) ( ) ( )
2 2 2 2
7 1
6 6
= + = + =
1
I I I I I I
3

Therefore
( ) ( ) ( ) ( )
2
7
and
13 13
=
1
I I I I
6
= (13)

Substituting the expressions for I
1
() and I
2
() from Equation 13 into Equation 11 gives

( ) ( ) ( )
( ) ( )
( ) ( )
16 7 8 6
7 6
16 8 4.9
13 13 13
j j j j

| | | |
= = =
| |
\ . \ .
1
V I I I I
=
=
(14)

Apply KVL to the mesh consisting of the voltage source, resistor and left-hand coil to get

( ) ( ) 4 5.7 158 0 +
1
I V
Using Equation 14 gives

( ) ( ) 4 4.9 5.7 158 0 j + I I
Solving for I () gives
( )
5.7 158 5.7 158
0.901 107 A
4 4.9 6.325 51 j


= = =
+
I



Now the output voltage can be calculated using Equation 14:

( ) ( ) ( )
( )
( )( )
4.9
4.9 0.901 107
4.9 90 0.901 107 4.41 197 V
j
j
= =
=
= =
o 1
V V I


In the time domain, the output voltage is given by

( ) ( ) 4.41cos 4 197 V
o
v t t = +


217
26. AC Circuits with Transformers

Introduction

The circuits in this problem set contain ideal transformers. Each problem involves the steady
state response of such a circuit to a single sinusoidal input. That input is either the voltage of an
independent voltage source or the current of an independent current source.

Circuit analysis in the frequency-domain provides the solutions to these problems.


The solutions to these examples use the circuit equivalence illustrated in Figure 1.


(a)


(b)

Figure 1 The circuit shown in (b) is equivalent to the circuit shown in (a).


Figure 1a shows Circuit A connected to the left-hand coil of an ideal transformer. An impedance,
Z(w), is connected in parallel with the right-hand coil of the ideal transformer. In Figure 1b the
ideal transformer and impedance have been replaced by a single equivalent impedance, Z
eq
(w).
The equivalent impedance is related to the original impedance by
218

( ) ( )
1
2
2
N
N

| |
=
|
\ .
eq
Z Z

The two circuits in Figure 1 are equivalent. All the currents and voltages of Circuit A, including
I(w) and V(w), are the same in Figure 1b as they are in Figure 1a. We can determine the values
of I(w) and V(w) in Figure 1a by calculating values of I(w) and V(w) in Figure 1b.

Ideal transformers are described in Section 11.10 of Introduction to Electric Circuits by R.C.
Dorf and J.A Svoboda. In particular, Table 11.14-2 summarizes the equations that represent
ideal transformers in the frequency-domain. Circuit analysis in the frequency-domain is
described in Sections 10.6 thru 10.11. Table 10.7-1 summarizes the correspondence between the
time-domain and the frequency-domain.




Worked Examples

Example 1:
Consider the circuit shown in Figure 2. The input to the circuit is the voltage of the voltage
source, v
s
(t). The output is the voltage across the 9 W resistor, v
o
(t). Determine the output
voltage, v
o
(t).



Figure 2 The circuit considered in Example 1.


Solution: In Figure 2, the 9 W resistor is connected in parallel with the right hand coil of the
transformer. Replace the transformer and the 9 W resistor with an equivalent resistor as shown in
Figure 3.
219


Figure 3 The circuit from Figure 2, after replacing the transformer and the 9 W resistor with an
equivalent resistor


The equivalent resistance is given by

2
2
9 4
3
eq
R
| |
= =
|
\ .


In Figure 3, the 8 W is connected in series with the equivalent resistor, R
eq
. The current i(t) is the
current in the series resistors and v
s
(t) is the voltage across the series resistors. Applying Ohms
Law gives


( ) ( )
( )
48 cos 4 114
( ) 4 cos 4 114 A
8 8 4
s
eq
v t t
i t t
R
+
= = = +
+ +
(1)

Because the circuits in Figures 2 and 3 are equivalent, the current i(t) in Figure 2 is also given by
Equation 1. Figure 4 shows the circuit from Figure 2 redrawn with the current i(t) labeled. Also,
the current in the right-hand coil of the transformer has been labeled as i
2
(t). Since i(t) and i
2
(t)
are the currents in the coils of the ideal transformer, they are related by the equations describing
the transformer.

( ) ( ) ( )
2
2 8
cos 4 114 A
3 3
i t i t t
| |
= = +
|
\ .


Next, notice that i
2
(t) and v
o
(t), the current and voltage of the 9 W resistor in Figure 3, do not
adhere to the passive convention. Consequently, using Ohms law gives

( ) ( ) ( )
2
9 24 cos 4 114 V
o
v t i t t = = +


220


Figure 4. The circuit from Figure 1 after determining the current i(t).


Example 2:
Consider the circuit shown in Figure 5. The input to the circuit is the voltage of the voltage
source, v
s
(t). The output is the voltage across the 9 H inductor, v
o
(t). Determine the output
voltage, v
o
(t).



Figure 5 The circuit considered in Example 2.

Solution: The input voltage is a sinusoid. The output voltage is also a sinusoid and has the same
frequency as the input voltage. Apparently the circuit is at steady state. Consequently, the circuit
in Figure 5 can be represented in the frequency domain, using phasors and impedances. Figure 6
shows the frequency domain representation of the circuit from Figure 5.

In Figure 6, the impedance of the inductor is connected in series with the impedance of the 30 W
resistor. This series impedance is connected in parallel with the right hand coil of the
transformer. Replace the transformer and the series impedance with the equivalent impedance as
shown in Figure 7. The equivalent impedance is given by

2
3
(30 36) 67.5 81
2
eq
j j
| |
= + = +
|
\ .
Z

221


Figure 6 The circuit from Figure 5, represented in the frequency domain, using impedances and
phasors.



Figure 7 The circuit from Figure 6, after replacing the transformer and the impedance of the
series resistor and inductor with the equivalent impedance.


In Figure 7, the impedance of the 8 W resistor is connected in series with the equivalent
impedance, Z
eq
(w). The current I(w), is the current in this series impedance and V
s
(w) is the
voltage across the series impedance. Applying Ohms Law gives


( )
( )
75.5 26 75.5 26
( ) 0.682 21 A
8 8 67.5 81 110.73 47 j


= = = =
+ + +
s
eq
V
I
Z
(2)

Because the circuits in Figures 6 and 7 are equivalent, the current I(w) in Figure 6 is also given
by Equation 2. Figure 8 shows the circuit from Figure 6 redrawn with the current I(w) labeled.

Also, the current in the right-hand coil of the transformer has been labeled as I
2
(w). Since I(w)
and I
2
(w) are the currents in the coils of the ideal transformer, they are related by the equations
describing the transformer.

( ) ( )
3
1.023 21 A
2

| |
= =
|
\ .
2
I I

222
Notice that I
2
(w) and V
o
(w), the current and voltage of the j36 W impedance in Figure 8, do not
adhere to the passive convention. Consequently, using Ohms law gives

( ) ( ) ( )( ) ( )( ) 36 36 1.023 -21 36 90 1.023 -21 36.82 69 V j j = = = =
o 2
V I

In the time domain, the output voltage is given by

( ) ( ) 36.82 cos 4 69 V
o
v t t = +




Figure 8. The circuit from Figure 6 after determining the current I(w).

223
27. Network Functions for Simple Circuits
Introduction

Each of the circuits in this problem set is represented by a network function. Network functions
are defined, in the frequency-domain, to be quotient obtained by dividing the phasor
corresponding to the circuit output by the phasor corresponding to the circuit input. We calculate
the network function of a circuit by representing and analyzing the circuit in the frequency-
domain.

Network functions are described in Section 13.3 of Introduction to Electric Circuits by R.C.
Dorf and J.A Svoboda. Also, Table 10.7-1 summarizes the correspondence between the time-
domain and the frequency domain.

Worked Examples
Example 1:
Consider the circuit shown in Figure 1. The input to the circuit is the voltage of the voltage
source, v
i
(t). The output is the voltage across the 8 W resistor, v
o
(t). The network function that
represents this circuit is

( )
( )
( )
0.66
1
30
j

= =
+
o
i
V
H
V
(1)
Determine the value of the inductance, L.




Figure 1 The circuit considered in Example 1.

Solution: The circuit has been represented twice, by a circuit diagram and also by a network
function. The unknown inductance, L, appears in the circuit diagram, but not in the given
network function. We can analyze the circuit to determine its network function. This second
network function will depend on the unknown inductance. We will determine the value of the
inductance by equating the two network functions.
224
A network function is the ratio of the output phasor to the input phasor. Phasors exist in
the frequency domain. Consequently, our first step is to represent the circuit in the frequency
domain, using phasors and impedances. Figure 2 shows the frequency domain representation of
the circuit from Figure 1.



Figure 2 The circuit from Figure 1, represented in the frequency domain, using impedances and
phasors.

The impedances of the inductor and the two resistors are connected in series in Figure 2. V
i
(w) is
the voltage across these three series impedances and V
o
(w) is the voltage across one of the
impedances. Apply the voltage division principle to get

( ) ( )
8
4 8 j L

=
+ +
o i
V V

Divide both sides of this equation by V
i
(w) to obtain the network function of the circuit

( )
( )
( )
8
12 j L


= =
+
o
i
V
H
V
(2)

The network functions given in Equations 1 and 2 must be equal. That is


( )
( )
( )
( )
8 0.66
12
1
30
8 1 0.66 12
30
8
8 8 0.66
30
8
0.66
30
8
30 0.66
0.4 H
j L
j
j j
j j
L
L
L

=
+
+
| |
+ = +
|
\ .
+ = +
=
=
=
L
L

225

We can simply the algebra required to find L by putting the network function in Equation 2 into
the same form as the network function in Equation 1 before equating the two network functions.
Notice that the real part of the denominator of the network function is 1 in Equation 1. Lets
make the real part of the denominator be 1 in the network function given by Equation 2. Divide
the numerator and denominator by 12 to get

( )
( )
( )
8
0.66
12
12
1
12 12
j L
j

= = =
+
+
o
i
V
H
V
L
(3)

Equating the network functions given by Equations 1 and 3 gives:

0.66 0.66 1
0.4 H
12 30
1 1
12 30
L
L
L
j j

= = =
+ +


The same result is obtained with less algebra.


Example 2:
Consider the circuit shown in Figure 3. The input to the circuit is the voltage of the voltage
source, v
i
(t). The output is the voltage across the 4 W resistor, v
o
(t). This circuit is an example of
a first order low-pass filter. The network function that represents a first order low-pass filter
has the form
( )
( )
( )
1
k
j
p

= =
+
o
i
V
H
V
(4)
This network function depends on two parameters, k and p. The parameter k is called the dc
gain of the first order low-pass filter and p is the pole of the first order low-pass filter.
Determine the values of k and of p for the first order low-pass filter in Figure 3.




Figure 3 The circuit considered in Example 2.
226

Solution: We will analyze the circuit to determine its network function and then put the network
function into the form given in Equation 4. A network function is the ratio of the output phasor
to the input phasor. Phasors exist in the frequency domain. Consequently, our first step is to
represent the circuit in the frequency domain, using phasors and impedances. Figure 4 shows the
frequency domain representation of the circuit from Figure 3.



Figure 4 The circuit from Figure 3, represented in the frequency domain, using impedances and
phasors.

The impedances of the inductor and the two resistors are connected in series in Figure 4. V
i
(w) is
the voltage across these three series impedances and V
o
(w) is the voltage across one of the
impedances. Apply the voltage division principle to get

( )
( )
( )
4
9 4 0.25 j

=
+ +
o i
V V

Divide both sides of this equation by V
i
(w) to obtain the network function of the circuit

( )
( )
( ) ( )
4
13 0.25 j


= =
+
o
i
V
H
V
(5)

Next, we put the network function into the form specified by Equation 4. Notice that the real part
of the denominator is 1 in Equation 4. Divide the numerator and denominator by 13 in Equation
5 to get

( )
( )
( ) ( )
4
0.308
13
0.25
13
1
52
13 13
j j


= = =
+
+
o
i
V
H
V
(6)

Comparing the network functions given by Equations 4 and 6 gives

k = 0.308 V/V and p = 52 rad/s
227
Example 3:
Consider the circuit shown in Figure 5. The input to the circuit is the voltage of the voltage
source, v
i
(t). The output is the voltage across the 5 W resistor, v
o
(t). The network function that
represents this circuit is

( )
( )
( )
0.208
1
3
j
j

= =
+
o
i
V
H
V
(7)
Determine the value of the capacitance, C.




Figure 5 The circuit considered in Example 3.


Solution: The circuit has been represented twice, by a circuit diagram and also by a network
function. The unknown capacitance, C, appears in the circuit diagram, but not in the given
network function. We can analyze the circuit to determine its network function. This second
network function will depend on the unknown capacitance. We will determine the value of the
capacitance by equating the two network functions.
A network function is the ratio of the output phasor to the input phasor. Phasors exist in
the frequency domain. Consequently, our first step is to represent the circuit in the frequency
domain, using phasors and impedances. Figure 6 shows the frequency domain representation of
the circuit from Figure 5.




Figure 6 The circuit from Figure 5, represented in the frequency domain, using impedances and
phasors.

228
The impedances of the capacitor and the two resistors are connected in series in Figure 6. V
i
(w)
is the voltage across these three series impedances and V
o
(w) is the voltage across one of the
impedances. Apply the voltage division principle to get

( ) ( )
5
1
5 3
j C

=
+ +
o i
V V

Divide both sides of this equation by V
i
(w) to obtain the network function of the circuit

( )
( )
( )
5
1
5 3
j C

= =
+ +
o
i
V
H
V
(8)

We can simply the algebra required to find C by putting the network function in Equation 8 into
the same form as the network function in Equation 7 before equating the two network functions.
Lets multiply the numerator and denominator by to get


j C
( )
( )
( ) ( )
5
5
1
1 8
8
j C j
C
j C j C
j C

= = =
+
+
o
i
V
H
V
(9)

Equating the network functions given by Equations 7 and 9 gives:

( )
5 0.208
1 8
1
3
j j
C
j C
j

=
+
+


Comparing corresponding parts of this equation indicates that:

1
5 0.208 8
3
C C = = and

The values of C obtained from these equations must agree. (If they do not, weve made an error.)
Solving these equations gives

C = 41.60 mF and C = 41.67 mF

These values agree, but there is some uncertainty in the third significant figure. Its appropriate
to report our result with two significant figures:

C = 42 mF
229
Example 4:
Consider the circuit shown in Figure 7. The input to the circuit is the voltage of the voltage
source, v
i
(t). The output is the voltage across the 8 W resistor, v
o
(t). This circuit is an example of
a first order high-pass filter. The network function that represents a first order high-pass filter
has the form

( )
( )
( )
1
j
k
j
p

= =
+
o
i
V
H
V
(10)

The network function depends on two parameters, k and p. The parameter p is called the pole of
the first order high-pass filter. The parameter k is sometime referred to as a gain, but the high-
frequency gain of the circuit is given by the product kp. Determine the values of k and of p for
the first order high-pass filter in Figure 7.




Figure 7 The circuit considered in Example 4.

Solution: We will analyze the circuit to determine its network function and then put the network
function into the form given in Equation 10. A network function is the ratio of the output phasor
to the input phasor. Phasors exist in the frequency domain. Consequently, our first step is to
represent the circuit in the frequency domain, using phasors and impedances. Figure 8 shows the
frequency domain representation of the circuit from Figure 7.



Figure 8 The circuit from Figure 7, represented in the frequency domain, using impedances and
phasors.

The impedances of the inductor and one 8 W resistor are connected in parallel in Figure 8. The
equivalent impedance is
230
( )
( ) ( )
( ) ( )
8 0.8
8 0.8
j
j

=
+
e
Z

The parallel impedance is connected in series with the other 8 W resistor. V
i
(w) is the voltage
across the series impedances and V
o
(w) is the voltage across the equivalent impedance, Z
e
(w).
Apply the voltage division principle to get


( )
( ) ( )
( ) ( )
( ) ( )
( ) ( )
( )
( ) ( )
( ) ( ) ( ) ( ) ( )
( )
( )
( )( )( )
( )
( )
( )
( )
8 0.8
8 0.8
8 0.8
8
8 0.8
8 0.8
8 8 0.8 8 0.8
6.4
64 8 2 0.8
6.4
64 12.8
j
j
j
j
j
j j
j
j
j
j

+
=
+
+
=
+ +
=
+
=
+
o i
i
i
i
V V
V
V
V


Divide both sides of this equation by V
i
(w) to obtain the network function of the circuit

( )
( )
( )
( )
( )
6.4
64 12.8
j
j


= =
+
o
i
V
H
V
(11)

Next, we put the network function into the form specified by Equation 10. Notice that the real
part of the denominator is 1 in Equation 10. Divide the numerator and denominator by 64 in
Equation 11 to get

( )
( )
( )
( )
( ) ( )
6.4
64
0.1
64 12.8 12.8
1
64 64
j
j
j j


= = =
+
+
o
i
V
H
V
(12)

Comparing the network functions given by Equations 10 and 12 gives

k = 0.1 V/V and
64
5 rad/s
12.8
p = = .

231
Example 5:
Consider the circuit shown in Figure 9. The input to the circuit is the voltage of the voltage
source, v
i
(t). The output is the voltage, v
o
(t), across the series connection of the capacitor and 16
kW resistor. The network function that represents a this circuit has the form
( )
( )
( )
1
1
j
z
j
p

+
= =
+
o
i
V
H
V
(13)

The network function depends on two parameters, z and p. The parameter z is called the zero of
the circuit and the parameter p is called the pole of the circuit. Determine the values of z and of p
for the circuit in Figure 9.



Figure 9 The circuit considered in Example 5.

Solution: We will analyze the circuit to determine its network function and then put the network
function into the form given in Equation 13. A network function is the ratio of the output phasor
to the input phasor. Phasors exist in the frequency domain. Consequently, our first step is to
represent the circuit in the frequency domain, using phasors and impedances. Figure 10 shows
the frequency domain representation of the circuit from Figure 9.


Figure 10 The circuit from Figure 9, represented in the frequency domain, using impedances and
phasors.

232
The impedances of the capacitor and the 16 kW resistor are connected in series in Figure 10. The
equivalent impedance is
( )
( )
6
10
16000
0.23 j

= +
e
Z

The equivalent impedance is connected in series with the 8 kW resistor. V
i
(w) is the voltage
across the series impedances and V
o
(w) is the voltage across the equivalent impedance, Z
e
(w).
Apply the voltage division principle to get


( )
( )
( )
( )
( ) ( )
( ) ( )
( )
( )
( )
( )
( )
( )
( )
6
6
6
6
6
6
6
6
10
16000
0.23
10
8000 16000
0.23
10 0.23 16000
10 0.23 24000
10 3680
10
10 5520
10
1 0.00368
1 0.00552
j
j
j
j
j
j
j
j

+
=
+ +
+
=
+
+
=
+
+
=
+
o i
i
i
i
V V
V
V
V


Divide both sides of this equation by V
i
(w) to obtain the network function of the circuit

( )
( )
( )
( )
( )
1 0.00368
1 0.00552
j
j


+
= =
+
o
i
V
H
V
(14)

Equating the network functions given by Equations 13 and 14 gives


( )
( )
1
1 0.00368
1 0.00552
1
j
j
z
j
j
p

+
+
=
+
+


Comparing these network functions gives

1
271.74 rad/s
0.00368
z = = and
1
181.16 rad/s
0.00552
p = = .

233
Example 6:
Consider the circuit shown in Figure 11. The input to the circuit is the voltage of the voltage
source, v
i
(t). The output is the voltage, v
o
(t), across series connection of the inductor the 2 W
resistor. The network function that represents this circuit is

( )
( )
( )
1
5
0.2
1
25
j
j

+
= =
+
o
i
V
H
V
(15)
Determine the value of the inductance, L.



Figure 11 The circuit considered in Example 6.

Solution: The circuit has been represented twice, by a circuit diagram and also by a network
function. The unknown inductance, L, appears in the circuit diagram, but not in the given
network function. We can analyze the circuit to determine its network function. This second
network function will depend on the unknown inductance. We will determine the value of the
inductance by equating the two network functions.
A network function is the ratio of the output phasor to the input phasor. Phasors exist in
the frequency domain. Consequently, our first step is to represent the circuit in the frequency
domain, using phasors and impedances. Figure 12 shows the frequency domain representation of
the circuit from Figure 11.



Figure 12 The circuit from Figure 11, represented in the frequency domain, using impedances
and phasors.
234

The impedances of the inductor and the 2 W resistor are connected in series in Figure 12. The
equivalent impedance is


The equivalent impedance is connected in series with the 8 W resistor. V
i
(w) is the voltage
across the series impedances and V
o
(w) is the voltage across the equivalent impedance, Z
e
(w).
Apply the voltage division principle to get


( ) 2 j L = +
e
Z
( ) ( ) ( )
2 2
8 2 10
j L j L
j L j L



+ +
= =
+ + +
o i i
V V V

Divide both sides of this equation by V
i
(w) to obtain the network function of the circuit

( )
( )
( )
2
10
j L
j L


+
= =
+
o
i
V
H
V


Next, we put the network function into the form specified by Equation 15. Factoring 2 out of
both terms in the numerator and also factoring 10 out of both terms in the denominator we get

( )
2 1
1
2
2
0.2
1 10 1
10 10
L
L
j
j
L L
j j


| |
+
+ |
\ .
= =
| |
+ +
|
\ .
H (16)

Equating the network functions given by Equations 15 and 16 gives


1 1
5 2
0.2 0.2
1 1
10 25
L
j j
L
j j

+ +
=
+ +


Comparing these network functions gives

1
2 5
L
= and
1
10 25
L
=

The values of L obtained from these equations must agree, and they do. (If they do not, weve
made an error.) Solving each of these equations gives L = 0.4 H.


235
Example 7:
Consider the circuit shown in Figure 13. The input to the circuit is the voltage of the voltage
source, v
i
(t). The output is the voltage, v
o
(t), across series connection of the capacitor the 4 kW
resistor. The network function that represents this circuit is
( )
( )
( )
1
69.6
1
55.7
j
j

+
= =
+
o
i
V
H
V
(17)
Determine the value of the capacitance, C.



Figure 13 The circuit considered in Example 7.

Solution: The circuit has been represented twice, by a circuit diagram and also by a network
function. The unknown capacitance, C, appears in the circuit diagram, but not in the given
network function. We can analyze the circuit to determine its network function. This second
network function will depend on the unknown capacitance. We will determine the value of the
capacitance by equating the two network functions.



Figure 14 The circuit from Figure 13, represented in the frequency domain, using impedances
and phasors.

A network function is the ratio of the output phasor to the input phasor. Phasors exist in the
frequency domain. Consequently, our first step is to represent the circuit in the frequency
236
domain, using phasors and impedances. Figure 14 shows the frequency domain representation of
the circuit from Figure 13.

The impedances of the capacitor and the 4 kW resistor are connected in series in Figure 14. The
equivalent impedance is
( )
1
4000
j C

= +
e
Z

The equivalent impedance is connected in series with the 1 kW resistor. V
i
(w) is the voltage
across the series impedances and V
o
(w) is the voltage across the equivalent impedance, Z
e
(w).
Apply the voltage division principle to get

( ) ( )
( )
( )
( )
1
4000
1 4000
1
1 5000
1000 4000
j C
j C
j C
j C

+
+
= =
+
+ +
o i i
V V V

Divide both sides of this equation by V
i
(w) to obtain the network function of the circuit

( )
( )
( )
( )
( )
1 4000
1 5000
j C
j C


+
= =
+
o
i
V
H
V
(18)

Equating the network functions given by Equations 17 and 18 gives


( )
( )
1
1 4000
69.6
1 5000
1
55.7
j
j C
j C
j

+
+
=
+
+


Comparing these network functions gives

1
4000
69.9
C = and
1
5000
55.7
C =

The values of C obtained from these equations must agree, and they do. (If they do not, weve
made an error.) Solving these equations gives

C = 3.577 F and C = 3.591 F

These values agree, but there is some uncertainty in the third significant figure. Its appropriate
to report our result with two significant figures:

C = 3.6 F
237
28. Network Functions of Circuits Containing Dependents Sources
Introduction
Each of the circuits in this problem set is represented by a network function. Network functions
are defined, in the frequency-domain, to be quotient obtained by dividing the phasor
corresponding to the circuit output by the phasor corresponding to the circuit input. We calculate
the network function of a circuit by representing and analyzing the circuit in the frequency-
domain.
Network functions are described in Section 13.3 of Introduction to Electric Circuits by R.C.
Dorf and J.A Svoboda. Also, Table 10.7-1 summarizes the correspondence between the time-
domain and the frequency domain.
Worked Examples





Example 1:
Consider the circuit shown in Figure 1. The input to the circuit is the voltage of the voltage
source, v
i
(t). The output is the voltage across the capacitor, v
o
(t). The network function that
represents this circuit is

( )
( )
( )
3
1 1
2 5
j j


= =
| | | |
+ +
| |
\ . \ .
o
i
V
H
V
(1)

Determine the value of the inductance, L, and of the gain, A, of the Voltage Controlled Voltage
Source (VCVS).




Figure 1 The circuit considered in Example 1.


238
Solution: The circuit has been represented twice, by a circuit diagram and also by the given
network function. The unknown parameters, L and A, appear in the circuit diagram, but not in the
given network function. We can analyze the circuit to determine its network function. This
version of the network function will depend on the unknown parameters. We will determine the
value of these parameters by equating the two version of the network function.
A network function is the ratio of the output phasor to the input phasor. Phasors exist in
the frequency domain. Consequently, our first step is to represent the circuit in the frequency
domain, using phasors and impedances. Figure 2 shows the frequency domain representation of
the circuit from Figure 1.




Figure 2 The circuit from Figure 1, represented in the frequency domain, using impedances and
phasors.


=
The circuit in Figure 2 consists of two meshes. The mesh current of the left-hand mesh is labeled
as I
1
(w) and the mesh current of the right-hand mesh is labeled as I
2
(w). Apply Kirchhoffs
Voltage Law (KVL) to the left-hand mesh to get

( ) ( ) ( ) 4 0 j L +
1 1 i
I I V
Solve for I
1
(w) to get
( )
( )
( )
0.25
4
1
4
L
j L
j

= =
+
+
1
i
i
V
I V
Next use Ohms Law to obtain represent V
a
(w) as

( ) ( ) ( )
1
4
1
4
L
j

= =
+
1 a i
V I V

(2)
Apply Kirchhoffs Voltage Law (KVL) to the right-hand mesh to get
( ) ( ) ( )
20
4 0 A
j

+
2 2 a
I I V =
Solve for I
2
(w) to get
239
( ) ( ) ( ) ( )
20
20
4 20
4 1
5
j
A
A j A
j
j
j

= = =
+
+ +
2 a a
I V V
a
V

The output voltage is obtained by multiplying the mesh current I
2
(w) by the impedance of the
capacitor
( ) ( ) ( ) ( )
20 20
20
1 1
5 5
j
A
A
j j
j j




= = =
+ +
2 o a
V I V V
a
(3)


Substituting the expression for V
a
(w) from Equation 2 into Equation 3 gives
( ) ( ) ( )
1
1 1 1 1
4 5 4 5
A A
L L
j j j j



= =
| | | |
+ + + +
| |
\ . \ .
o i i
V V V


Divide both sides of this equation by V
i
(w) to obtain the network function of the circuit

( )
( )
( )
1 1
4 5
A
L
j j

= =
| | |
+ +
|
\ . \
o
i
V
H
V |
|
.
Comparing the network functions given by Equations 1 and 4 gives A = 3 V/V and L = 2 H.
(4)



240
Example 2:
Consider the circuit shown in Figure 3. The input to the circuit is the voltage of the voltage
source, v
i
(t). The output is the voltage across the capacitor, v
o
(t). This circuit is an example of a
second order low-pass filter. The network function that represents a second order low-pass
filter has the form

( )
( )
( )
1 2
1 1
k
j j
p p


= =
| | |
+ +
|
\ . \
o
i
V
H
V |
|
.



(5)
This network function depends on three parameters, k, p
1
and p
2
. The parameter k is called the
dc gain of the second order low-pass filter. Both p
1
and p
2
are poles of the second order low-
pass filter. Determine the values of k, p
1
and p
2
for the second order low-pass filter in Figure 3.


Figure 3 The circuit considered in Example 2.

Solution: We will analyze the circuit to determine its network function and then put the network
function into the form given in Equation 4. A network function is the ratio of the output phasor
to the input phasor. Phasors exist in the frequency domain. Consequently, our first step is to
represent the circuit in the frequency domain, using phasors and impedances. Figure 4 shows the
frequency domain representation of the circuit from Figure 3.





Figure 4 The circuit from Figure 3, represented in the frequency domain, using impedances and
phasors.
241
The circuit in Figure 4 consists of two meshes. The mesh current of the left-hand mesh is labeled
as I
1
(w) and the mesh current of the right-hand mesh is labeled as I
2
(w). Apply Kirchhoffs
Voltage Law (KVL) to the left-hand mesh to get

( ) ( ) ( ) ( ) 0.66 4 0 j +
1 1 i
I I V =
Solve for I w) to get

1
(
( )
( )
( ) ( )
( )
0.25
0.66 4 1 0.165 j j



= =
+ +
1
i
i
V
I V

Next use Ohms Law to obtain represent V
a
(w) as

( ) ( )
( )
( )
1
4
1 0.165 j

= =
+
1 a i
V I V

(6)
Apply Kirchhoffs Voltage Law (KVL) to the right-hand mesh to get

( )
( )
( ) ( )
1000
6 1
12.82 j

+
2 2 a
I I V 5 0 =
Solve for I
2
(w) to get

( )
( )
( )
( )
( )
( )
12.82 15
15
1000
12.82 6 1000
6
12.82
j
j
j

= =
+
+
2 a a
I V V


The output voltage is obtained by multiplying the mesh current I
2
(w) by the impedance of the
capacitor


( )
( )
( )
( )
( )
( )
( )
( )
( )
( )
12.82 15
1000 1000
12.82 12.82 12.82 6 1000
15
12.82 6
1
1000
15
1
13
j
j j j
j
j

= =
+
=
+
=
+
2 o a
a
a
V I
V
V
V

Substituting the expression for V w) from Equation 6 into Equation 7 gives

(7)
a
(
( )
( )
( )
15 1
1 0.165
1
13
j
j


=
+
+
i o
V V
242

Divide both sides of this equation by V
i
(w) to obtain the network function of the circuit

( )
( )
( )
( ) ( )
15
1 1 0.165
13
j
j

= =
| |
+ +
|
\ .
o
i
V
H
V
(8)

Comparing the network functions given by Equations 4 and 8 gives

k = 15 V/V, p p



Example 3:

1
= 13 rad/s and
2
= 6.06 rad/s
(Or perhaps p
1
= 6.06 rad/s and p
2
= 13 rad/s. One pole is named p
1
and the other pole is named
p
2
.)

Consider the circuit shown in Figure 5. The input to the circuit is the voltage of the voltage
source, v
i
(t). The output is the voltage across the capacitor, v
o
(t). The network function that
represents this circuit is

( )
( )
( )
5
1
10
j j

= =
| |
+
|
\ .
o
i
V
H
V
(9)
Determine the value of the inductance, L, and of the gain, A, of the Current Controlled Current
Source (CCCS).




Figure 5 The circuit considered in Example 3.

Solution: The circuit has been represented twice, by a circuit diagram and also by the given
network function. The unknown parameters, L and A, appear in the circuit diagram, but not in the
given network function. We can analyze the circuit to determine its network function. This
243
version of the network function will depend on the unknown parameters. We will determine the
value of these parameters by equating the two version of the network function.
A network function is the ratio of the output phasor to the input phasor. Phasors exist in
the frequency domain. Consequently, our first step is to represent the circuit in the frequency
domain, using phasors and impedances. Figure 6 shows the frequency domain representation of
the circuit from Figure 5.





The circuit in Figure 6 consists of two meshes. The mesh current of the left-hand mesh is the
same current the controlling current of the CCCS, I
a
(w). Apply Kirchhoffs Voltage Law (KVL)
to the left-hand mesh to get
=
Solve for I w) to get

Figure 6 The circuit from Figure 5, represented in the frequency domain, using impedances and
phasors.

( ) ( ) ( ) 20 0 j L +
a a i
I I V
a
(
( )
( )
( )
0.05
20
1
20
L
j L
j

= =
+
+
i
a i
V
I V (10)

The mesh current of the right-hand mesh is the same current the controlled current of the CCCS,
A I
a
(w). The output voltage is obtained by multiplying this mesh current by the impedance of the
capacitor


( ) ( )
1
A
j

=
o a
V I (11)


Substituting the expression for I
a
(w) from Equation 10 into Equation 11 gives

( ) ( ) ( )
1 0.05 0.05
1 1
20 20
A
A
L L j
j j j


= =
| |
+ +
|
\ .
o i i
V V V

244
Divide both sides of this equation by V
i
(w) to obtain the network function of the circuit

( )
( )
( )
0.05
1
20
A
L
j j


= =
| |
+
|
\ .
o
i
V
H
V
(12)

Comparing the network functions given by Equations 9 and 12 gives A = 100 V/V and L = 2 H.

Consider the circuit shown in Figure 7. The input to the circuit is the voltage of the voltage
source, v
i
(t). The output is the voltage across the capacitor, v t). The network function that
represents this circuit is
Example 4:
o
(

( )
( )
( )
9
1
18
j j

= =
| |
+
|
\ .
o
i
V
H
V
(13)

Determine the value of the capacitance, C, and of the gain, A, of the Current Controlled Voltage
Source (CCVS).





Figure 7 The circuit considered in Example 4.

A network function is the ratio of the output phasor to the input phasor. Phasors exist in
the frequency domain. Consequently, our first step is to represent the circuit in the frequency
domain, using phasors and impedances. Figure 8 shows the frequency domain representation of
the circuit from Figure 7.

Solution: The circuit has been represented twice, by a circuit diagram and also by the given
network function. The unknown parameters, C and A, appear in the circuit diagram, but not in
the given network function. We can analyze the circuit to determine its network function. This
version of the network function will depend on the unknown parameters. We will determine the
value of these parameters by equating the two version of the network function.

245




Figure 8 The circuit from Figure 7, represented in the frequency domain, using impedances and
phasors.
The circuit in Figure 6 consists of two meshes. The mesh current of the left-hand mesh is the
same current the controlling current of the CCVS, I
a
(w). Apply Kirchhoffs Voltage Law (KVL)
to the left-hand mesh to get

( ) ( ) 0 j =
a i
I V
Solve for I
a
(w) to get
( ) ( )
1
j

=
a i
I V


(14)
Next consider the right-hand mesh. Use the voltage division principle to get
( ) ( )
( )
( )
1
1
1 4
4
A j C
A
j C
j C

= =
+
+
o a
V I
a
I


(15)
Substituting the expression for I
a
(w) from Equation 14 into Equation 15 gives
( )
( )
( )
( ) ( )
( )
1
1 4 1 4
A A
j j C j j C


= =
+ +
o i i
V V V



Divide both sides of this equation by V
i
(w) to obtain the network function of the circuit

( )
( )
( ) ( ) ( )
1 4
A
j j C


= =
+
o
i
V
H
V
(16)

Comparing the network functions given by Equations 13 and 16 gives A = 9 V/V and
1
13.89 mF
18 (4)
C = = .
246
29. Complete Response of Switched Circuits
Introduction
Each of the circuits in this problem set contains a switch. In each problem, the circuit is initially
at steady state. The switch that opens or closes at time t = 0, changing the circuit and disturbing
currents and voltages of the circuit elements. Eventually, the disturbance dies out and the circuit
comes to a new steady state.

Consider the circuit shown in Figure 1. The input to the circuit is the voltage of the voltage
source, 24 V. The output of this circuit, the voltage across the capacitor, is given to be
(1)




Circuit analysis using the Laplace transform determines the response of these circuits to the
disturbance of opening or closing the switch.
Circuit analysis using the Laplace transform is described in Section 14.8 of Introduction to
Electric Circuits by R.C. Dorf and J.A Svoboda. In particular, Table 14.8-1 summarizes the
Laplace-transform models of circuit elements. The Laplace transform itself is described in
Section 14.3. Switches are described in Section 2.10.

Worked Examples
Example 1

( )
o
0.6
16 12 V when 0
t
v t e t

= >

Determine the value of the capacitance, C.



Figure 1 The circuit considered in Example 1.

247
Solution: Before the switch closes, the circuit will be at steady state. Because the only input to
this circuit is the constant voltage of the voltage source, all of the element currents and voltages,
including the capacitor voltage, will have constant values. Closing the switch disturbs the circuit
by shorting out the 18 resistor. Eventually the disturbance dies out and the circuit is again at
steady state. All the element currents and voltages will again have constant values, but probably
different constant values than they had before the switch closed.




During the disturbance the element voltages and currents are not constant. For example,
Equation 1 describes the capacitor voltage after the switch closes. Notice that there are two parts
to the capacitor voltage. One part, 12 , dies out as the value of t increases. That part is called
the transient part of the response, or just the transient response. The other part, 12, does not
die out and is the steady state response. The sum of the transient response and the steady state
response is called the complete response. The output voltage described by Equation 1 is the
complete response of this circuit.
0.6 t
e

Figure 2 shows a plot of the capacitor voltage given by equation 1. Notice that the capacitor
voltage is continuous. This is expected because, in the absence of unbounded currents, the
voltage of a capacitor must be continuous. In particular, the value of the capacitor voltage
immediately after the switch is closed is equal to the value immediately before the switch is
closed. From Figure 2, we see that at time t = 0, when the switch closes, the value of the
capacitor voltage is v
o
(0) = 4 V.



Figure 2 The capacitor voltage, v t), from the circuit shown in Figure 1.


How does the value of the capacitance C affect the capacitor voltage? To answer this question
we must analyze the circuit. Because we want to determine the complete response, we will
analyze the circuit using Laplace transforms. Figure 3 shows the frequency-domain
representation of the circuit. The closed switch is represented by a short circuit. That short circuit
o
(
248
is connected in parallel with the 18 resistor. A short circuit in parallel with a resistor is
equivalent to a short circuit so the closed switch and 18 resistor have been replaced by a single
short circuit. The frequency-domain model of the capacitor consists of two parts, an impedance
and a voltage source. The voltage of the voltage source depends on the initial condition of the
capacitor, i.e. v ) = 4 V.

o
(0



Figure 3 The circuit represented in the frequency domain, using the Laplace transform.

We can analyze the circuit in Figure 3 by writing and solving two mesh equations. Apply KVL
to the left mesh to get


( ) ( ) ( ) ( )
1 2 1
24
4 2 I s I s I s
s
+ 0 =
Solving for I
1
(s) gives

( ) ( )
1 2
2 4
3
I s I s
s
= + (2)

Apply KVL to the right mesh to get

( ) ( ) ( ) ( )
2 1 2
1 4
4 0 I s I s I s
Cs s
+ =

Collecting the terms involving I
2
(s) gives

( ) ( )
2 1
1 4
4 4 I s I
Cs s
| |
+ = +
|
\ .
s

Substituting the expression for I
1
(s) from Equation 2 gives

( ) ( ) ( )
2 2
1 4 2 4 12 8
4 4
3 3
2
I s I s
Cs s s s
| | | |
+ = + + = +
| |
\ . \ .
I s

Collecting the terms involving I
2
(s) gives
249

( )
2
1 4 12
3
I s
Cs s
| |
+ =
|
\ .


Multiply both sides of this equation by
3
4
s to get

( )
2
3
9
4
s I s
C
| |
+ =
|
\ .

Solving for I
2
(s) gives
( )
2
9
3
4
I s
s
C
=
+
(3)
Referring to Figure 3, we see that the capacitor voltage is related to the mesh current of the right
mesh by
( ) ( )
o 2
1 4
V s I s
Cs s
= +

Substituting the expression for I
2
(s) from Equation 3 gives

( )
o
9
1 9 4
3
3
4
4
C
V s
Cs s s
s
s s
C
C
| |
= + =
|
| |
\ .
+
+
|
\ .

4
+
Performing partial fraction expansion gives

( )
o
12 12 4 16 12
3 3
4 4
V s
s s s
s s
C C
= + =
+ +
(4)

Recall that v
o
(t) is given in Equation 1. Taking the Lapalce transform of v
o
(t) gives

( ) ( ) ( )
o o
16 12
0.6
16 12
0.6
t
V s v t e u t
s s
( | |
= = = (
|
(
+ \ .
L L (5)

Comparing equations 4 and 5 shows that


3
0.6 1.25 F
4
C
C
= =


250
Example 2:
Consider the circuit shown in Figure 4. The input to the circuit is the voltage of the voltage
source, 24 V. The output of this circuit, the voltage across the 6 resistor, is given to

(6)

Determine the value of the inductance, L and of the resistances R
1
and R
2
.

( )
o
0.35
12 6 V when 0
t
v t e t

= >


Figure 4 The circuit considered in Example 2.


Solution: Before the switch closes, the circuit will be at steady state. Because the only input to
this circuit is the constant voltage of the voltage source, all of the element currents and voltages,
including the inductor current, will have constant values. Closing the switch disturbs the circuit
by shorting out the resistor R
1
. Eventually the disturbance dies out and the circuit is again at
steady state. All the element currents and voltages will again have constant values, but probably
different constant values than they had before the switch closed.

Equation 6 describes the output voltage after the switch closes. Notice that there are two parts to
this voltage. One part, , dies out as the value of t increases. That part is called the
transient part of the response, or just the transient response. The other part, 12, does not die
out and is the steady state response. The sum of the transient response and the steady state
response is called the complete response. The output voltage described by Equation 6 is the
complete response of this circuit.

How do the values of the circuit parameters L, R
1
and R
2
affect the output voltage? To answer
this question we must analyze the circuit. Because we want to determine the complete response,
we will analyze using Laplace transforms. The frequency-domain model of the inductor consists
of two parts, an impedance and a voltage or current source. The value of the voltage source
voltage or current source current depends on the initial condition of the inductor, that is, the
0.35
6
t
e

251
inductor current at time t = 0. We need to find the initial inductor current before we can represent
the circuit using Laplace transforms.

Referring to Figure 4, we see that the inductor current is equal to the current in the 6 resistor.
Consequently,


0.35
( ) 12 6
0.35
( ) 2 A when 0
6 6
t
v t e
t
i t e t


= = = > (7)

In the absence of unbounded voltages, the current in any inductor is continuous. Consequently,
the value of the inductor current immediately before t = 0 is equal to the value immediately after
t = 0. To find the initial inductor current we set t = 0 in Equation 7 to get i(0) = 1 A.

Figure 5 shows the frequency domain representation of the circuit. We selected the model of the
inductor that uses a voltage source to account for the initial condition in anticipation of writing a
mesh equation. The voltage of this voltage source is



In Figure 5, the closed switch is represented by a short circuit. That short circuit is connected in
parallel with resistor R
1
. A short circuit in parallel with a resistor is equivalent to a short circuit
so the closed switch and R
1
have been replaced by a single short circuit.

To analyze the circuit in Figure 5, we write and solve a single mesh equation. Apply KVL to the
mesh to get

( ) ( )( ) 0 1 Li L L = =
( ) ( )
2
24
6 R Ls I s L
s
+ + = +



Figure 5 The circuit represented in the frequency domain, using the Laplace transform.

Solving for I(s) gives
( )
2 2
24 24
6 6
L s
s L
I s
R Ls R
s s
L
+ +
= =
+ + + | |
+
|
\ .

252

Using Ohms law gives
( ) ( )
( )( )
o
2
6 24
6
6
6
s
L
V s I s
R
s s
L
+
= =
+ | |
+
|
\ .


Partial fraction expansion gives
( )
( )( ) ( )
2
2 2
o
2
6 18
6 24
6 6
6
R
R R
V s
R
s
s
L

+ +
=
+
+
(8)

Recall that v
o
(t) is given in Equation 6. Taking the Lapalce transform of v
o
(t) gives

( ) ( ) ( )
o o
12 6
0.35
12 6
0.35
t
V s v t e u t
s s
( | |
= = = (
|
(
+ \ .
L L (9)

Comparing equations 8 and 9 shows that


( )( )
2
2
6 24
12 6
6
R
R
= =
+

and

2
6
12 12
0.35 34.29 H
0.35
R
L
L L
+
= = = =

How can we find R
1
? Resistor R
1
is removed from the circuit by closing the switch but R
1
was
part of the circuit before the switch closed. The initial inductor current depends on the value of
the resistance R
1
. The only input to the circuit in Figure 4 is a constant, 24 V. Consequently,
when the circuit is at steady state, the inductor will act like a short circuit. Figure 6 shows the
steady state circuit when the switch is open. The open switch is modeled as an open circuit. The
inductor is modeled as short circuit. Writing and solving a mesh equation gives

( )
1
24
6 6
i t
R
=
+ +

Letting t = 0 gives
( )
1
1
24
0 1 12
6 6
i R
R
= = =
+ +


253


Figure 4 The circuit before the switch closes


Example 3:
Consider the circuit shown in Figure 7. The input to the circuit is the voltage of the voltage
source, 18 V. The output of this circuit is the voltage across the capacitor, v
o
(t). Determine the
value of the capacitor voltage 1 second after the switch opens.



Figure 7 The circuit considered in Example 1.


Solution: Before the switch opens, the circuit will be at steady state. Because the only input to
this circuit is the constant voltage of the voltage source, all of the element currents and voltages,
including the capacitor voltage, will have constant values. Opening the switch disturbs the
circuit. Eventually the disturbance dies out and the circuit is again at steady state. All the element
currents and voltages will again have constant values, but probably different constant values than
they had before the switch closed.

During the disturbance the element voltages and currents are not constant. We can find these
voltages and currents by analyzing the circuit using Laplace Transforms. The Laplace Transform
model of the capacitor consists of two parts, an impedance and either a series voltage or a
parallel current source. The voltage of the voltage source or the current of the current source
254
depend on the initial condition of the capacitor, i.e. the capacitor voltage at time t = 0. Before we
can model the capacitor using Laplace transforms, we must determine v
o
(0).

Figure 8 shows the circuit before t = 0, when the switch is closed and the circuit is at steady state.
The closed switch is modeled as a short circuit. The parallel combination of resistor and a short
circuit is equivalent to a short circuit. Consequently, a short circuit replaces the switch and the 30
resistor. A capacitor in a steady-state dc circuit acts like an open circuit, so an open circuit
replaces the capacitor. The voltage across that open circuit is the capacitor voltage, v
o
(t).

Because the circuit in Figure 8 is at steady state, the value of the capacitor voltage will be
constant. This constant is the value of the capacitor voltage just before the switch opens. In the
absence of unbounded currents, the voltage of a capacitor must be continuous. The value of the
capacitor voltage immediately after the switch opens is equal to the value immediately before the
switch opens. This value is the initial condition of the capacitor and has been labeled as v
o
(0) in
Figure 8. Using the voltage division principle, the initial condition is calculated to be

( ) ( )
4
0 18 1
4 2
o
v = =
+
2 V






Figure 8 The steady-state circuit before t = 0





Figure 9 The circuit represented in the frequency domain, using the Laplace transform.

255
Figure 9 shows the Laplace transform representation of the circuit. The Laplace transform model
of the capacitor consists of two parts, an impedance and a voltage source. The voltage of the
voltage source is given by


( ) 0
12
o
v
s s
=

We can analyze the circuit in Figure 9 by writing and solving two mesh equations. Apply KVL
to the left mesh to get

( ) ( ) ( ) ( ) ( )
1 1 2 1
18
30 4 2 0 I s I s I s I s
s
+ + =

Collecting the terms involving I
1
(s) gives

( ) ( )
1 2
18
36 4 I s I s
s
= (10)
Apply KVL to the right mesh to get

( ) ( ) ( ) ( )
2 1 2
2 12
4 0 I s I s I s
s s
+ =
Solving for I
1
(s) gives

( ) ( )
1 2
0.5 3
1 I s I s
s s
| |
= + +
|
\ .
(11)

Substituting I
1
(s) from Equation 11 into Equation 10 gives

( ) ( )
2 2
0.5 3 18
36 1 4 I s I s
s s
| | | |
+ + =
| |
\ . \ .

s

Collecting the terms involving I
2
(s) gives

( )
2
18 90
32 I s
s s
| |
+ =
|
\ .

Solving for I
2
(s) gives
( )
2
90
32
9
16
I s
s

=
+


Referring to Figure 9, we see that the capacitor voltage is related to the mesh current of the right
mesh by
256
( ) ( )
2
2 1
o
V s I s
s s
= +
2

Substituting the expression for I s) from Equation 3 gives

2
(
( )
90 45
2 12
32 8
9 9
16 16
o
V s
s s
s s s
| |

|
= + =
|
| |
|
+ + |
|
\ .
\ .
12
s
+




Partial fraction expansion gives

( )
10 10 12 2 10
9 9
16 16
o
V s
s s s
s s

= + + = +
+ +


Taking the inverse Laplace transform gives

( )
9
16
2 10 for 0
o
t
v t e t

= + >

The value of the capacitor voltage 1 second after the switch opens is

( )
9
(1)
16
1 2 10 7.698 V
o
v e

= + =


257
Example 4:
Consider the circuit shown in Figure 10. The input to the circuit is the voltage of the voltage
source, 20 V. The output of this circuit is the voltage across the capacitor, v
o
(t). Determine the
value of the capacitor voltage 2 seconds after the switch closes.



Figure 10 The circuit considered in Example 4.


Solution: Before the switch closes, the circuit will be at steady state. Because the only input to
this circuit is the constant voltage of the voltage source, all of the element currents and voltages,
including the capacitor voltage, will have constant values. Closing the switch disturbs the circuit
by shorting out the 40 resistor. Eventually the disturbance dies out and the circuit is again at
steady state. All the element currents and voltages will again have constant values, but probably
different constant values than they had before the switch closed.



Because the circuit in Figure 11 is at steady state, the value of the capacitor voltage will be
constant. This constant is the value of the capacitor voltage just before the switch closes. In the
absence of unbounded currents, the voltage of a capacitor must be continuous. The value of the
capacitor voltage immediately after the switch closes is equal to the value immediately before the
During the disturbance the element voltages and currents are not constant. We can find these
voltages and currents by analyzing the circuit using Laplace transforms. The Laplace transform
model of the capacitor consists of two parts, an impedance and either a series voltage source or a
parallel current source. The voltage of the voltage source or the current of the current source
depend on the initial condition of the capacitor, i.e. the capacitor voltage at time t = 0. Before we
can model the capacitor using Laplace transforms, we must determine v
o
(0).
Figure 11 shows the circuit before t = 0, when the switch is open and the circuit is at steady state.
The open switch is modeled as an open circuit. A capacitor in a steady-state dc circuit acts like
an open circuit, so an open circuit replaces the capacitor. The voltage across that open circuit is
the capacitor voltage, v
o
(t).
258
switch closes. This value is the initial condition of the capacitor and has been labeled as v
o
(0) in
Figure 11. The circuit in Figure 11 is analyzed be writing mesh equations:

and
2



( )
1 1 2
40 10 20 0 i i i + =
( ) ( )
2 1
10 0 10 0
o
i v i i + =



Figure 11 The steady-state circuit before t = 0




The current in the right mesh, i
2
, is zero because it is equal to the element the current of an open
circuit. Letting i
2
= 0 in the mesh equations gives

1 1 1
20
40 10 20 0 0.4 A
50
i i i + = = =
and
( ) ( )
1 1
0 10 0 0 10 4 V
o o
v i v i = = =





Figure 12 The circuit represented in the frequency domain, using the Laplace transform.

259
Figure 12 shows the Laplace transform representation of the circuit. We can analyze the circuit
in Figure 3 by writing and solving two mesh equations. Apply KVL to the left mesh to get

( ) ( ) ( )
1 2
20
10 0 I s I s
s
=
Solving for I
1
(s) gives
( ) ( )
1 2
2
I s I s
s
= + (12)
Apply KVL to the right mesh to get

( ) ( ) ( ) ( ) ( )
2 2 1 2
4 4
10 10 0 I s I s I s I s
s s
+ + =

Collecting the terms involving I
2
(s) gives

( ) ( )
2 1
4
20 10 0 I s I s
s s
| |
+ =
|
\ .
4
=


(13)
Substituting I
1
(s) from Equation 12 into Equation 13 gives
( ) ( )
2 2
4 2
20 10 0 I s I s
s s
| | | |
+ = +
| |
\ . \ .
4
s
=




Collecting the terms involving I
2
(s) gives
( )
2
4 20 4
10 I s
s s s
| |
+ =
|
\ .

16
s
=

Multiplying both sides of this equation by
10
s
gives

( )
2
4 1
10 10
s I s
| |
+ =
|
\ .
6

Solving for I s) gives


2
(
( )
2
1.6
0.4
I s
s
=
+
(14)
Referring to Figure 12, we see that the capacitor voltage is related to the mesh current of the
right mesh by

( ) ( )
2
4 4
o
V s I s
s s
= +

260
Substituting the expression for I
2
(s) from Equation 14 gives

( )
( )
4 1.6 4 6.4 4
0.4 0.4
o
V s
s s s s s s
| |
= + =
|
+ +
\ .
+
Partial fraction expansion gives



( )
16 16 4 20 16
0.4 0.4
o
V s
s s s s s
= + =
+ +


Taking the inverse Laplace transform gives




( )
0.4
20 16 for 0
o
t
v t e t

= >
The value of the capacitor voltage 2 seconds after the switch closes is
( )
0.4 (2)
2 20 16 12.81 V
o
v e

= =
261

Das könnte Ihnen auch gefallen